■ちょっとした疑問はここへ書いてね(はぁと)6■

このエントリーをはてなブックマークに追加
1(=`Å´=)
         )
   , ―――'
 γ∞γ~  \     ____________
 人w/ 从从) )   /いちいちスレッド立てないで
  ヽ | | l  l |〃 <  ちょっとした疑問はここに書いてね。
  `wハ~ ーノ)    \              Part 6
   / \`「        ̄ ̄ ̄ ̄ ̄ ̄ ̄ ̄ ̄ ̄ ̄ ̄
書き込む前に>>2の注意事項を読んでね。
数式の書き方(参考)はこちら>>3
2(=`Å´=):01/11/22 19:08 ID:???
書き込む際の注意

1.)
板の性格上、UNIX端末からの閲覧も多いと推察されます。
機種依存文字(ローマ数字、丸数字等)は避けて下さい。

2.)
以下のような質問に物理板住人は飽き飽きしているので、たぶん無視されます。
しないで下さい。
「相対性理論は間違っています」
「量子力学は間違っています」
「宇宙論は間違っています」
「シュレディンガーの猫は変です」
「永久機関を作りました」

3.)
宿題を聞くときは、どこまでやってみてどこが分からないのかを書くこと。
丸投げはダメよ。

4.)
厨房を放置できない人も厨房です。
3(=`Å´=):01/11/22 19:08 ID:???
【掲示板での数学記号の書き方例】
●スカラー:a,b,c,...,z, A,B,C,...,Z, α,β,γ,...,ω, Α,Β,Γ,...,Ω, ...
●ベクトル:x=[x[1],x[2],...], |x>, x↑, vector(x) (← 混同しない場合はスカラーと同じ記号でいい.通常は(成分を横で書いても)縦ベクトルとして扱う.)
●行列(1成分表示):A[i,j], I[i,j]=δ_(ij)
●行列(全成分表示):A=[[A[1,1],A[2,1],...],[A[1,2],A[2,2],...],...]=[a1,a2,a3,...], I=[[1,0,0,...],[0,1,0,...],[0,0,1,...],...] (← ここでは列ごとに表示(縦ベクトルを横に並べる).行ごとに表示しても構わないが,統一して使わないと混同するので注意.)
●関数:f(x), f[x]
●数列:a(n), a[n], a_n
●足し算:a+b
●引き算:a-b
●掛け算:a*b, ab (← 通常"*"を使い,"x"は使わない.)
●割り算・分数:a/b, a/(b+c), a/(bc) (← 通常"/"を使い,"÷"は使わない.)
●複号:a±b=a士b, a干b (← "±"は「きごう」で変換可.他に漢字の"士""干"なども利用できる.)
●平方根:√(a+b)=(a+b)^(1/2) (← "√"は「るーと」で変換可.)
●指数・指数関数:a^b, x^(n+1), exp(x+y)=e^(x+y) (← "^"を使う."exp"はeの指数.)
●対数・対数関数:log_{a}(b), log(x/2)=log_{10}(x/2), ln(x/2)=log_{e}(x/2) (← 底を省略する場合,"log"は常用対数,"ln"は自然対数.)
●三角比・三角関数:sin(a), cos(x+y), tan(x/2)
●内積・外積・スカラー3重積:a・b=(a,b), axb=[a,b], a・(bxc)=(axb)・c=[a,b,c]=det([a,b,c])
●行列式・トレース:|A|=det(A), tr(A)
●絶対値:|x|
●ガウス記号:[x] (← 関数の変数表示などと混同しないように注意.)
●共役複素数:z~
●階乗:n!=n*(n-1)*(n-2)*...*2*1, n!!=n*(n-2)*(n-4)*...
●順列・組合せ:P[n,k]=nPk, C[n.k]=nCk, Π[n,k]=nΠk, H[n,k]=nHk (← "Π"は「ぱい」で変換可.)
●微分・偏微分:y', dy/dx, ∂y/∂x (← "∂"は「きごう」で変換可.)
●ベクトル微分:∇f=grad(f), ∇・A=div(A),∇xA=rot(A), (∇^2)f=Δf (← "∇"は「きごう」,"Δ"は「でるた」で変換可.)
●積分:∫[0,1]f(x)dx=F(x)|_[x=0,1], ∫[y=0,x]f(x,y)dy, ∬_[D]f(x,y)dxdy, 点[C]f(r)dl (← "∫"は「いんてぐらる」,"∬"は「きごう」で変換可.)
●数列和・数列積:Σ[k=1,n]a(k), Π[k=1,n]a(k) (← "Σ"は「しぐま」,"Π"は「ぱい」で変換可.)
●極限:lim[x→∞]f(x) (← "∞"は「むげんだい」で変換可.)
●図形:"△"は「さんかく」,"∠"は「かく」,"⊥"は「すいちょく」で変換可.

●その他
・関数等の変数表示や式の括弧は,括弧()だけでなく[]{}を適当に組み合わせると見やすい場合がある.
・ギリシャ文字はその読み方で変換可.
・上記のほとんどの数学記号や上記以外の数学記号"⇒∀≠≧≒∈±≡∩∽"などは「きごう」で順次変換できる.
4ご冗談でしょう?名無しさん:01/11/22 19:10 ID:???
>>1
使ってくれてどうもありがと
5:01/11/22 19:16 ID:Hj4QpSz9
今日学校に行かず独学している僕に教授が、・君は大学を舐めている。これから色々難しくなっていくんだから、独学だと難しくて試験に通るのが難しくなる。・と言われました。今一年です。何か勉強姿勢に対しアドバイス下さい
6>1:01/11/22 19:15 ID:???
過去スレ
Part1:http://cheese.2ch.net/sci/kako/968/968116983.html
Part2:http://cheese.2ch.net/sci/kako/981/981661589.html
Part3:http://cheese.2ch.net/test/read.cgi/sci/991491349/
Part4:http://cheese.2ch.net/test/read.cgi/sci/997037229/
Part5:http://cheese.2ch.net/test/read.cgi/sci/1003082106/

姉妹スレ
[★先生や友達にききにくい質問はここに書け!★]
http://cheese.2ch.net/test/read.cgi/sci/993274148/
しつこく生き残ってしまった重複スレ
[★物理板版解らない問題はココへ書いてね★]
http://cheese.2ch.net/test/read.cgi/sci/1001430417/
7ご冗談でしょう?名無しさん:01/11/22 19:19 ID:???
次は過去スレにもリンクを張る形で一つ…
8(=`Å´=):01/11/22 19:23 ID:???
sorry...
9ご冗談でしょう?名無しさん:01/11/22 19:48 ID:???
あとは類似のスレと過去スレにそれぞれ誘導リンク貼ればいいんかな
10素人:01/11/22 20:04 ID:???
早速質問です。

原子核の励起状態には、それぞれ対応するエネルギーと、下位レベルに落ちるまでの
Decay time(life time)があると思うのですが、このLifetimeに関連してEnergy width
があると聞きましたが、いまいちよく分かりません。
例えば、8^Beの第一励起状態(J^π=2^+)は基底状態から2.9MeV上にあって、
3.0MeVのEnergy width(FWHM)があるらしいのですが、この3.0MeVというのは
どういう風にLifetimeに換算出来るのでしょうか?
11東工大一年:01/11/23 05:30 ID:cre0GEjg
半導体の抵抗率の式、f(T)=f(∞)exp(Q/kT)について
なぜこのような式になるのか、だれか簡単に説明していただけませんか?
キャリア濃度がexp(Q/kT)に比例するのというのはわかったのですが
そこから先が・・・
12ご冗談でしょう?名無しさん:01/11/23 05:42 ID:GEyCqINo
>>11
キャリアの変化以外の項は小さすぎるので無視できるから。
13ご冗談でしょう?名無しさん:01/11/23 13:20 ID:FgF5qDZ6
>>10
ΔEΔt〜h
14さん:01/11/23 14:53 ID:9764csoV
実証物理ではノーベル賞は難しいのは本当ですか?
カミオカンデでニュートリノの質量を見付けたみたいだが
ノーベル賞は取れるんですか?
15小柴:01/11/23 17:26 ID:???
俺が知りたい
16Bacall:01/11/23 18:42 ID:???
俺も知りたい。
太陽ニュートリノでノーベル賞取れるの?
17ご冗談でしょう?名無しさん:01/11/23 23:27 ID:tzDV2dEt
落下中の物体(投身自殺してる人の体とします)は
体重が増えているといえるのですか?
18ご冗談でしょう?名無しさん:01/11/24 01:08 ID:woe0dyk6
あのねえ
お宅んとこの駄犬がうちの庭に穴掘って困ってるんだけど。
http://mentai.2ch.net/test/read.cgi/kikai/1006403824
薬殺の義務があるはずだよ。
19高校生:01/11/24 01:46 ID:saqe6LTo
本当に高校生っぽい質問で申し訳ありません。
どうしてもわからないので教えてください。

地球からある方向へ光速の2/3の速さで宇宙船Aを飛ばし、
また、正反対の方向に光速の2/3の速さで宇宙船Bを飛ばすと
宇宙船Aからの宇宙船Bの相対速度は光速を越えますよね?
その場合、宇宙船Aからみた宇宙船Bの時の流れはどうなるのですか??
20ご冗談でしょう?名無しさん:01/11/24 01:50 ID:t6ftoixG
>宇宙船Aからの宇宙船Bの相対速度は光速を越えますよね?

超えないんだってば。相対論の本を読め。
21>:01/11/24 02:39 ID:j7yIYZK5
空気の窒素、酸素などの気体の分子が
重力によって地面におちてつもってこないのは
何故ですか?
と子どもに聞かれたらどう説明する。
22ご冗談でしょう?名無しさん:01/11/24 02:46 ID:???
>>21
積もってるじゃねーか、と答えるよ。だから空高く上がっていけば空気は
なくなるじゃねーか、とな。それがなにか?
23ご冗談でしょう?名無しさん:01/11/24 03:41 ID:???
何でつぶれない
24ご冗談でしょう?名無しさん:01/11/24 04:17 ID:???
えらくバイナリな世界を考えてないか?
25々々々:01/11/24 05:07 ID:???
>>21
太陽(その他の熱源)が空気分子を暖めているから。
 常温25℃において空気分子は平均速度毎秒0.5キロ(計算正しい
かな?)の猛スピードで走りながら、壁とか地面とか、別の空気分子に
ぶつかって跳ね返っているので、地面に積もることはない。空気分子を
真空中で自由落下させて積もらせるためには、絶対ゼロ度(-273℃)付近
まで冷却しなければならない。空気(窒素ガス)の場合は、その前に
-196℃付近で氷になりますが。
26ご冗談でしょう?名無しさん:01/11/24 05:40 ID:???
>気体の分子が
>重力によって地面におちてつもってこないのは
>何故ですか?

落ちて積もってるでしょ。
大気圧は高度が下がれば高い。上空に行くほど大気は薄くなる。
27ご冗談でしょう?名無しさん:01/11/24 08:22 ID:???
質量って何ですか。
28ご冗談でしょう?名無しさん:01/11/24 08:48 ID:???
>>27
力/加速度
29ご冗談でしょう?名無しさん:01/11/24 09:12 ID:O2vBMtE0
>>27
四元運動量ベクトルの長さ
30ご冗談でしょう?名無しさん:01/11/24 09:25 ID:???
>>26
それを「積もってる」とは普通言わない。
少なくとも子どもへの説明では混乱を生むだけ。
31ご冗談でしょう?名無しさん:01/11/24 11:07 ID:???
>>26
子供の年齢、程度によるのでは?
気体なりに積もっていることを理解しないと、やはり誤解する。
分子運動と体積を理解させるとよろし。
32名ご冗談でしょう?名無しさん :01/11/24 12:13 ID:wcjLgmjX
中学で出た問題です。
寒くて、思わず手をこすり合わせた。
そのとき、手はエネルギーの変換を行っている。
○か×か?
教えてください。
33ご冗談でしょう?名無しさん:01/11/24 12:14 ID:???
熱って何ですか。
34ご冗談でしょう?名無しさん:01/11/24 13:05 ID:???
愛って何ですか。
35 :01/11/24 13:06 ID:eucY0YIn
すべての現象は次の瞬間の現象になんらかの影響を与えている
っていう考え方なんて名前でしたっけ?
まじでわからん助けて…「〜理論」だったはず
36ご冗談でしょう?名無しさん:01/11/24 13:11 ID:t6ftoixG
>>35
てゆーかそれってすげー普通。超当たり前のことを言ってるだけなんですけど。

ひょっとして君が言っているのはカオス理論のことか?だったら全然話は
違うぞ。
37ご冗談でしょう?名無しさん:01/11/24 18:27 ID:???
>>17
kgなら変わらないけどkg重ならゼロということでいーんじゃない。
38ご冗談でしょう?名無しさん:01/11/24 18:33 ID:qjd4RnOh
>>17
体重の定義は?
39ご冗談も顔だけ?脳無しさん:01/11/24 18:37 ID:4vAimWOO
>>17
他スレの方が伸びるのもイヤなので、バカなりに回答します。

>落下中の物体(投身自殺してる人の体とします)は
>体重が増えているといえるのですか?

(1)落下物の「質量」は静止時と変わらないよ。
(2)いや、地上基準で静止時より落下中の方が物体の全エネルギーは多いはずだ。「相対論的質量」は増えている。
(3)いやいや、この場合相対論は関係ない。体重計に落としてみろ。「重量」は増えただろ。

どれを選ぶかはあなた次第!        
40ご冗談でしょう?名無しさん:01/11/24 18:45 ID:???
>>39
等価原理を考慮して、加速度系における自然な体重測定法を定義すべきだ。
もうあるのかも、または自明なのかもしれんが。
41ご冗談も顔だけ?脳無しさん:01/11/24 18:57 ID:4vAimWOO
(4)でも、落下中のエレベータの中で体重計に乗ろうにも乗れないのでは?「体重計」はゼロ?
42ご冗談も顔だけ?脳無しさん:01/11/24 19:06 ID:4vAimWOO
>>41>>39に追加したい項目でした。
43縞栗鼠(シマリス)の親方:01/11/24 19:12 ID:YVfqXaBG
吉祥寺にある大検・大学受験予備校の中央高等学院
ここは、完全に狂ってる。
授業料は一年分一括前払いなので、
金が入れば、生徒は要らない
金を振り込んだら、何とかその生徒を辞めさせようと
講師どもが、あの手、この手でイヤガラセをしてきますね。
セクハラはもちろん、脈絡の無い罵倒は日常茶飯だね。
酒臭い講師もいるし・・・ 人生の最果て中央高等学院
http://www.chuo-school.ac/
学歴詐称 経歴詐称 デタラメ授業
http://chs-f.com/index.html 福岡校
44ご冗談でしょう?名無しさん:01/11/24 19:51 ID:Fg0ZaMco
ラグランジュの未定乗数法のよい教科書・参考書を教えてください。
できれば基礎からわかるものを。
45 :01/11/24 20:00 ID:???
ついさっき爆笑問題が出てるTV番組でUFOの原理がどうとかいうコーナー
があってハチスン理論について興味を持ちました。
皆様の見解を述べてください。
46理学部:01/11/24 20:07 ID:QC8XiGrc
>>44
数ページだけど、ライフの「統計熱物理学の基礎(上)」の
最後に載っている付録のが良かった。
47ご冗談でしょう?名無しさん:01/11/24 20:52 ID:haNeKW1H
化学反応で、価電子以外の電子が関係してくるのはどんな場合ですか?
48ご冗談でしょう?名無しさん:01/11/24 22:36 ID:CBpsyMKK
この写真の真中の上のかわいらしい現象は何と呼ぶのがふさわしいでしょうか
http://www.key3media.co.jp/broadband/arena.html
49ご冗談でしょう?名無しさん:01/11/24 23:08 ID:???
騙しとトリックの理論 >>45
50ご冗談でしょう?名無しさん:01/11/24 23:15 ID:bFChLepA
物理板のトップにいる7:3分けは誰?
51ご冗談でしょう?名無しさん:01/11/24 23:36 ID:zaugjzXn
倉木だろう
52:01/11/25 02:06 ID:mJyeHngv
すみません!悩んでます。何故地表近傍での運動方程式にW×(W×R)の項が消えるのですか?そもそもどのように運動を考えたら良いのでしょうか?
53ご冗談でしょう?名無しさん:01/11/25 02:23 ID:???
>>52
>W×(W×R)
一般的な成分で書き下してみろ
54学習塾講師(数学・理科):01/11/25 02:25 ID:unD4Cmb6
前スレに書いちゃったみたいなので、ここにコピペします。
スンマセン。
------------------
スンマセン。ちょっと教えてください。
自分が何を勘違いしてるか、わからなくなりました。
中学理科の、仕事とエネルギー分野の「杭と地面の摩擦力」を出す問題です。

【問題】杭の頭から10mの高さから、10kg重のオモリを落とす。
    杭は、地面に2mめり込んで止る。杭と地面の間の摩擦力は?
【解答】杭にぶつかる瞬間の運動エネルギーは、
    10kg重×10m=100kg重m
    摩擦力をXkg重として、
    Xkg重×2m=100kg重m
    これを解いて、X=50kg重 が答えだと思う。

ここで悩んでしまった。その時、オモリは「さらに2m、落下してる」。
そのとき失われた位置エネルギーはどこへ行ったのか???

それとも、Xkg重×2m=10kg重×12mなのか?

ああ、オレって、、、塾講師なのに、、、工学部機械工学科出身なのに、、
55一年:01/11/25 02:26 ID:DUk+MquD
検流計の電流感度1μA/DIVの
DVIって何ですか?ヒントください!
56ご冗談でしょう?名無しさん:01/11/25 02:35 ID:1WH4S6n5
>>54
重りは最初に比べて12[m]低い位置にあるので、120g[J]失ってます。
57ご冗談でしょう?名無しさん:01/11/25 02:36 ID:???
divはオシロの1メモリ。
実験でスケールメモって無かったならアウトだな

dviはtexの中間ファイル
58学習塾講師(数学・理科):01/11/25 02:36 ID:unD4Cmb6
Xkg重×2m=10kg重×12m
これを解いて、X=60kg重 でいいのかな?
そしたら、問題集の【解答】が間違ってることになる。

眠い、、、寝るか、、、。
明日、本屋で参考書&問題集を立ち読みしてこよ。
59学習塾講師(数学・理科):01/11/25 02:39 ID:unD4Cmb6
>>56
ありがとう。
その失われた位置エネルギーが、全部、仕事に使われたら、、
答えは、60kg重でいいのか。。。

すると、問題集の【解答】が間違えていて、今日質問した生徒は、、
月曜日、期末試験だ、、、どーしよ、、、
60ご冗談でしょう?名無しさん:01/11/25 02:43 ID:???
>>59
明日電話
61ご冗談でしょう?名無しさん:01/11/25 02:46 ID:q72khRne
2桁の自然数のうち、次のような数はいくつあるか。
・各位の数字の積が偶数。

この答えを出す過程を教えてください。
答えは、65個なのですが、計算過程が・・・
高一レベルですが、お願いします。
62ご冗談でしょう?名無しさん:01/11/25 02:51 ID:???
>>57 ワラタ
63ご冗談でしょう?名無しさん:01/11/25 02:53 ID:1WH4S6n5
>>61
10〜99の90個のうちから、各桁が奇数であるものの個数を引けばいいと思います。
(偶数が一個でも掛かると全体の積は偶数になってしまうから)
奇数は1、3、5、7、9なのでこれの組み合わせは5×5=25個。
よって90-25=65個。
64学習塾講師(数学・理科):01/11/25 02:54 ID:???
くそーーーーー!!!
問題集の【解答】を作ったやつ、誰ダーーーーーー?!?!?!
ヽ(`Д´)ノ ゴルァ! 物理・院生のバイトだろーーーー!!
ちきしょーーーーー。
寝るーーーーー!!!!!
65ご冗談でしょう?名無しさん:01/11/25 02:56 ID:???
>>61
ちなみに板違い。
66一年:01/11/25 02:58 ID:DUk+MquD
>>57
ありがとうございます!

divで思いついたのは、59さんのtex-divとダイバージェンスdivとHTMLのDIV要素。
67一年:01/11/25 03:00 ID:DUk+MquD
訂正 tex-dvi
68ご冗談でしょう?名無しさん:01/11/25 03:19 ID:???
しかも57
69一年:01/11/25 04:01 ID:DUk+MquD
>>68
失礼!
70リコネクション:01/11/25 04:51 ID:rdFr00/q
U型磁石がふたつあります。向かい合わせて少し離しておいてあります。⊂ ⊃
S極N極はお互いにくっついていく位置関係です。

今、それぞれの磁力線はそれぞれのN極S極をつなぐ半円状になっています。⊂)(⊃
近づけていきます。
最終的にはお互いの極どうしを結びあうブリッジ状になります。⊂⊃

そうなるまでの間に磁力線はつなぎかわっとるんか???
つなぎ変わる瞬間はとんな形しとるんじゃ??

おしえろゴルァー
71ご冗談でしょう?名無しさん:01/11/25 05:51 ID:vsDk0UAZ
>>71
砂鉄蒔いて自分で見れ
72ご冗談でしょう?名無しさん:01/11/25 06:46 ID:???
おぅ、早くやってくれ>>71
7371:01/11/25 06:49 ID:???
しまった。砂鉄蒔くのはもちろん>>70
7470:01/11/25 07:12 ID:rdFr00/q
お前ら仏痢やってんだろが、理論から考えてけれや
磁石も砂鉄も持ち合わせておらんので
お願いします、今日はこれで徹夜してしまいました
教えてください
7571:01/11/25 07:33 ID:???
>>74
物理は実験が基本。知らなかった?
76ご冗談でしょう?名無しさん:01/11/25 07:35 ID:???
磁力は線じゃないから。
77ご冗談でしょう?名無しさん:01/11/25 07:36 ID:???
>>74
どうやってここに書けというんだ?

とりあえず正方形の4点に交互に正負のモノポールを
配置した時の磁力線は分かるだろう。
その辺を引き伸ばしたり圧縮したりする事を考えればよい。
78 :01/11/25 08:38 ID:XCTBxSC/
>>36
カオス理論に付随してたような気が…
>>35の続きなのですが、よって分子単位でシミュレートするとすべての事柄
が予測できるとかなんとか…あー気になる
79ご冗談でしょう?名無しさん:01/11/25 08:46 ID:???
>>78
だからそれは当たり前だってば
8070:01/11/25 09:26 ID:rdFr00/q
いや、だから、電磁気学だったら磁力線はそれ自身で閉じるか
無限遠まで伸びるかだよね?2本の磁力線が交わってつなぎ
かわるのってありえないでしょ?でも磁力線のモロホロジーは
前と後で変わってるからどこかでつなぎ変わってんじゃん。
どうよ?
81ご冗談でしょう?名無しさん:01/11/25 14:11 ID:???
線といっても仮想的なもんだし、
瞬間的につなぎ変わるってことでいいんじゃない?
82ご冗談でしょう?名無しさん:01/11/25 15:38 ID:???
どなたか、群速度と位相速度の違いを丁寧に教えてください。
とくに、位相速度が光速を超えることがあるってそんな事が
あるのですか?
83ご冗談も顔だけ?脳無しさん:01/11/25 19:31 ID:WBkHAtfw
>>82
落第したオレの回答でいい?

「位相速度」は単一の振動数の波(つまり混じりけのない正弦波)が進行する速度。
(物が移動しているわけではないので、進行する、と言う表現はマズイか・・・)
「群速度」は混じり合った波が作る、振幅の大きな部分のかたまりや振幅の小さな部分
のかたまりが進行する速度。

で、突然「位相速度が光速を超えることがあるってそんな事があるのですか?」
とか言われても・・・。
どういった流れでその話が出たか、前後のお話も書いてくれませんか?
84ご冗談でしょう?名無しさん:01/11/25 20:14 ID:???
>>78
ひょっとしてラプラスの悪魔の話か?だとしたら、そうとう間違った把握の
しかたをしてるような気がする。
85:01/11/25 20:54 ID:urRBcTGf
量子力学で超微細相互作用定数Aの持つ意味について教えて下さい。
A=gμΔHのAのことです。
86ご冗談でしょう?名無しさん:01/11/25 21:47 ID:LUAuImH8
質問

galaxyを銀河と訳したのは日本人だと思いますが誰ですか。
87ご冗談でしょう?名無しさん:01/11/25 22:12 ID:???
>85
Aは、核スピンと電子スピンの相互作用です。
88ご冗談でしょう?名無しさん:01/11/25 22:21 ID:QeBV3wWf
>>78はマルコフ過程のことか? とも思ったが、何か違うな。
モンテカルロは理論の名前じゃないし。
89ご冗談でしょう?名無しさん:01/11/26 00:26 ID:XcVSvBM9
極座標であらわされたベクトルの外積や内積って
どのように表されるんでしょうか?
(r1,θ1,φ1)×(r2,θ2,φ2)=?
極座標の回転とか発散とかはよくでてくるんだけど、
外積内積ってあまりみないような。
90ご冗談でしょう?名無しさん:01/11/26 01:04 ID:IUuErJCQ
磁場の大きさをあらわす単位Oeと
MKS単位系でのAmpere/mの関係を教えてください。

あと、磁場の大きさで(G)と書かれた文献があるのですが、
どういう意味でしょうか?
91理学部:01/11/26 02:32 ID:Nbmifop5
>>89
ん? (r*cosφ*cosθ,r*cosφ*sinθ,r*sinφ)とすれば、
あとはそのままじゃない?
92たかぴょん:01/11/26 02:33 ID:kcKPhF6W
物理実験で伝播誤差を計算することになったのですが
R=exp(a)
という式では、偏微分して求めるやり方と、
普通にexp{a(max)}とexp{a(min)}を計算したのでは
答えが違います・・・どちらをとればいいんでしょうか?
93ご冗談でしょう?名無しさん:01/11/26 02:43 ID:???
>>89
分配法則とか使って計算すりゃ一発じゃん
94ご冗談でしょう?名無しさん:01/11/26 02:44 ID:???
>>90
Oeって単位、初めて見た。
Gはガウスでしょう。
95工学生:01/11/26 02:45 ID:NgLyfO69
全微分みたいな式を使った場合では,パラメータの微小な変動に
対しては精度がよいかもしれませんが,大きな変動の場合には
誤差が出てくると思います.より高次の式で誤差の
計算を行うのが良いかも知れません.
96たかぴょん:01/11/26 02:55 ID:kcKPhF6W
>>95
アドバイスありがとうございます!
97ご冗談でしょう?名無しさん:01/11/26 03:00 ID:CYWqRUwV
Oeはエルステッドでしょう。
次元はガウスと同じで扱いはA/mと同じ。(いわゆるH場)
98 :01/11/26 05:51 ID:???
>>84
検索したところラプラスの悪魔の話でした…
しかもかなり間違った覚え方してました…欝
記憶は極性化されるものであるということなんで勘弁してちょ
9989:01/11/26 15:35 ID:3yoZWLq0
そういえばたしかにそのまんま計算できますね。
直交座標にして外積をとって極座標に戻すんだけど、
極座標に戻すときに三角関数の逆関数が出てきてすごく複雑になりそう。
(r1,θ1,φ1)×(r2,θ2,φ2)=(r3,θ3,φ3)
のθ3とかをr1,θ1,φ1,r2,θ2,φ2でうまくあらわせるのかなぁと
思いまして。
あらわせなさそうですね。
100ご冗談でしょう?名無しさん:01/11/26 15:44 ID:???
力学で極座標習うあたりでやるだろその計算>99
101ご冗談でしょう?名無しさん:01/11/26 19:49 ID:???
>>90
>磁場の大きさをあらわす単位Oeと
> MKS単位系でのAmpere/mの関係を教えてください。
> あと、磁場の大きさで(G)と書かれた文献があるのですが、
> どういう意味でしょうか?

Oe は、>>97 のいうとおり、H(A/m)を表す時の単位。昔々おられた
エルステッド先生に因んでつけられている単位。

一方 G(gauss) は、磁束密度 B を表したり磁場 H を表したり色々。
cgs 系ではμが1だったのが原因だったはず。MKSA 単位では、B
を表す単位になっている。
#ここ、今ひとつ自信がないので、識者の方のコメントをキボーン。

ちなみに SI で読み替えるときは、
H 1Oe → だいたい 80A/m
B 10,000 Gauss → 1T。
102ご冗談でしょう?名無しさん:01/11/27 02:54 ID:???
>>99
デカルト座標で結果が出ているなら、
普通に極座標への変換をすればいいのでわ
r=√(x^2+y^2+z^2)
θ=arccos(z/r)
φ=arctan(y/x)
103ご冗談でしょう?名無しさん:01/11/27 22:09 ID:???
1[fm] を 1[Yukawa] と呼ぶと聞いたのですが、
これは本当なのでしょうか?

もし本当ならば [Yukawa] とはどう表記するのでしょうか。
104ご冗談でしょう?名無しさん:01/11/28 12:17 ID:gOWqvU3F
>>103
ネタだと思っていたら本当にあったよ・・・
1[ユカワ] = 10^(-13)[cm]
だってよ・・・

おれ高卒サラリーマンのペーペーなので、どう表記するのかは知らない。すまんね。
105ご冗談でしょう?名無しさん:01/11/28 14:23 ID:5VWzds+F
>>103-104
すなおにフェムトメータと呼ぶのがいいのだが、
初めてfmオーダーの領域に切り込んでいった湯川の業績に敬意を
表してユカワと呼ぶのはまだよかろう。
しかしfmという字面につられてフェルミと呼ぶやつが
(日本人にさえ)ごろごろいるのは納得いかない
106ご冗談でしょう?名無しさん:01/11/28 20:41 ID:a6V80yeH
http://cheese.2ch.net/test/read.cgi/sci/985343079/
IFという言葉がこのスレでよく出てきますがどういう意味ですが
107ご冗談でしょう?名無しさん:01/11/28 23:04 ID:wUjd3UYs
アニオンの水和自由エネルギーが分かりません。
具体的には、Cl-、Br-、I-、NO3-です。
googleで一生懸命検索したのですが、わかりませんでした。

これは、まだ分かっていないことなのでしょうか?
どなたか、助けてください・・・
108ご冗談でしょう?名無しさん:01/11/28 23:20 ID:hvLLHtY6
スレ違いかも知れないですが、一応ここでお訊きします。
物理学者の寺田寅彦って、今の物理学やっている人たちにとって、
どれくらいの存在でしょうか?
誰それレベルなのか、基本だぞレベルなのか。
文学版の方から来ましたんで、よくわかりません〜。ホントすみません!
109ご冗談でしょう?名無しさん:01/11/28 23:59 ID:???
110ご冗談でしょう?名無しさん:01/11/29 11:46 ID:qAnx/y8F
>>109
妙にうすっぺらいな(わら

>>108
個人的には基本だぞレベル
111ご冗談でしょう?名無しさん:01/11/29 12:18 ID:???
>>108
誰それ?
112ご冗談でしょう?名無しさん:01/11/29 15:24 ID:???
この板の名無しさんの「ご冗談でしょう?」というのは、ファインマンさんのことですよね?
113 :01/11/29 18:06 ID:HGr78yto
そうです。
114ご冗談でしょう?名無しさん:01/11/29 18:39 ID:5G3LyHNX
曲線座標で
Q_1(x,y,z)=q_1
Q_2(x,y,z)=q_2
Q_3(x,y,z)=q_3
としたとき

x=X(q_1,q_2,q_3)
y=Y(q_1,q_2,q_3)
z=Z(q_1,q_2,q_3)
と出来たときの
g_ij=
∂X/∂q_i * ∂X/∂q_j
+∂Y/∂q_i * ∂Y/∂q_j
+∂Z/∂q_i * ∂Z/∂q_j

h_i = √(g_ij) = √{{∂X/∂q_i)^2 +(∂Y/∂q_i)^2 +(∂Z/∂q_i)^2}

としたときのg_ijとh_iってなにを意味するものなのでしょうか?

hの方は長さっぽい感じがしますが、何の長さなのか。
ちなみに直角座標ではg_ij = 0, h_i = 0
円柱座標ではg_ij = 0 h_1=1 h_2=ρ h_3=1
旧座標では g_ij = 0 h_1=1 h_2=r h_3=rsinθ
となるようです。(g_ij はi≠jで)
115ご冗談でしょう?名無しさん:01/11/29 21:26 ID:i8YnqRFc
2つの物体が衝突するとき、エネルギーは保存されず、運動量は保存されるのはどうして?また、運動量が保存されない場合ってあるんですか?
116ご冗談でしょう?名無しさん:01/11/29 21:33 ID:???
>>115
は? エネルギーも運動量も保存されますが?
117K.O:01/11/29 21:37 ID:xldr8Cv6
>>115
>エネルギーは保存されず、運動量は保存されるのはどうして?

逆。
運動量は保存されず、エネルギーは保存される。

運動量が保存されないのは、非弾性衝突の時。
運動エネルギーが物体を変形させる力や熱エネルギーに変換されるから。
118ご冗談でしょう?名無しさん:01/11/29 21:44 ID:???
>>117
おいおい…自分で書いてておかしいと思わないのか?
119ご冗談でしょう?名無しさん:01/11/29 21:46 ID:???
>>117
>運動量は保存されず、エネルギーは保存される。
>運動量が保存されないのは、非弾性衝突の時。
うそつけ
120ご冗談でしょう?名無しさん:01/11/29 22:11 ID:A7cz8zIn
フィラメントから電子が出るのですが、
加速電圧(例えば1V)ってどういう意味でしょうか?

1eVのエネルギーをもった電子が出てくると思うのですが、
その1Vの電位差はどこから来るのですか?
121K.O:01/11/29 22:14 ID:xldr8Cv6
スマソ。

ちょうど落下してきた物体が地表に衝突する問題を考えていたので
とっさに出てしまった。

訂正。
「自由に動ける」物体同士の衝突では運動量は保存される。
非弾性衝突の際、「力学的」エネルギーは保存されないが、
熱エネルギーなどに変換されるため、系全体のエネルギーは保存される。

>>118 >>119
指摘サンクス。
122ご冗談でしょう?名無しさん:01/11/29 22:18 ID:i8YnqRFc
>>116〜121
どうもありがとう。
123K.O:01/11/29 22:21 ID:xldr8Cv6
>>115

現実の世界を考えた時には、固定の地表など無いので
運動量は常に保存される。
124ご冗談でしょう?名無しさん:01/11/29 22:40 ID:i8YnqRFc
運動量って結局何なんですか?例えばパンチした時の拳の運動量のもとは
自分自身のエネルギー(この場合カロリーといった方がいいかな)ですよね。
だから運動量とエネルギーはもともとは一緒のものなんでしょうかね?
運動量っていうのは、あれば便利だな〜って感じにただ導入しただけのものなんですか?
もっと言うと、質量や速さや力や電気量や熱などの物理量もエネルギーと同一のものなんでしょうね。多分。
125ご冗談でしょう?名無しさん:01/11/30 08:31 ID:e9eOl5Hb
>>106誰か知りませんか
126ご冗談でしょう?名無しさん:01/11/30 18:06 ID:xV4x3ijM
>>124
ニュートンの時代に、仕事はmvか?mv^2か?で揉めたとかなんとか・・・
127質問猿:01/11/30 20:18 ID:dlsI99uC
196X年、テイラー隊長ら優秀な科学者を乗せた宇宙ロケットがアメリカのケープケネディを後にした。
1年6ヶ月のあいだ宇宙を跳び続けた後、計器の故障で、地球より320光年離れた、オリオン座のとある惑星に不時着した。 宇
宙飛行中の時間差で地球時間では2000年の歳月のが経過していた。

・・・・・これって計算合ってるんですかね??
128名無しさん:01/11/30 20:55 ID:DfBjOavm
地震について

よくニュースで震源50kmとか伝えられるのですが、
もし震源が海上であったらこの震源は海面から50km
なのでしょうか、それとも海底から50kmなのでしょうか?
129 ◆AOHU.zVE :01/11/30 23:08 ID:mE6Crycz
凹レンズ、凸レンズ、凹面鏡、凸面鏡それぞれの光の屈折の仕方を作図していたら、
何か混乱してきてしまいました。
アドバイスや、いいHPがありましたら教えてください…。
130名無しさん:01/12/01 01:49 ID:hz4pnH3n
エアコンと断熱器の熱効率の違いってどう説明すればいいんですか?誰か教えて下さい!
131やりチン:01/12/01 03:28 ID:???
コイルの属性
          長さ l=0.03m、直径2r=0.108m、巻数N=68turn、
          真空中の透磁率μ_0=4π*10^(-7)H/m
長岡係数K=0,388、導線(銅)の直径d=0.0004m、
          銅の抵抗率ρ=1.712*10^(-8)Ωm

の時のコイルのインダクタンスLと直列抵抗R_Lを計算したいんですが、
R_Lの方が分かりません。
L=0.6885[mH]と出てきたんですが・・・

ここの人たちにしたら糞みたいなものだと思いますが、
教えていただけませんでしょうか?
132  :01/12/01 13:32 ID:3mD0lgqq
>>128
震源地が海上なら、地震じゃなく空震?
133量子力学:01/12/01 15:30 ID:880b4Q69
調和振動子、ばね定数K、質量m、が規定状態にあります。
時間t=0で、ばね定数が突然K+ΔKに変化
a)時間依存のシュレーディンガー方程式を用いて
t=0+における系の状態を求めなさい。

っていう問題なんですが、解けません。
どなたか教えて下さい。
134ばか:01/12/01 17:44 ID:uMaCPIYu
すみません。この科学手品のタネ明かしを教えて下さい。

ゴムシートの上の1円玉が、指でおすとゴムシートを通りぬけてビンの中に落ちてしまいます。
http://www.gakushu.net/magic/0112b/index.html
135ご冗談でしょう?名無しさん:01/12/01 18:08 ID:JZdWL107
光に時間はあるの?
136ご冗談でしょう?名無しさん:01/12/01 18:31 ID:JtpUa09j
空間がゆがむとそこにある物質はどうなるの?
137ご冗談でしょう?名無しさん:01/12/01 19:29 ID:???
>>133 どう解けないのか言え。
138134:01/12/01 20:56 ID:uMaCPIYu
ゴムシートなんてどこの家庭にでもあるものでは無い
と思うかもしれませんが
コンドームで代用してみて下さい。
本当に通り抜けてしまうんです。
139ご冗談でしょう?名無しさん:01/12/01 20:57 ID:???
>>138
コンドームでやってみましたが通り抜けませんでした・・・
使い古しでは駄目ですか?
140ご冗談でしょう?名無しさん:01/12/01 20:59 ID:???
>>138
自分でゆっくり動かして実験してみりゃいーじゃん。

>>136
重力を感じます。
141 :01/12/01 21:47 ID:???
表面張力におけるpHの影響とは・・

誰か詳しい人お願いします。
142133:01/12/01 22:02 ID:880b4Q69
>>137

|ψ(t)>=exp(iH´t/h)  ここでhはエイチバーの方
H´=H。+1/2*ΔkX^2
|ψ(t)>=exp(iωt/2) |ψ。>+exp(iΔkX^2/2h) |ψ。>

exp(iΔkX^2/2h) |ψ。>
この式が解けるのかなと・・

そこで
X^2=1/2([a+a+]+[aa+]+[a+a]+[aa])
を用いてexp項、テーラー展開して解くと
|ψ。>と |ψ2n>の項が最後に出てくるが、Σが |ψ2n>の項に
ついてきて、これをノーマライズするわけ??ってことで
解けない訳なんです。
多分,ほかに良い方法があると思うんですけど、わかりません。

宜しくお願いします。
143133:01/12/01 22:11 ID:880b4Q69
142 の書き込みで訂正です。

訂正前:|ψ(t)>=exp(iH´t/h)
訂正後:|ψ(t)>=exp(iH´t/h)|ψ(0)>
144名無しさん:01/12/02 00:26 ID:85UZRfJM
真空中にある半径Rの金属球にQ〔C〕の電荷を与えた。この金属の電気容量を求めよ。ただし,位置rにおいて電場がE(r)と表せる場合、電位Vは、V=∫E(r)drで与えられる。また電位の基準は無限遠とすること。

どうやれば良いのか分かりません。誰かお願いします。
145ご冗談でしょう?名無しさん:01/12/02 00:58 ID:ClJ4dorf
ふろなどに張った水を
栓をゆっくり抜くと時間がたてば必ず
時計回りに渦を巻くのはなぜなんでしょうか
どなたか理由が分かるかたおられますか?
146ご冗談でしょう?名無しさん:01/12/02 01:07 ID:oe96cLpp
>>145
別にどっち周りかは偶然だと思う。
渦ができるのは単に排水溝の形状の問題でしょう。

間違っても、よく言われるコリオリ力のせいではありません。
147ご冗談でしょう?名無しさん:01/12/02 01:51 ID:???
>>145
聞いたことある。
確か北半球が時計回りで南半球が半時計回りだね。
有名な話。
148ご冗談でしょう?名無しさん:01/12/02 01:56 ID:tkYbjQyn
>>147
有名なDQNです。風呂桶程度の大きさではっきり効果が現れるほど
コリオリ力は大きくありません。風呂桶が台風くらい大きければ別ですが。
149ご冗談でしょう?名無しさん:01/12/02 02:27 ID:???
>>142
調和振動子なんだからH'のまま普通に解けるのでは?
150物理演習レポート宿題:01/12/02 06:17 ID:???
2) 光子(電磁波)のボーズ・アインシュタイン凝縮の可否について、
適宜条件を与えた上で議論せよ。(A4紙1ページ程度)

すいません、教えて下さい。
151ご冗談でしょう?名無しさん:01/12/02 06:29 ID:/RZpGVdp
重力って空間の圧力だと最近思っているんですが、
おかしいかどうか教えてください。

ちなみに空間の圧力というのは、
物質が空間を張り、張られた物質の空間のために押しのけられた
空間が物質に対して反発している?
まあ水圧による浮力みたいな感じです。
152133:01/12/02 06:52 ID:WpyRBDt8
>>149
固有ベクトルと状態ベクトルは別。
単純な問題ではないと思われます。
153ご冗談でしょう?名無しさん:01/12/02 09:02 ID:q1O7Ojul
>>151
 おかしい
154疑問:01/12/02 13:41 ID:???
宇宙の膨張速度って、光速より速いんですか?
155ご冗談でしょう?名無しさん:01/12/02 13:45 ID:hQlX3MWM
>>154
 速い
156ご冗談でしょう?名無しさん:01/12/02 13:50 ID:tWjLgNfh
>>133
問題写し間違えてない?
>t=0+における系の状態を求めなさい。
t=0+における系の状態として、ばね定数Kに対する基底状態を使うっていうんじゃないの?
157133 :01/12/02 15:00 ID:WpyRBDt8
A harmonic oscillator, spring constant K, mass M is
in the ground state. At t=0, the spring constant is
suddenly to K+ΔK.

a) Use the time dependent Schrondinger equation to
find the state of the system at t=0+.

が、もとの文です。かなりいい加減に訳してましたでしょうか?
158ご冗談でしょう?名無しさん:01/12/02 15:48 ID:???
>>157
H'|ψ(t)>=i(d/dt)|ψ(t)>
H'=H+dH

|ψ(t)>=exp(-iH't)|ψ(0)>=任_m exp(-iE'_m t)|m>

|ψ(0)>=任_m|E'_m>=|E_0>, C_m=<E'_m|E_0>
じゃだめなの?
159154:01/12/02 15:47 ID:???
>>155
それはどんな理由から言えるのですか?
160ご冗談でしょう?名無しさん:01/12/02 16:18 ID:NpY585JZ
>>159
3K背景輻射と赤方偏移
161ご冗談でしょう?名無しさん:01/12/02 16:57 ID:zCe0cmwx
>>159
ハッブル定数が1/150億年 程度と考えられているので
150億光年より遠くにあるものは光速より速く遠ざかって
いるでしょう。
162ご冗談でしょう?名無しさん:01/12/02 18:06 ID:???
>>160
膨張していなくても同じ大きさの赤方偏移は出るぞ
163ご冗談でしょう?名無しさん:01/12/02 18:13 ID:609l3UEJ
>>160
 だから?
 背景輻射を上げてるでしょ
164ご冗談でしょう?名無しさん:01/12/02 18:53 ID:???
>>163
宇宙の底が3000Kってことだけで、輻射と膨張とは関係無いってこと
165ご冗談でしょう?名無しさん:01/12/02 19:05 ID:NS9QRi6A
>>164
底って何?
166ご冗談でしょう?名無しさん:01/12/02 19:16 ID:6Rpub3DO
宇宙構造の種ってなんですか?
167ご冗談でしょう?名無しさん:01/12/02 20:11 ID:XQkS7kvF
ブランク定数をh〔Js〕、電子の電荷をe〔C〕,電子の質量をm(kg)
とするとき、原子核の周りを回る最も速い電子の速度と,量子数(n)に応じた特性光の波長(λ〔m〕)はどうやって求めればよいのでしょうか?
168ご冗談でしょう?名無しさん:01/12/02 20:23 ID:???
>>164
???
それはあんたの自説でしょ?<関係ない
169ご冗談でしょう?名無しさん:01/12/02 20:34 ID:???
「空間が膨張している」なんて言われて簡単に納得すんなよ
170ニュー速板より:01/12/02 20:38 ID:???
442 :  :01/12/02 20:13 ID:6mszCjLX
えっとね、宇宙のずっと遠くの星から地球まで、長い長い一本の棒が繋がってるの。
長さが何光年もあるすごく長い棒ね。とても頑丈だから、ちぎれたり折れたりはしない。
その棒を、地球で、押したり引いたりしたら、相手の星でも、棒の先が動くはずだよね。
つまり、それで通信をしたら、光よりも速く情報が伝わらない?
電波や光でも、何年もかかるような遠いところなのに、長い棒を使ったら、
すぐにできちゃうんじゃないのか、ということ。
どう?
171ご冗談でしょう?名無しさん:01/12/02 20:38 ID:???
>>164
3000K・・・
172ご冗談でしょう?名無しさん:01/12/02 20:40 ID:3EDJpUy+
>>170
 愚問
173154:01/12/02 20:43 ID:???
すいません・・背景輻射ってなんですか?
赤方偏移は分かるのですが・・
174ご冗談でしょう?名無しさん:01/12/02 20:45 ID:???
>>170
激しく既出。原子と原子の間に働く力は電磁力であり、それが光速より
速く伝わることはない。
175ご冗談でしょう?名無しさん:01/12/02 20:47 ID:???
>>173
ビッグバンの名残。遠くを見るほど過去を見ることになるのはわかる?
非常に遠くを見れば、宇宙が生まれたとき(正確には真空が光を通す
ようになった「晴れ上がり」のとき)の様子が見える。当然宇宙のどの
方向を見てもそのときのエネルギーが見える。それが背景輻射。
176170:01/12/02 20:55 ID:lhBpXg33
>>174
なるほど。さんきゅ。
177ご冗談でしょう?名無しさん:01/12/02 20:58 ID:CFzCcflj
Directed Percolationの浸透確率の臨界値は
有理数ですか 無理数ですか?
178 :01/12/02 21:04 ID:3EDJpUy+
>>177
 数学へ
179ご冗談でしょう?名無しさん:01/12/02 21:08 ID:???
弱い力って、β崩壊を引き起こす以外はどんな働きをしますか?
180ご冗談でしょう?名無しさん:01/12/02 21:26 ID:u4bD/GIn
PDPの動作原理の話で出てくる放電って、
希ガスの原子が電離するという事ですか?
181ご冗談でしょう?名無しさん:01/12/02 21:54 ID:3EDJpUy+
>>179
 それくらいでしょ。
182154:01/12/03 00:06 ID:???
>>175
 なるほど・・背景輻射がなんなのかは分かりました。
 でもですね、膨張速度が光速より遅くても、その宇宙の端とも言うべき
 ところに光が溜まって、同じような現象が見られませんか?
 見当違いな事言ってたらすいません。
183ご冗談でしょう?名無しさん:01/12/03 00:42 ID:???
は、はぁ?
184ご冗談でしょう?名無しさん:01/12/03 01:43 ID:???
>>179
アホの181は放っておいて。
弱い力というのは粒子を変身させる力で、
素粒子の崩壊は大抵これの作用した結果として起こります。
例えば荷電K中間子のπμν崩壊とか中性K中間子のπ0π0崩壊とか。
(これらの場合、本質的にはsクォーク→dクォーク+W粒子)
Wが電子と電子ニュートリノに崩壊するとβ崩壊として見えるわけですな。
185184:01/12/03 01:48 ID:???
あう、
×dクォーク
○uクォーク
です
186175:01/12/03 02:13 ID:???
>>182
うーん・・・宇宙の果てにはなんか壁みたいなものがあると思ってるわけね。
素朴なイメージだなあ・・・

膨張速度が光速未満ということは、必然的に時空の曲率が正であり、
つまり宇宙は閉じていることになると思うんだけど(詳しい人、間違ってたら
指摘きぼん)、その場合われわれが「球の内部にいる」というイメージで
とらえちゃいけないのよ。

銀河も星も我々も、宇宙に存在する物質はすべて球の表面にいるわけ。
表面に沿ってしか移動できず、球の中にも外にも動けない。これだと、
どこにも「光子が溜まる」ような場所はないでしょ。

ただし、このイメージは実際の宇宙よりも1次元少ない。我々は球の表面
という曲がった平面ではなく、超球に沿って曲がった空間の中にいること
になる。

で、最近の観測によれば宇宙はどうやら開いているらしいんだけど、この
場合どこかに端のようなものがあるのかどうか俺にはわからない。あると
しても、それは光速よりも速く遠ざかっているから観測不可能ではある
はず。このへんも誰か解説してほしいなあ・・・
187ご冗談でしょう?名無しさん:01/12/03 07:27 ID:hxzo/6UZ
>>179
ニュートリノが絡むほぼ全ての反応。(標準模型の範囲内で)
188ご冗談でしょう?名無しさん:01/12/03 07:44 ID:???
>>187
じゃさ、弱い力によらないニュートリノ反応って何よ?(純粋に興味
189ご冗談でしょう?名無しさん:01/12/03 07:46 ID:???
>>184
Z交換など、粒子が変身しないこともある

>>187
ニュートリノが絡まないこともある
逆にニュートリノが絡むなら必ず弱い相互作用(あ、重力もあるか)
190ご冗談でしょう?名無しさん:01/12/03 09:30 ID:???
そこまで分かってるのなら、弱い力という名称をなんとかしてもらいたいものだ。
191ご冗談でしょう?名無しさん:01/12/03 11:07 ID:???
なんで?
192高一生:01/12/03 11:56 ID:QKcQZp97
来年、物理を選択しようと思ってるんですが、やっぱ難しいですか?(僕の高校では平均が20点位らしい・・・)
193ご冗談でしょう?名無しさん:01/12/03 11:59 ID:???
難しいかどうかは知らんが、楽しいよ
194ご冗談でしょう?名無しさん:01/12/03 11:59 ID:???
>>192
あきらめて文系にしましょう.
195高一生:01/12/03 12:05 ID:QKcQZp97
あきらめるって、べつに理系科目が苦手なわけではないですよ。むしろ得意です。
196高一生:01/12/03 12:06 ID:QKcQZp97
あと理系では、英語がサブ科目ってホント?
197ご冗談でしょう?名無しさん:01/12/03 12:07 ID:???

>>196
サブ科目ってなんだ?
一般的な言葉じゃないから言い換えろ。
198ご冗談でしょう?名無しさん:01/12/03 12:09 ID:???
>>192
そんなもん担当教師に聞け。
お前個人の資質を一般に問うてどうする。
アホか
199高一生:01/12/03 12:13 ID:QKcQZp97
それほど勉強しなくてもいい教科のこと。例えば、文系なら化学とか。
200ご冗談でしょう?名無しさん:01/12/03 12:15 ID:2Tbu3QBH
英語=それほど勉強しなくてもいい教科?
それはマズイと思うぞ
201ご冗談でしょう?名無しさん:01/12/03 12:16 ID:???
勉強しなくても出来るならしなくて良いぞ。
202高一生:01/12/03 12:19 ID:QKcQZp97
>>201
出来るんですっ!(川平風に)
203ご冗談でしょう?名無しさん:01/12/03 12:26 ID:???
物理と無関係の受験の話は他の人の邪魔だからよそ逝け
204ご冗談でしょう?名無しさん:01/12/03 12:34 ID:as+bkCkl
高校の授業で登場する理系科目ができただけで
得意だと思い込むのは危険です。戸田アレクシくんのように
精神破綻してしまう危険性があります。長い目で将来は
決めていくべき。
205受験生:01/12/03 14:25 ID:m5Kb6xfS
ジュール[J]
って、どんな定義でしたっけ?
時間的な定義も含まれてるんですよね?一分!一秒?
ホントドキュソな質問でスマン‥
(>_<)
206ご冗談でしょう?名無しさん:01/12/03 14:32 ID:???
>>205
単位面積、単位厚さ(cm)の牛肉が単位時間(分)でミディアムに焼けるエネルギー
207受験生:01/12/03 14:37 ID:Ng/U2uDA
》205
ご、ご冗談でしょう?
208ご冗談でしょう?名無しさん:01/12/03 14:55 ID:???
>>205は当局の諜報部員のモヨリ
ついに厨房レス抹殺に向けて動き出したか・・・
209受験生:01/12/03 15:00 ID:6HdblBEq
???
だ‥だれかぁ〜
(;_;)
210ご冗談でしょう?名無しさん:01/12/03 15:51 ID:???
0.4ワットの仕事率を2.5秒維持したときのエネルギー
211ご冗談でしょう?名無しさん:01/12/03 15:52 ID:???
とあるゲームにて、
実弾兵器:熱量が多大に発生
エネルギー:熱量が全く無いが威力が高い
という設定になっておるんですが、これに対して
「実際と反対ではないか!」という批判が多いです。

物理板の住人さんから見て、エネルギー兵器(ビーム、パルス、レーザー)に熱量がないというのは妥当なんでしょうか?
212ご冗談でしょう?名無しさん:01/12/03 15:58 ID:???
>>211
むしろ逆じゃないの?
213ご冗談でしょう?名無しさん:01/12/03 15:59 ID:???
ビーム・パルス・レーザーという分け方からしてタメ息を誘うな・・・
214ご冗談でしょう?名無しさん:01/12/03 16:02 ID:???
やはり逆ですよね?
あとビームは形状のことでパルスとかとは違うのかな。

まあ、ありがとうございます。
215154:01/12/03 16:03 ID:???
>>186
成る程・・。漠然とですが納得しました。
「宇宙に存在する物質はすべて球の表面にいる」
っていう考えが、まだ自分には想像しにくいですが・・
他に色々知っている方居たら、ぜひ話聞かせてください。
216ご冗談でしょう?名無しさん:01/12/03 17:33 ID:Q3yctdeX
波の干渉で強めあう条件がどうしてそうなるのか分からないんで教えてください。
217ご冗談でしょう?名無しさん:01/12/03 17:36 ID:???
物理でいう「ビーム」ってなんでしょうかw
218ご冗談でしょう?名無しさん:01/12/03 17:53 ID:???
>>217
粒子線のことだYO!
きちんというと、
「ほとんど同じ方向、同じ速さで進行している、空間に局在した多数の同種粒子の集まり」。
これ最強。
219ななし:01/12/03 20:03 ID:6HdblBEq
重原子を収束加速したもの
高エネルギーのため自発的に発光
だからチリのない宇宙空間で、チンダル現象で道筋が見えるわけじゃないから光線として見える
このスレはただのマニアのバカばかりだ
市ねよ
220ご冗談でしょう?名無しさん:01/12/03 20:07 ID:???
>>219
…いきなり何の話だ?
221ご冗談でしょう?名無しさん:01/12/03 20:10 ID:???
>>219
きにいらなきゃ書かなきゃ良いだろ、ボケナスの童貞厨房
222ご冗談でしょう?名無しさん:01/12/03 22:10 ID:???
ロケットは噴射速度以上に加速可能ですか?
223150:01/12/03 22:55 ID:???
物理達人の皆様 >>150 をヒントでもなんでも良いから教えて下さい。
そろそろ提出なんですが周りの人もさっぱり分からんと逝っています。
224ご冗談でしょう?名無しさん:01/12/03 23:05 ID:???
>>150
授業でなよ。
レーザー関係の本でコヒーレント状態って調べてみ。
ヒントだけだから後は自分で頑張ってね。
225ご冗談でしょう?名無しさん:01/12/03 23:13 ID:???
a)光子(電磁波)を教えてほしい
b)ボーズ・アインシュタイン凝縮を教えてほしい
c) a)のb)は可か否か教えてほしい
d) c)の適宜条件を教えてほしい
e) d)をふまえて議論のポイントを教えてほしい
f) A4紙1ページ程度に納まるよう議論展開してほしい
どの辺逝きますか?
226150:01/12/04 00:17 ID:???
物理達人の皆様 ありがとうございます

>>225
a) と b) はなんとなく分かる気がします。e) と f) は何とかします。
c) と d) が分からないです。
ボーズ・アインシュタイン凝縮を考えるには化学ポテンシャルの値の計算が
必要らしいんですが、光子の場合にはその出し方が分からないです。
超流動ヘリウムの場合には密度から化学ポテンシャルは導けるみたいですが。

>>224
レーザーのコヒーレント状態の化学ポテンシャルの出し方も分からないです。
ところで、レーザーのコヒーレント状態はボーズ・アインシュタイン凝縮
してるのですか?

レーザーのコヒーレント状態は波長が一定で干渉性がよいという程度の理解
しかないです。
量子力学の教科書には消滅演算子の固有状態で粒子数分布がポアソン分布と
書いてあるみたいですが、ボーズ・アインシュタイン分布とは書いてないし…

レーザーの温度って低いんですか?
なんとなく、ボーズ・アインシュタイン凝縮は低い温度で起きそうなんですが。
227224:01/12/04 00:49 ID:???
そっち系か。
ずばり書いたつもりだったんだけど、提出までの道のりは長いかもしれない。
今話題の原子のBECって、別名原子波レーザーと呼ばれているとかいないとか。
日経サイエンス、パリティなんか読むと載ってるかもね。
228ご冗談でしょう?名無しさん:01/12/04 00:51 ID:???
レーザーの電磁場がボーズ凝縮してるんだったら、
一様電場や一様磁場ってのもボーズ凝縮してるのか?
229ご冗談でしょう?名無しさん:01/12/04 01:21 ID:???
BCEしたルビジウムとかの原子レーザーは滅茶苦茶低温。
温度は忘れたけど、超流動Heなんかよりはずっーーと低温。
もっとも、原子レーザーといっても超流動Heが流れているのと本質的に
変わらんと思うが。
光のレーザーは知らん。
230ご冗談でしょう?名無しさん:01/12/04 01:56 ID:???
レーザー媒質の温度は負です。でも、高い温度です。
231ご冗談でしょう?名無しさん:01/12/04 03:25 ID:2VfLUcH/
エンタルピー、エントロピーってなに?
232150:01/12/04 03:39 ID:???
物理達人の皆様、いろいろありがとうございます!!

>>224
パリティの2000年8月号の11ページに載っていました。(以下部分引用)

> コラム2 光子はボーズ凝縮するか?
>
> .... もし、光子の気体が光子数が保存されながら熱平衡状態にあるとすると、
> たとえば熱運動する電子気体とコンプトン散乱しているような場合、
> 原理的には光子のボーズ凝縮が実現される。
> BCEを達成するには、その粒子が系の中に留まり続ける時間よりも
> 短い時間内に熱平衡に達する必要があり、
> これは光子および原子の両方において当てはまる原則である。

でも、やっぱり分からないです。(物理の才能ないなあ…)

例えば、鏡で囲まれた部屋にある温度の黒体をおいたとすると、
この部屋の中の光子はボーズ凝縮してる事になるのですか?
単なる黒体放射でプランク分布しているだけのような気もするし、
パリティの記事の状態になっているような気もするし…

>>228
一様磁場の中の光子数ってどうやって数えるのですか?
化学ポテンシャルはどうなっているのですか?
この場合の温度は絶対零度で良いのですか?

>>230
レーザー光線とレーザー媒質は熱平行なんですか?
レーザー光線の温度は負なんですか?
結局、レーザー光線ってのはボーズ凝縮してるんですか?
233ご冗談でしょう?名無しさん:01/12/04 20:54 ID:???
age
234ご冗談でしょう?名無しさん:01/12/04 22:48 ID:???
ラグランジュ出したあとに質点系はどうやったら速度出せるんですか?
普通に運動方程式解くんですか?
例えば
L=mv^2/2 + mgh
(v=dh/dt)
でお願いします。初心者なので分かりません。教えて下さい。
235ご冗談でしょう?名無しさん:01/12/04 22:57 ID:eEMfNSdV
ちょっとした質問です。
回折角は波長に比例するのですか?
例外的に比例しないものもあるんですか?
誰かお暇な方、お答えください。
236224:01/12/04 23:09 ID:???
>>232
うーん、そういう書き方になっていたのか。
またまた適切な助言にならなかったようだ。
手元に何もないので、誰かきちんと説明してある本を
紹介してあげてくれないだろうか...。

>なるのですか?
ならないと思いますが。
237ご冗談でしょう?名無しさん:01/12/05 00:10 ID:???
>>231
決して分かりやすいとはいえんが、
↓このへん見て勉強すれ。
ttp://www.ns.kogakuin.ac.jp/~ft11196/lecture/Physics-II/05-entropy/contents.html
238ご冗談でしょう?名無しさん:01/12/05 00:13 ID:GRyGL4WY
 ちょっと前に考えついたんだけど…

 人が光速で走ると時間は流れないんですか?
239ご冗談でしょう?名無しさん:01/12/05 00:21 ID:???
ながれます
240親切な人:01/12/05 00:36 ID:???

ヤフーオークションで、幻の人気商品、発見!!!

今は無き「コピーガードキャンセラー」↓
http://page2.auctions.yahoo.co.jp/jp/auction/b18032656

注目のオークションに適用される事、
間違いないですね。

買わないと、一生の損だと思います。
241ご冗談でしょう?名無しさん:01/12/05 07:14 ID:???
ちょっと質問なんですが人の人生が光速で流れるとどうなるんですか?
242ご冗談でしょう?名無しさん:01/12/05 08:56 ID:???
>>241
あなたのような質問をする人になります
243ご冗談でしょう?名無しさん:01/12/05 10:30 ID:Cn9gr49B
>>232
レーザー媒質とレーザー光は「非平衡・定常」な関係で、熱平衡ではないと思います。
あと、単色のレーザー光それ自体はプランク分布でないため温度は定義できません。

うまく制御されたレーザー空洞(ファブリーペロー干渉計など)内のフォトンは
(量子論的な)コヒーレント状態にあり、一つの量子状態に巨視的な数の状態が
縮退できます。一般にこの状態はエネルギーの基底ではありません。
量子光学の教科書を参照されると良いでしょう。

それを(普通は基底状態に巨視的な数の縮退を表す)BECと言うかどうかは
言葉の問題でしょう。 >>乞う専門家の意見。
244物理的無知人:01/12/05 11:01 ID:J35YfeVL
ホーキング、宇宙を語るってちょっと読んでみたんですけど
宇宙がある規則を持つように創られたなら
個人の空想行為や個人のマウスの1クリック、個人の起床時間などは、
神によって最初から全てプログラムされてるって事なんですか?
245ご冗談でしょう?名無しさん:01/12/05 11:04 ID:???
>>244
あなたの昨日のオナーニもすべて定められたものなのです
246ご冗談でしょう?名無しさん:01/12/05 11:09 ID:???
>>244
 規則といっても、確率的なものもあるでしょ。
いまどき、そんなことを言う人はいないから、
アインシュタインさんに聞いてみたらどう?
247物理DQN:01/12/05 13:06 ID:J35YfeVL
ふ〜ん。
その確率的ってものは神にも予測不可能なのか?>アインシュタイン
248ご冗談でしょう?名無しさん:01/12/05 13:40 ID:5Xw4crIO
物理に神を持ち出すな と怒られるぞ。
249文学板からやってきました。:01/12/05 15:32 ID:Ick9vTJi
球体の内側全てを鏡にして中に入った場合、
鏡には自分はどのように映るのでしょうか?

江戸川乱歩の『鏡地獄』を読んで、
自分自身気が狂いそうになりました。
どうか教えてやってください。

過去スレにもいくつか似たような質問があるようですが、
ことごとく相手にされていませんでした。
激しくスレ違い板違い、若しくは質問自体がなってないのであれば申し訳ない。
答えてやろうという奇特な方がおられたら、よろしゅう御願いおま。
250ご冗談でしょう?名無しさん:01/12/05 16:12 ID:J6HfAqA5
光が入らないから、真っ暗
251ご冗談でしょう?名無しさん:01/12/05 16:31 ID:???
>>249
>ことごとく相手にされていませんでした。
え?わざわざレイトレースまでして画像をアップした人がいたと思ったけど
俺その画像見たよ
252@:01/12/05 16:45 ID:FEYWhPR0
数学とか物理とか、ぜんぜん解らないので
質問することもアンポンタンなことだと思いますが
教えてくれたらうれしいです。
彗星の軌道で双曲線をとるのがあるとらしいですが。
光円錐(双曲線みたいな形)と関係あるのでしょうか?
あと、ロバチェフスキーの双曲線とは?どうなんでしょう??
何がなんだかさっぱりわかりません。
私の言っていることも、さっぱりわかりませんね・・・
253ご冗談でしょう?名無しさん:01/12/05 17:20 ID:???
あ、こんなスレッドがあったのですね。
先ほど以下のスレッドに質問を書きこんでしまいました。
失礼いたしました。

http://cheese.2ch.net/test/read.cgi/sci/996914195/l50
の91です。

どなたかご教授いただけると助かります。
254ご冗談でしょう?名無しさん:01/12/05 17:31 ID:???
>>251
リンク着盆
255ご冗談でしょう?名無しさん:01/12/05 19:04 ID:WaZFbxAX
>>250
照明もって入ればいいだろ。
応用性の欠如。

あと、息ができなくなるというの不可。
256ご冗談でしょう?名無しさん:01/12/05 19:15 ID:Rg2LF6am
等加速運動とは加速値が同じ幅だけ変化する事ですか?
20−40−60−80
20−400−8000−16000
その幅は等差または等比?
257ご冗談でしょう?名無しさん:01/12/05 20:05 ID:???
>>256
等加速なら加速値はずっと初期値に等しいんでは?
258ご冗談でしょう?名無しさん:01/12/05 20:07 ID:???
加速値などという自分語を使ってる限り、理解には程遠いだろう。
259ご冗談でしょう?名無しさん:01/12/05 20:29 ID:???
速度が変化していく運動を等速運動と言うのであれば
加速度が変化する運動を等加速度運動と言ってもいいかもしれない
260ご冗談でしょう?名無しさん:01/12/05 20:39 ID:???
速度が一定の変化する状態を等加速運動と言うのならば
加速度が一定の変化する運動はなんというの?
261ご冗談でしょう?名無しさん:01/12/05 20:42 ID:???
>>260
は? 速度が一定の運動は等速度運動ですが?
262ご冗談でしょう?名無しさん:01/12/05 20:58 ID:???
網にかかった・・・
263ご冗談でしょう?名無しさん:01/12/05 20:59 ID:???
>>260
頼むからせめて日本語を書いておくれ
264ご冗談でしょう?名無しさん:01/12/05 21:05 ID:???
>>255
お前なんか言え
265:01/12/05 21:34 ID:Ik0b6Qnk
単位を取るために物理を勉強する奴の気が知れない。何で大学来たのか聞きたい
266:01/12/05 22:36 ID:???
特殊相対性理論で物の長さが√(1―C~2/V~2)だけ縮むとありますが
それは進行方向にだけ縮むのですか?それとも縦横高さいずれも縮むのですか?
267:01/12/05 22:39 ID:???
>266
すまそ
V~2/C~2 のまちがい
268ご冗談でしょう?名無しさん:01/12/05 22:52 ID:Q8U7wAhn
>>266
進行方向だと思われます。
269ご冗談でしょう?名無しさん:01/12/05 23:03 ID:???
>>266-268
もし進行方向に垂直な方向の長さが変化するとパラドックスが生じます
270ご冗談でしょう?名無しさん:01/12/05 23:54 ID:s4+7z2oo
問1)半径aの球の表面に電荷が一様に面密度ρで分布している。
極角がθ〜θ+dθの微小部分a*dθに含まれる電荷を求めよ。

この問題、答えはdQ=ρ2π(a~2)sinθdθと分かっているのですが、
導出ができません。どうするんでしょうか‥お願いします。
271ご冗談でしょう?名無しさん:01/12/06 00:28 ID:???
>>270
電荷面密度×微小部分の面積
272ご冗談でしょう?名無しさん:01/12/06 01:06 ID:d5xEWXNq
BECの干渉縞を説明できないのは何故ですか?
273ご冗談でしょう?名無しさん:01/12/06 02:16 ID:RTgRkcSN
トランジスタとは? 教えて下さい!
274後1時間:01/12/06 05:42 ID:3A0GJYPH
スペースシャトルの発射でも見ようぜ
http://choco.2ch.net/test/read.cgi/news/1007491230/l50
エンデバー打ち上げ実況その2
http://choco.2ch.net/test/read.cgi/dome/1007505898/l50

射場:フロリダ州ケネディ宇宙センター(KSC)39B発射台
http://playlist.broadcast.com/makeplaylist.dll?id=138293&segment= Windows Media Video (28.8k)
http://playlist.broadcast.com/makeplaylist.dll?id=138294&segment= Windows Media Video (56k)
http://playlist.broadcast.com/makeplaylist.dll?id=330814&segment= Windows Media Video (100k)
http://playlist.broadcast.com/makeplaylist.dll?id=139072&segment= Windows Media Video (300k)
Yahoo! Broadcast以外にもありますので、つながりにくい人はどうぞ。
http://spaceflight.nasa.gov/realdata/nasatv/HighSpeed.asx  (232k)
http://nasatechnology.nasa.gov/video/nasatv_live_hsf_56k.asx  (56k)
http://spaceflight.nasa.gov/realdata/nasatv/nasa56k.asx  (56k)
MSNBC
http://www.msnbc.com/m/lv/a/msnbc4.asp  (28.8k)
http://www.msnbc.com/m/lv/a/msnbc24.asp  (28.8k)
カウントダウンタイムテーブル
http://science.ksc.nasa.gov/shuttle/countdown/count.html
カウンロダウンタイマー(java)
http://163.205.10.51/shuttle/countdown/cdt/

シャトルの現在位置(軌道上にあるときは、スペースシャトルの現在位置も表示されます。)
http://38.201.67.72/realdata/tracking/
>250
そりゃそうだ…自分のアホさに号泣


>251
すみません、見落としたみたいです。もう一度探してきます
画像ぜひとも見たいです。言葉で説明されても理解できないと思うし(阿呆
レスありがとうございました
276ご冗談でしょう?名無しさん:01/12/06 21:53 ID:???
http://mentai.2ch.net/test/read.cgi/philo/1006807130/
自由意志やら因果律やらとまた中途半端な
科学認識で語ってるのですが、どうしましょう
277ご冗談でしょう?名無しさん:01/12/06 22:25 ID:???
肉眼で可視域の光子の1個1個を認識する事は可能ですか?
278ご冗談でしょう?名無しさん:01/12/06 22:28 ID:???
>>272
BECの干渉縞って何ですか?
279あほ:01/12/06 22:49 ID:???
相対性理論では未来に行くことは可能ですが、
過去に行くにはどうすれば可能でしょうか?
以前にワームホール云々で可能だとあったような気がします。
わかりやすく説明してください。
280ご冗談でしょう?名無しさん:01/12/06 22:59 ID:3P2URYli
交流の原理を誰か短く簡潔に教えてくれないでしょうか。
簡単な掴みだけで結構ですのでよろしくおねがいします
281ご冗談でしょう?名無しさん:01/12/06 23:34 ID:g+n3FjNY
>>277
人間の目では無理です
282名無し:01/12/07 00:12 ID:5+TBszn4
>>277
カエルの中には光子を一個一個確認できるやつがいるってさ。
283ご冗談でしょう?名無しさん:01/12/07 00:14 ID:u5OFddyk
>>280
電圧が+と-で周期的に交互に滑らかに変化します。
284ご冗談でしょう?名無しさん:01/12/07 02:49 ID:???
まずは自己紹介から
285ご冗談でしょう?名無しさん:01/12/07 10:50 ID:KXcQs1vP
すいません、誰か共振鍋について詳しく知っている人はいないでしょうか?
もしいらしたら教えていただきたいのですが。
286ご冗談でしょう?名無しさん:01/12/07 10:58 ID:???
>>285
Aがなべをつつく。
傾きの最高点でBが鍋をつつく。
再び傾きの最高点でAが鍋をつつく。
この繰り返しにより、ある時間tに鍋が引っくり返る。

(゚Д゚)(゚Д゚)アチ-
287ご冗談でしょう?名無しさん:01/12/07 11:22 ID:???
>>282
スゲー!たくさん繁殖させてスーパーカミオカンデで働いてもらおう
カミオカンデの光電管の検出効率は入射光子の20%くらいだったはず
288   :01/12/07 13:36 ID:tj+HhWpM
光はなぜ光るのでしょうか?
289工房:01/12/07 13:36 ID:???
教科書には放射線はα線、β線、γ線(X線)の3種類しかない
と書いてありましたが、これは正しいのですか?
放射線技師免許の教科書にもそんな風に書いてありました。

中性子線、陽子線、π線をはじめとするハドロン線、
μ線やニュートリ線などのレプトン線は放射線ではないのですか?
290工房:01/12/07 13:39 ID:???
>>289 訂正 ニュートリ → ニュートリノ
291ご冗談でしょう?名無しさん:01/12/07 14:07 ID:???
>>287
カバーする領域が狭すぎるぞ
292ご冗談でしょう?名無しさん:01/12/07 14:35 ID:???
>>289
ニュートリノは別だけど、その他の粒子線も放射線といっていいと思う。
狭い意味では放射性核の崩壊で出てくるαβγだけを指すみたいだけど。
293ご冗談でしょう?名無しさん:01/12/07 17:17 ID:MJRtaoeH
カー・ブラックホールでのエルゴ領域で粒子のエネルギーが
負になるのはなぜですか。本にはキリングベクトルが未来方向
に向いてるからって書いてあるけど、何のことやらさっぱりです。
294工房:01/12/07 17:35 ID:???
>>292
なぜ、ニュートリノは別なのですか?
295ご冗談でしょう?名無しさん:01/12/07 17:55 ID:???
>>294
厳密には放射線に関する法律なんかで定義されてるんだろうけど、
放射線っていうのは害を与えるかどうかに着目した分類です。
たとえば高エネルギーの電子(β線)や陽子線は放射線というけど、
あなたの体内に多量にある電子や陽子は放射線と言わない。

で、ニュートリノはほとんど相互作用せず、地球でさえスカスカに
通り抜けてしまうので、人体への害は事実上ないので、放射線として
扱われることはない。
296ご冗談でしょう?名無しさん:01/12/07 18:25 ID:ZGeKSOL1
はじめまして。
ちょっと聞きたいことがあります。
電磁波を抑えようとして、シールドケース(アルミ製)を
とっつけたんですが、200MHz以下の放射ノイズは大分減っ
たんです。でもそれ以上の周波数になると、取り付ける前
より増加しているんです。

これって何でか分かりますか?
教えてください。
297ご冗談でしょう?名無しさん:01/12/07 19:48 ID:39jcAP+F
>>296
そもそも、その放射ノイズってどうやってはかってるん?
298工房:01/12/07 20:03 ID:???
>>295
エネルギーが数TeVのニュートリノは放射線として扱った方が
良いような気がしますが、どうでしょう?
核子(質量1GeV)を相手とすると√sがちょうどWの質量程度になり、
バンバン相互作用しそうです。
299ご冗談でしょう?名無しさん:01/12/07 23:21 ID:HOXygHWL
>>297
電波暗室を借りて測ってます。
3m法で測ってます。
なにとぞお教えください。なにとぞ!
300ご冗談でしょう?名無しさん:01/12/07 23:40 ID:???
機械・工学板の方が良レス期待できるぞ
電源ケーブルとかは大丈夫か?
アルミ版そのものに電流が流れてたら意味無いぞ
301物理素人:01/12/07 23:45 ID:luubGYXJ
完全に何も存在しない空間(重力場も質量の存在もない)に
時間は流れてるのですか?
302ご冗談でしょう?名無しさん:01/12/07 23:51 ID:bGTTDx0O
話の脈絡とは違うけど、
一つ質問。あまりにアフォな質問なので、
屁も出ないかもしれないけど、まぁ聞いてくれよ。
よくE=MC^2って言うけど、光速の二乗って、
光速の何をかけてるの?メートル毎秒?
キロメートル毎時とか?
それとも、質量に比べとてつもなく大きい値だという、
ある意味抽象的な事しかあらわしてなくて、
実際の値とかで計算はできないの?
ようするに1とか2とかそういう具体的な数字にはならないの?
できれば例を示してくれるとありがたいんだが。
303ご冗談でしょう?名無しさん:01/12/07 23:54 ID:???
ケース内で電磁波が共鳴してるとか。
ケースのサイズが0.1cmから1mぐらいのオーダーだから大体
周波数c/0.1〜10^9Hz,c/1〜10^8Hz、
おおよそ100MHzくらいからケース内に定在波ができる気がする
などと適当なことを言ってみる。
304ご冗談でしょう?名無しさん:01/12/08 00:00 ID:???
>>302
お好きな単位でどうぞ。

移動距離=速度×時間
という誰でも知ってる関係式で、速度を時速で表そうが秒速で表そうが
求まる移動距離は同じでしょう。それと同じ事。
要は単位同士の変換の問題なので、単位に気をつけて正しく
変換できればなんでもいいです。
305ご冗談でしょう?名無しさん:01/12/08 00:11 ID:hPdfOiQ+
>>304
なるほど。そういえばそういうことか。
ではmを5(g)とし、cをkm/sで表すと
E=4500億gkm/s?よくわからん単位になってしまったが。。
これでいいのかなぁ?
読み方はグラムキロメーター毎秒とか?
306ご冗談でしょう?名無しさん:01/12/08 00:21 ID:???
>>305
単位についてはこちらのページなどもどうぞ
http://www.sendai.kopas.co.jp/METAL/PUBS/SI.html

エネルギーの単位としては
日常生活レベルでは
J(ジュール)=[N・m]=[kg・m^2/s^2]や
cal(カロリー)=4.18Jが、
素粒子などを扱うレベルでは
eV(エレクトロンボルト)=1.60218×10^(-19)J
がよく使われます。
(念のため:x^2=xの2乗)
307ご冗談でしょう?名無しさん:01/12/08 00:34 ID:???
ブラックホールとかで言われる「特異点」って
どういうヤツでしたっけ?
308ご冗談でしょう?名無しさん:01/12/08 00:41 ID:???
日常生活ではWh(ワットアワー)=3600Jが普通
309高畠 ◆hAAHAhA. :01/12/08 11:56 ID:5tUYHof8
ハミルトニアンの運動エネルギー項を第2量子化するのですが、
H=p^2 /2mに対して、

∫dr ∫dr' Ψ^†(r) <r|p^2/2m|r'> Ψ(r') はいいのですが、(Ψ^†,Ψは昇降演算子)
これを式変形していくと
=\hbar^2/2m ∫dr(∇Ψ^†(r))・(∇Ψ(r))

になるのはなぜでしょうか??
<r|p^2/2m|r'>の部分の扱いに困っています。
310ご冗談でしょう?名無しさん:01/12/08 13:32 ID:wTzLEx9u
>>306

ありがとうございまちょ。
311ご冗談でしょう?名無しさん:01/12/08 13:46 ID:K2+vu0EP
>>307
時空が定義できなくなり,物理法則が破綻する点です.普通はブラックホールの地平線に隠れて見えないと
されていますが,見えてしまう「裸の特異点」の例も
提出されており,「宇宙検閲仮説」が正しいのかという
完全な証明はまだありません.

http://spaceboy.nasda.go.jp/note/Kagaku/J/kag103_tokuiten.html

関連事項として「特異点定理」というものがあります.おおざっぱに言えば,一般相対論の枠内ではこの宇宙は
特異点から生まれた事が帰結として出てきます.
ここからも重力の量子論の必要性が出てきます.
312ご冗談でしょう?名無しさん:01/12/08 15:44 ID:???
古典物理学においてσi={±1}で表される
スピン系をなにモデルって言うの?
313ご冗談でしょう?名無しさん:01/12/08 17:56 ID:ENqIniac
(1)宇宙の外側ってどうなってるの?
(2)宇宙って本当にビッグバンではじまったの?じゃあ、その前は?
3144回生のものです:01/12/08 19:32 ID:Ty355yh9
はじめまして.
突然ですが,今卒業研究でSiC焼結体の電気抵抗を4探針法で
測定しています.
装置を何とか自分で作ってみたのですが,試料となかなか接触が,
上手くいきません.一つ測定するのに,大変な時では三時間ほど,
接触させるのに時間がかかります.
電気的に接触させる何か良い方法はないでしょうか?
何か良い案がありましたら教えてください.
お願いします.
315:01/12/08 23:43 ID:lmPcvTt2
皆さんはどうもいきつまってやる気が出ない時はどうしてますか?
316ご冗談でしょう?名無しさん:01/12/09 00:10 ID:???
蛍光の消しゴムとかおもちゃってなんで光るの?
崩壊してるの?
317名無し:01/12/09 00:20 ID:???
>>316
励起寿命のおもいっきり長い状態をもった分子を使ってるから。
光が当たってているときに励起されて、暗闇でゆっくりとその光を
放出していると思えばいいんじゃないかな。
318ご冗談でしょう?名無しさん:01/12/09 00:35 ID:???
正しくは燐光?
319オーロラ社:01/12/09 01:35 ID:???
蓄光では?
320ご冗談でしょう?名無しさん:01/12/09 04:34 ID:JDwyov+y
熱の伝わり方についての疑問なんですが、
私の頭のなかでは熱い鉄の原子はすごい勢いでガクガクブルブルしています。
もし、その鉄とより温度の低い鉄をくっつけた場合熱の移動はどのようにして起こるのでしょうか?
実際に接触面の原子がガクガクブルブルしてぶつかり合ってもう一方の鉄も熱くなっているのですか?
それとも実際は原子がぶつかって振動を伝えるのではなく、他の物質が媒介して居るんですか?
321ご冗談でしょう?名無しさん:01/12/09 05:36 ID:???
>>311
ども有り難しです。

あーそっかー。
322ご冗談でしょう?名無しさん:01/12/09 15:03 ID:???
>>316
目に見えない波長の光を吸収して良く見える波長の光を出してるんだよ
323ご冗談でしょう?名無しさん:01/12/09 16:58 ID:WRshlKZ1
300>>
電源ケーブルはシールドケーブル使ってるんで
大丈夫なはずです。
シールドケースにもアース線つけてます。
ただ、シールドケースを取り付ける際に微小な隙間
が開いちゃってるんで、そこになんか原因があんのかなぁ
とも思うんですけど。
324ご冗談でしょう?名無しさん:01/12/09 19:56 ID:???
>>316
蛍光消しゴムや蛍光ペンって、全然光ってない気がするのは漏れだけ?
325ご冗談でしょう?名無しさん:01/12/09 21:42 ID:oqHQDShb
>>324
紫外線を当ててみれば光るだろう。
通常の光源のもとでは、可視光の反射(散乱)が多いので目立たないだけ。
326高校生:01/12/10 05:21 ID:Xo+afCa6
長さって何ですか?
327ご冗談でしょう?名無しさん:01/12/10 10:25 ID:???
>>326
物の量の一次元的な尺度
328社会人:01/12/10 22:00 ID:???
 2枚の鏡を対面させて、中間に自分が入る。
すると、自分の像が無限に重なって見える。
さて、ここで問題。
1.この空間を真っ暗にして、フラッシュをたいて写真を撮る。
  フラッシュ及びシャッター速度(及びそれらの間隔)を
  短くすることで、「自分の像が数人しか写ってない写真」を
  作ることは物理的に可能か?
2.この答えを明日の朝礼で発表しなければならない僕は、
  会社的にどうなんだろう。
 
329ご冗談でしょう?名無しさん:01/12/10 23:55 ID:???
>328

シャッターの開いている時間が有限であるのならば、
写真に写る像の数も有限になります。
330名無し:01/12/11 00:55 ID:???
>>328
そりゃ可能でしょう。
331まじ男:01/12/11 02:27 ID:1JPUNLAV
負の質量ってあるんですか?ちょっと気になってしまって。眠れません。
332ご冗談でしょう?名無しさん:01/12/11 03:49 ID:???
漏れ、今夜は徹夜。付き合え。
333ご冗談でしょう?名無しさん:01/12/11 17:10 ID:???
単発質問スレ防止上げ
334ご冗談でしょう?名無しさん:01/12/11 17:10 ID:???
>>331
ない。
335:01/12/11 18:55 ID:HO4ME0wn
ガウスの法則の左辺の面積分は、電気力線が貫く領域の面積で良いのですか?
336ご冗談でしょう?名無しさん:01/12/11 19:34 ID:???
>>335
右辺の体積積分の領域の表面(の面積)。
領域によっては貫くこともあるし、貫かないこともある。
337ご冗談でしょう?名無しさん:01/12/11 20:04 ID:l6QJDzgs
>>295
放射線には「エネルギーの流れ」みたいなもんが必要なんで,
電子や陽子がそこにあるだけでは「放射線」にはならないかと。
(広義の放射線の定義は「粒子や波動を介して空間を伝播するエネルギーの流れ」だったかな)
338工房:01/12/11 21:25 ID:???
>>337
では、電波(kHz〜GHz)は放射線なのですか?
また、その定義はどこで聞いたのですか?
339ご冗談でしょう?名無しさん:01/12/11 21:49 ID:???
放射線と放射能さえきちんと区別してくれれば
後の細かい定義なんてどうでもいいよ
340工房 :01/12/11 22:24 ID:???
では、放射能と放射線の正確な区別を教えて下さい。
例えば、熱中性子は放射能ですか?それとも放射線?
341ご冗談でしょう?名無しさん:01/12/11 22:51 ID:Eo0hxz00
AとBの綱引きについて
二人の体重が同じだとすると摩擦力は等しくなり、Bの力よりAの力の方が強いとする
(M、μNは二人とも同じ、F(A)>F(B))
作用反作用の法則より綱を引くのと同じ力が自分にもかかる。

このとき二人の運動方程式を立てると
A:Ma=F(A)−μN
B:Ma=F(B)−μN
となって、Aが負けることになってしまうのですが…
どこが違っているのか指摘をお願いします。
342ご冗談でしょう?名無しさん:01/12/11 22:54 ID:???
人間の体重ってウンコ出すとへるの?
343ご冗談でしょう?名無しさん:01/12/11 22:54 ID:gX+C2NbL
>>340
放射線

>>341
綱引きって地面を蹴って後ろにジャンプするから、
そういう単純な運動方程式にはなりません
344ご冗談でしょう?名無しさん:01/12/11 23:29 ID:???
高速中性子は放射線でしょう。
止まっている中性子は15分くらいでベータ崩壊するから放射能ですね。
熱中性子は微妙だなぁ…
345ご冗談でしょう?名無しさん:01/12/12 02:07 ID:???
流体力学や材料力学の質問はここでいいかな?
346ご冗談でしょう?名無しさん:01/12/12 02:12 ID:???
>>345
ここでもいいと思いますけど、もしかしたら機械・工学板のほうがいいかも
しれないです。
特に材料力学は
347ご冗談でしょう?名無しさん:01/12/12 02:46 ID:???
材料物性板は?行ったことないけど
348レポ:01/12/12 04:50 ID:X8dMOIle
気体分子のランダム運動と温度の関係について簡単に教えてください。
349ご冗談でしょう?名無しさん:01/12/12 06:47 ID:jmR5sSVQ
ガンダムにでてくるスペースコロニーのことで質問があります。
スペースコロニーは筒状の人工衛星で、その中心を軸にして自転
する事で、筒の内側の縁に遠心力が発生しています。
スペースコロニーに住んでいる人たちはその内側の縁に家をたてたり
して生活しているわけですが、そのようなところでジャンプをした
ばあい、着地点はジャンプを始めた点と一緒になるのでしょうか。
あほな質問ですがだれかおしえてください。
350337:01/12/12 06:48 ID:wxwrp3TN
>>338
放射線です。
(放射線のなかの 非電離放射線 になります)
351337:01/12/12 07:09 ID:wxwrp3TN
350に追加
で、よく使われてる(狭義の)放射線(=電離放射線)は 広義の放射線のうち気体を電離させることができるもの だったかと。
(実際の定義はもうちょい違うけど)
ニュートリノビームが放射線でないってのは、狭義の放射線で必要な電離性がない ってことによるような気がする。
(気がするだけで、ホントどうかは調べてみないとなんとも言えんが。)
352:01/12/12 09:39 ID:DgD+bYpH
物理入門コース電磁気一は頑張れば何時間位で終わらせることができますか?
353ご冗談でしょう?名無しさん:01/12/12 12:22 ID:S9Vgg4dV
統計に出てくるAPPEL関数の性質ってなんですか?
354ご冗談でしょう?名無しさん:01/12/12 16:30 ID:RAGotPbm
>>349
真上に跳ぼうとした場合、着地点はコロニーの回転の向きにずれます。慣性系で人と地面の軌跡を書いてそれぞれの速度を考えればわかると思います。人は等速直線運動、地面は等速円運動をし、人は実際には真上に跳ばないことに注意してください。
355ご冗談でしょう?名無しさん:01/12/12 19:45 ID:X1dkbWBG
>354
おいおい「電車の中でジャンプ」と類似のネタ質問に、その回答はまずいだろ
356ご冗談でしょう?名無しさん:01/12/12 20:27 ID:sSvYcRkZ
宇宙飛行士はどうやって体重を量るのでしょうか?
357ご冗談でしょう?名無しさん:01/12/12 20:32 ID:???
>>356
地上で計る、じゃ怒る?
ブランコみたいなのに座らせてグルグル回せば慣性質量は計れるかなぁ?
358356:01/12/12 20:41 ID:sSvYcRkZ
実際にスペースシャトルの乗員は定期的に
健康チェックのため体重のデータをNASAに送ってるらしいんですが
うーん、わからん
359ご冗談でしょう?名無しさん:01/12/12 20:46 ID:???
NASAに聞く、じゃ怒る?
戦闘機とかに付いてるパイロット脱出用の装置(シートごと飛び出すやつ)
みたいな感じに、座って静止している人を一定の速度まで加速するのに要した
力積を計れば、なんとなく・・・
360356:01/12/12 20:52 ID:sSvYcRkZ
がんばって調べてみます。ありがとうございました。
(あの機内の省スペースで測定する装置が思い浮かばないんですよねえ…)
361ご冗談でしょう?名無しさん:01/12/12 21:28 ID:???
そもそも宇宙で体重を測っていることは確かなんでしょうか?
362工房:01/12/12 22:58 ID:???
>>338,>>350 ありがとうございまっす。
どうやら、TeVニュートリノは電離放射線だけど、低エネルギのニュートリノは
非電離放射線となる。で良いみたいですね。

ところで、関連法規を調べてみると

|<<<原子力基本法 第3条(定義)>>>
|「放射線」とは、電磁波又は粒子線のうち、直接又は問接に空気を電離する
|能力をもつもので、政令で定めるものをいう。
|<<<核燃料物質、核原料物質、原子炉及び放射線の定義に関する政令 第4条(放射線)>>>
|原子力基本法第3条第5号の放射線は、次に掲げる電磁波又は粒子線とする。
|(1)アルファ線、重陽子線、陽子線その他の重荷電粒子線及びべータ線
|(2)中性子線
|(3)ガンマ線及び特性エックス線
| (軌道電子捕獲に伴って発生する特性エックス線に限る。)
|(4)1メガ電子ボルト以上のエネルギ一を有する電子線及びエックス線

|<<<電離放射線障害防止規則 第二条(定義等)>>>
|この省令で「電離放射線」(以下「放射線」という。)とは、次の粒子線又は電磁波をいう。
|一 アルファ線、重陽子線及び陽子線
|二 ベータ線及び電子線
|三 中性子線
|四 ガンマ線及びエックス線

となってました。陽子を除く荷電バリオンは原子力基本法では放射線ですが、
電離放射線障害防止規則では放射線ではないようです。
逆に、ポジトロニウム崩壊で出てくるような0.5MeVのγは原子力基本法では
放射線ではないが、電離放射線障害防止規則では放射線になるようです。

また、ニュートリノ、π0等の全てのメソン、電子以外のレプトンもこれらの法規上は
放射線では無いように読みとれます。
363ご冗談でしょう?名無しさん:01/12/12 23:27 ID:???
>>361
>そもそも宇宙で体重を測っていることは確かなんでしょうか?

宇宙飛行士もモルモットですから、測っていると思います。測らなくていいほど
微小重力状態にいった人の数もいないわけですし。

なお測定については、200kg 程度の標準分銅さえあれば質量は測れるわけ
で、それぐらいの荷物と天秤+滑車の機構くらいは積んでいそうな気もしますが
特に気にする必要もないのでは?
364ご冗談でしょう?名無しさん:01/12/12 23:59 ID:IYtl6+wM
>>363
>天秤+滑車の機構
ん? 何か激しく勘違いしてない?
365ご冗談でしょう?名無しさん:01/12/13 00:30 ID:???
>>356
ばね定数の知れてるばねがあれば
周期の測定から無重力でも量れるか。
力と加速度より簡単かな?
366356:01/12/13 02:01 ID:SOBQEIIj
>>365 おおっなるほどー!!(禿しく感動
バネ付きの台車みたいなのにのっかってビヨーンってやれば、
あとは直線往復運動になるように工夫すればうまくいきそうですね。

これでグスーリ眠れます。ありがとうございました。
367:01/12/13 02:53 ID:HYxDafxg
物理をやる上で色々と役に立つパソコンソフトを教えて下さい
368350:01/12/13 08:38 ID:4YDTAQLM
>>362
「直接または間接に電離」ってのが一癖あったような.
裳華房 ポピュラーサイエンスシリーズの「放射線ものがたり」あたりを見れば
きちんと説明されているとおもう.
369    :01/12/13 09:49 ID:TAkozInf
物理法則が破綻する点があるそうですが、破綻すること自体が
、その法則が間違いであることの証明ではないかと思います。
どうなんでしょうか?
370ご冗談でしょう?名無しさん:01/12/13 09:59 ID:???
その場合も、近似的に成り立つ法則としては有用です。
それを間違いであるとしてしまうときりがないんです。
371ご冗談でしょう?名無しさん:01/12/13 10:23 ID:???
F=G*m1*m2/r^2で
r=0がニュートンの万有引力の法則の破門点かな?
でも、r=0を考えなければ有用な法則ではあるよね
372ご冗談でしょう?名無しさん:01/12/13 10:58 ID:???
>>369 「役に立たない」は「間違い」だと思いますが、
「間違い(厳密には正しくないの意)」だから「役に立たない」と
短絡的に結びつけるべきではないと思います。

既存の物理法則が「厳密に見ても間違いが無く完璧」と信じられてる、
なんて考えない方が良いかと思います。
373名無し:01/12/13 11:06 ID:???
なぜ物理を理解していない奴が学位をもらえるのか?
374373:01/12/13 11:21 ID:???
思うに、こういう奴らに学位を与えるせいで、頭の悪い院生が増える.
そういう奴に限って、実験の授業でストレスの発散の為に
学部生には答えられそうに無い、難しい質問をしてをいじめる.
はっきりいって迷惑です.
375ご冗談でしょう?名無しさん:01/12/13 15:15 ID:zcR6NaRI
ミューオンの崩壊から出てくる電子を観測してミューオンの寿命を測定する実験では原子核に吸収されてしまうミューマイナスは電子を放出しないのでどのようにあつかったらいいんでしょうか?
376名無し募集中。。。:01/12/13 17:25 ID:q76h1m78
モーメントって何?
377>376:01/12/13 18:29 ID:???
トルク
378ご冗談でしょう?名無しさん:01/12/13 19:08 ID:???
ちょっと質問。
放射線の被曝するとかの単位でシーベルト[Sv]
(ここではパーミニッツで)ってあるじゃん。
あれって1分間に放射線を何本浴びたら1シーベルトなの?
379ご冗談でしょう?名無しさん:01/12/13 20:23 ID:???
>>378
例えばhttp://www.jca.apc.org/~kaimu/radiation.htmの
線量当量の説明を見よ
380378:01/12/13 22:13 ID:???
>>379
俺の質問よく読んで、、、、
381379:01/12/13 22:22 ID:???
>>380
リンク先読んだ?
吸収した放射線のエネルギーを元にした単位であって、
放射線を何本浴びたら1シーベルトかといわれても
答えようがないのよ
382ご冗談でしょう?名無しさん:01/12/13 22:24 ID:???
>>376
モーメントゆーてもいろいろあるから、何モーメントかゆわんと答えられん。
ちなみに>>377のトルクは力のモーメント
383378:01/12/13 23:01 ID:???
なるへそ。んじゃぁ、一人で困ってます。
384ご冗談でしょう?名無しさん:01/12/14 10:07 ID:C4elLLRe
>>375
どのくらいの割合で吸収されるかは結果からわかるだろ。
その分を引け。
385ご冗談でしょう?名無しさん:01/12/14 16:23 ID:DK6KUINc
すいません、突然ですが、
駿台文庫の「必修物理」を探しているんですけど、
どなたか古本屋で見かけた方いませんか?
386ご冗談でしょう?名無しさん:01/12/14 16:58 ID:???
自動車のスペックで馬力とトルクってありますよね。
馬力がエネルギー効率でトルクが力ですよね,
具体的にはどう違うんですか?
例えば加速に関係するのはどちらですか?
387ご冗談でしょう?名無しさん:01/12/14 17:08 ID:???
大気圏突入時に蒸発して燃え尽きたものはいったいどこへいくのでしょうか?
素人な質問で申し訳ありません
388ご冗談でしょう?名無しさん:01/12/14 18:00 ID:JLRVWpvv
>>387
気体になって大気内に拡散。
あるいは金属などの場合なら熱で溶けて細かい塵になって
やっぱり大気内に拡散。
389ご冗談でしょう?名無しさん:01/12/14 18:05 ID:???
>>388
サンクスです。
390工房:01/12/14 21:02 ID:???
>>378, >>379
体重60KGの人が全身に、運動エネルギが1MeVの電子線(Q-Factor=1)を

60 × (1J/1MeV) = 3.7 × 10^14 個

浴びると、このエネルギの電子線ならほぼ100%が人体に吸収されるとして、
1シーベルトの被曝となりそうなんだけど、こんな計算で良いの?
391ご冗談でしょう?名無しさん:01/12/14 21:24 ID:vZ7z0UKD
電磁気で、例えば誘電体境界面でポテンシャルが連続と
あるけど、これってどうやって導くんでしたっけ?
392ご冗談でしょう?名無しさん:01/12/14 21:30 ID:???
-∇φ=E
393ご冗談でしょう?名無しさん:01/12/14 21:45 ID:???
>>392
それじゃ全く証明になってないよ。
394ご冗談でしょう?名無しさん:01/12/14 22:08 ID:???
>>391
境界上でポテンシャルが不連続だと
∇φが無限大になってしまい、電場が無限大になる。
だから、境界を囲むようにしてガウスの定理を適用すると
境界上の電荷密度が無限大となり矛盾。
395ご冗談でしょう?名無しさん:01/12/15 01:46 ID:???
質問
雨粒が落下するときの終端速度について質問です。
重力加速度のみを考えると終端速度は無限大になるので、
速度に比例した空気抵抗を考えたとしても、結果とは大きく異なってしまいます。
どうしたら良いですか。
396ご冗談でしょう?名無しさん:01/12/15 02:42 ID:???
>>395
>速度に比例した空気抵抗を考えたとしても、結果とは大きく異なってしまいます。
具体的にはどういう計算をしたんですか?
397ご冗談でしょう?名無しさん:01/12/15 11:07 ID:???
d^2θ/dt^2=式
を求めるには
式をtで2回積分せよということですか?
398ご冗談でしょう?名無しさん:01/12/15 18:15 ID:TSiV32p+
プラズマ物理って何ですか?
プラズマの権威といえば誰でしょうか?
399ご冗談でしょう?名無しさん:01/12/15 19:23 ID:9+tHfRdn
大槻だろ。
400ご冗談でしょう?名無しさん:01/12/15 21:39 ID:???
大槻って物理界ではどういう位置なの?
すごい人?それとも…
401理学部:01/12/15 23:23 ID:Wbdaeanx
>>397
そうですが、角速度ωを使えば一階の微分方程式になりませんか。
まぁθで書かねばならないならtで2回積分して下さい。
402349:01/12/15 23:30 ID:RpwkMhbX
>>354,355
回答とつっこみありがとうございます。自分はネタでなくまじで分かりません。
電車の中でジャンプしたときに足下に着地する理屈はわかっているつもりです。
で、自分としては多分このばあいは着地点はズレるとおもうんですが、ほんとうは
どうなんでしょう
403ふとした質問:01/12/16 00:50 ID:cfgdNbM2
質量120億tのパチンコ玉大の物質を
地球上で人間の手の上にそっと置いたら、
どうなりますか?
404ご冗談でしょう?名無しさん:01/12/16 01:08 ID:+RiE0AKy
宇宙は膨張してるという話を聞くのですが、
それは宇宙というモノが広がってるんですか?
宇宙という空間自体が広がってるって事はないんですか?

何かわかりにくいですが、イメージ的には、
前者だと宇宙の端が遠ざかってる
後者だと宇宙の膨張と共に原子自体が膨張してる
って感じです。
405ただの化学屋:01/12/16 03:43 ID:AG79Jxt4
>>403
手のひらを完全に固定した場合は、全重量がそのままかかることになりますが、
たとえば、手のひらを9.8m/s^2以上の加速度で落下させてやれば、
暖簾に腕押しというか、どんなに重い物でも重さを感じずに済むはずですね。
おそらく120億t分の重さを感じる前に、支えきれずに手のひらを返してしまうか、
玉を載せたまま地面に落ちる(倒れる)かのいずれかでしょう。
後者の場合は、地面に到達した後で、手のひらに穴があくことになりますね(w
406ご冗談でしょう?名無しさん:01/12/16 09:34 ID:???
>>404
よくある例えとして点を打った風船を膨らますイメージです。
407ff:01/12/16 14:12 ID:???
質問です。量子の世界ではトンネル効果なるものがあるそうですね。
人間は壁を通過することはできますか?
たしか可能性は0じゃないですよね?いったい何%の確率で通過できます?
408ご冗談でしょう?名無しさん:01/12/16 14:14 ID:???
ほとんど0%
409ff:01/12/16 14:17 ID:???
408>それはわかってます。
プランクスケールの数値なんだろうけど、いくつくらい?
計算してほすぃ
410こうぼう:01/12/16 14:22 ID:PiOzooL1
ボールが壁にあたってはねかえるのはボールの表面の粒子と壁の表面の粒子がぶつかってるからなんですか?
それとも壁とボールの表面の粒子どうしが電磁気力で反発して、
それで跳ね返ってるように見えてるんでしょうか?
誰か教えてください・・
411ご冗談でしょう?名無しさん:01/12/16 14:48 ID:???
>>410
では粒子同士がぶつかるってどういうことだと思いますか?
412ご冗談でしょう?名無しさん:01/12/16 15:11 ID:R19yebyb
>>410
物体表面の反発とか吸引とかは、ほとんど関係ないよ。
主要因は、物質内部の原子間結合の伸縮や屈曲、分子間の反発等々。
413ご冗談でしょう?名無しさん:01/12/16 16:13 ID:???
く、科学雑誌とかにブラックホールのイラストが
次元を一つ落として、三次元で書かれているでしょ。
でも、じっさいはあの穴はどう理解したらいいんだ?
時空が引き伸ばされているっていうこと?
さらに分けのわからんのがワームホールとかいうもの。
もう一次元プラスして元に戻して考えると、なんだかとっても
イラストでごまかされているって気がするんですけど?。
空間に穴があく・・・想像できません!!
414ぴぃ(・ε・):01/12/16 16:18 ID:???
ポワンカレ予想(リンゴの〜〜)とかいうヤツは、もう出来ちゃったんでしょうか?
どなたか、ご存じないですか?
僕は英語は全く駄目で…。
415ご冗談でしょう?名無しさん:01/12/16 18:31 ID:???
>413
三次元じゃなくて二次元でそ。普通に宇宙空間に恒星が浮かんでるとそれは点光源とみなせる
でしょ。恒星から離れるほど受ける光も少なくなる。重力は「黒い光」を放射しているとでも
いえばよいかね。いや、イメージの話だよ。いっぺん3Dソフトなんかで色のついた光源を一杯
浮かべてくるくる回して見れば分かりやすいと思う。

ワームホールは分からん。
41677:01/12/16 19:48 ID:???
>>414
数学板に逝ったほうがいいよ
417ぴぃ(・ε・):01/12/16 21:06 ID:???
>>416
あちらじゃ、いやはや、放置プレイでして…。
出来たような感じだったのですが、諦めますわ、
どもでした。m(_ _)m
418ふとした質問:01/12/16 21:34 ID:???
どうもサンクス。
でも地表に物質が達するとどうなるの?
どこまで沈んでいくの?
419ff:01/12/16 22:42 ID:???
うわ
俺の質問こたえてねーじゃん。
早く数値だしてくれよ。人間の大きさは平均的な人間でいいよ。
壁は正方形厚さも適当でいい。
できないの?大学いってるの?
420ご冗談でしょう?名無しさん:01/12/16 22:53 ID:???
>>419
トンネル効果ってエネルギーのポテンシャルを乗り越える効果のことだよ。
421ff:01/12/16 23:04 ID:???
だよ
それで人間が壁を一体いくらの確率で通過できるかおしえてほしい。
0じゃないのは解かってる
422どちゅん:01/12/16 23:11 ID:???
素粒子って変形するんですか?
変形しないとすると、素粒子を何かで押した時
押された逆側への力の伝達速度は光速を超えるのでしょうか?
423ご冗談でしょう?名無しさん:01/12/16 23:13 ID:???
>>421
だいたい1/(10^10^10^10^10^10)くらいです。
424ご冗談でしょう?名無しさん:01/12/16 23:23 ID:???
>>422
素粒子には大きさがありません。
逆にいうと、大きさがないものしか素粒子とみなされません。
大きさがないものに変形を考えることは無意味です。

陽子みたいに大きさのある粒子は変形可能です。
原子核とかだと、変形核の研究は非常にちゃれんじんぐです。
425ご冗談でしょう?名無しさん:01/12/17 00:14 ID:QN6cSJFg
よくみるんですが、「をっさん」とか、「田中みつ人」って誰なんですか?
426フヘフヘフヘ ◆XYN2P2Tw :01/12/17 01:54 ID:XitpwHf7
こんばんは。初めて物理学板に来ました。
いまレポートしてるんですが、X線回折のことで教えてください。
X線回折の構造因子のダイアモンド型立方格子の場合なんですが、
ミラー指数hklの和が、4mの時とか4m±1の時とか4m+2の時とかってあるんですが、
mは何を指している文字なんですか?
調べてもmが何を指しているかの情報を見つけられませんでした。
明日の昼1:30が締め切りのレポートなんです。どなたか教えてください。
ではお風呂に入ります。出てきたらまた除きます。お願いします
427410:01/12/17 01:56 ID:???
>>411
例えば電子と電子は近づき過ぎると一気に反発しちゃいそうだからぶつかれなさそう・・。
原子核と原子核の場合は衝突すると(かなり激しく運動していないとありえないそうですが)核融合しそうです・・
だから普通は物質同士がぶつかることってないんじゃないかって思いました。

>>412
分子間の反発というのは電磁気力によるものですよね?
428ご冗談でしょう?名無しさん:01/12/17 02:10 ID:???
>>427
何をもって物質と呼ぶのか・・
大きさとは・・
429ご冗談でしょう?名無しさん:01/12/17 02:59 ID:???
>>426
整数ちゃうか。
430物理学者さん教えて:01/12/17 05:12 ID:2VKKjAM7
未来はひとつしかないの?
431ご冗談でしょう?名無しさん:01/12/17 05:33 ID:???
>>426
お前、前に半角板にいただろ。このクズ。
432プシン:01/12/17 05:41 ID:EQwpcrHW
宇宙で息を止めたら宇宙服を着なくても
生きていられるんですか?
(太陽が熱いとかは別として)
433ご冗談でしょう?名無しさん:01/12/17 05:59 ID:???
>>432
気圧の差で沸騰して死ぬというのは聞いたことある。
434ご冗談でしょう?名無しさん:01/12/17 08:33 ID:???
たった1気圧の差に耐えられないなら、
素潜りで10m潜ったらオブチさんじゃなくてオダブツさん((c)外務大臣)
435ご冗談でしょう?名無しさん:01/12/17 08:53 ID:???
掃除機程度の吸引力(陰圧)で皮膚がどれだけ膨らむか試そう
掃除機がもっと強力で全身がすっぽり入るくらい巨大だったらどうだろう
436ご冗談でしょう?名無しさん:01/12/17 09:15 ID:???
>>434
圧力の方向が逆だろが
437フヘフヘフヘ ◆XYN2P2Tw :01/12/17 11:18 ID:???
>>429
ありがとうございますそうですか整数ですか。
この場合のmは、nと同じように使われているということですか。わかりました。

>>431
>お前、前に半角板にいただろ。このクズ。

さあ・・なんのことか。。。
438ご冗談でしょう?名無しさん:01/12/17 12:31 ID:v0rEo9w7
とりも作用反作用だろ。
439ご冗談でしょう?名無しさん:01/12/17 12:39 ID:C1q+VQJK
相図で、転移線(?)が切れた先の高温高圧の領域はどうなってるの?
気体相でもないし液体相でもない相って何?
440sage:01/12/17 12:45 ID:???
>430
未来はまだひとつもありません。
441ご冗談でしょう?名無しさん:01/12/17 12:46 ID:z0Bvzheg
>>439
超臨界状態又は超臨界相
442ご冗談でしょう?名無しさん:01/12/17 12:57 ID:???
>>441 どうも。その言葉で検索してみたら色々でてきました。
”高密度の気体”というのが実態みたいですね。
443sage:01/12/17 13:08 ID:GQE/lVPz
突然割り込んですみませんが、質問です。
金属のNMRのナイトシフトはどうして正の値になるのですか?
どうか教えてください。
444ご冗談でしょう?名無しさん:01/12/17 13:23 ID:aTCgb9fK
絶対零度ってどうやって測ったらいいのですか?
445 :01/12/17 13:33 ID:IlmRZjhm
E1遷移確率って何か分りやすく短く説明していただけませんか?
マジでお願いします。
446ご冗談でしょう?名無しさん:01/12/17 18:05 ID:???
>>436
だから? 陰圧だと1気圧にも耐えられずに細胞壁や血管が破れて
体液が沸騰するとでも? そんなヤワじゃないぞ。
肺胞は弱そうなので完全に真空にさらしちゃうとまずいけど、
息は止めとく前提でしょ?
447ご冗談でしょう?名無しさん:01/12/17 18:16 ID:595nj8hp
>>431
半角板住人発見!
448これってできるの?:01/12/17 19:15 ID:n1TGi1AI
>女子トイレの便器にたまってる水に細工して、
>電気系統から漏電させておいた。

>会社のOL放尿したと思ったら、ぎゃーーーーー、
>という声、わざとらしく、どうしたの?って
>聞いたら、しどろもどろ、
>あそこを感電したなんて言えないのか。
449ご冗談でしょう?名無しさん:01/12/17 19:27 ID:???
同期機と誘導機の違いについて詳しく教えて下さい
450ご冗談でしょう?名無しさん:01/12/17 19:33 ID:4mSBT7o2
皆さん物理は好きですか
451ご冗談でしょう?名無しさん:01/12/17 21:20 ID:???
>>436 >>446
まあ実験しないとわかんないよねこういうのは

>>450
好きでもないのにこんな金にならないことしない
452ご冗談でしょう?名無しさん:01/12/17 22:18 ID:C2Wvs0AN
相対性理論入門のイイ本ないでしょうか?
453ご冗談でしょう?名無しさん:01/12/17 22:19 ID:???
>>451
アサハラで実験しましょう
454ご冗談でしょう?名無しさん:01/12/17 22:52 ID:???
今日は2ちゃんのいろんな板ですごいことになってるけど、
ここはいつもどうりで安心したよ。
455ご冗談でしょう?名無しさん:01/12/18 00:47 ID:???
ニュートリノってどれくらい浴びると健康に悪影響が出ますか?
456ご冗談でしょう?名無しさん:01/12/18 01:15 ID:???
"An oscilating field is a superposition of two fields rotating in opposite sense."
これを数学的に説明せよ、という質問には、どう答えたらよいですか?
457ご冗談でしょう?名無しさん:01/12/18 02:18 ID:BVWeUvUe
何故反物質が自然に存在していないんでしょうか?
新しい原子がよく見つかりますが、実際いくつまで出てくるんでしょう?
今現在分かっている範囲で、最も安定した物質は何でしょうか?
458理学部:01/12/18 02:46 ID:y40kCRC+
>>455
ニュートリノは地球ですらスイスイ貫通してしまいます、
人体内で相互作用を起こして悪影響を及ぼすことはありません。
459ご冗談でしょう?名無しさん:01/12/18 02:54 ID:???
>>458
いい加減な事を説明するのは止めましょう。
大量に浴びたら危険です。
本当に理学部なんですか?
460ご冗談でしょう?名無しさん:01/12/18 02:57 ID:???
>>459
んじゃー健康に害を及ぼす量のニュートリノビームきぼんぬ
さあよこせ
今よこせ
461ご冗談でしょう?名無しさん:01/12/18 02:57 ID:???
>>458
それでもベラボウな量のニュートリノを浴びるとよろしくない。

もし太陽くらいの距離のところで超新星爆発が起きたとすると
他の放射線・熱その他全てをシールドできたとしても
ニュートリノだけで致死量になるとか。
462ご冗談でしょう?名無しさん:01/12/18 03:08 ID:???
そりゃあ土砂だって大量に浴びりゃあねぇ
463ご冗談でしょう?名無しさん:01/12/18 03:11 ID:???
このスレの370以降論争が続いております。どなたか説明してやってはくれませぬか?
http://life.2ch.net/test/read.cgi/kankon/1004875847/l50
464ご冗談でしょう?名無しさん:01/12/18 03:11 ID:???
>>458 4体フェルミ相互作用の性質を知らないと思われ。
エネルギーが高いとニュートリノは電子などと大差ない断面積を持つ。
まあ、エネルギーが低くても大量に浴びればなぁ....
だれかマジな計算キボンヌ!
465 ◆P/Pe9sxI :01/12/18 03:14 ID:sRqGvWkR
>>457
●何故反物質が自然に存在していないんでしょうか?
  宇宙開闢時に、10^(-10)ほど(?)の僅かな非対称が原因です。
  物質:反物質=10000000000:9999999999だったという事です。
●新しい原子がよく見つかりますが、実際いくつまで出てくるんでしょう?
  見つかるのではなく見つける・作るというのが実際です。
  いくつまでかは分かりません。現在は…118番ですか。
  120番くらいからは安定する可能性があると誰かが言っていたような気がします。
●今現在分かっている範囲で、最も安定した物質は何でしょうか?
  これは例えば陽子とか中性子とかの話ですか?
  それとも水素とかヘリウムとかの話ですか?
  それともダイヤモンドとかポリエチレンとかの話ですか?
  あと、「安定」というのはどういう面での安定ですか?
466456:01/12/18 03:35 ID:???
>456の質問もお願いします。
467ご冗談でしょう?名無しさん:01/12/18 03:37 ID:???
>>464
じゃあエネルギー高いニュートリノをたくさん用意してみてよ
468ご冗談でしょう?名無しさん:01/12/18 03:46 ID:???
つーか大量に浴びて害のない高エネルギー粒子ってあるのかい>461
469ご冗談でしょう?名無しさん:01/12/18 03:47 ID:???
なんか、エネルギーフロンティアを狙おうとして、μをストレージして衝突させる実験を
考ようとすると、ストレージしてあるμの崩壊で高エネルギのニュートリノがおまけで出てくる
のだけど、人体にも有害で中途半端にしか遮蔽できないので、これが結構曲者らしい。
470ご冗談でしょう?名無しさん:01/12/18 04:04 ID:???
>>457
★自然界にも微量だが反陽子は存在する。「BESS 反陽子」 で検索しる。
 なぜ反陽子がほとんど無いかは、現時点では不明。幾つかのモデルは提唱されている。
 >>465 の説明もGood。「バリオン数 生成」や「バリオジェネシス」で検索しる。
★ナイーブには原子番号126(=マジック数)の安定な原子(核)が存在しそうなのですが、
 原子核屋じゃないのでパス。
 # 114とか184なんて言うマジック数も聞くがどうなってるの? >> 原子核な人
★陽子、電子が崩壊すると言う実験結果や観測はまだ存在しない。
 陽子の寿命は10^31年から10^33年程度より長いと言う事しか分かっていない。
 よって、観測事実として今の所安定。
 しかし、現在確立されている標準模型の範疇でも、陽子はインスタントン過程で崩壊すると
 予言されている。(っうーても、めっちゃ長い。)
 また、統一模型ではもっと早く陽子が崩壊する可能性がある。「陽子崩壊」などで検索しる。
471ご冗談でしょう?名無しさん:01/12/18 05:32 ID:???
>>468
だから、最初の質問はどれくらい浴びると危険なのか聞いてるのじゃないかな?
472ご冗談でしょう?名無しさん:01/12/18 11:19 ID:E9uTanKj
原点に点電荷Qがあるとする。原点にしか電荷がないので、原点以外では
静電ポテンシャルはポアッソンの方程式でρ=0としたラプラスの方程式Δφ=0
から求められるはずである。ところで、実は原点に点電荷Qがある場合の静電ポテンシャルは既にわかっていて、
φ0(x,y,z)=(Q/4πε0)(1/r)である。従ってφ0は原点以外でラプラスの方程式の解、すなわちラプラスの方程式を
満たしているはずである。このことをφ0を実際にラプラスの方程式の左辺に代入することにより確かめよ。
 長くてすいません。この問題はどうすればいいのですか。教えてください。お願いします。
473ご冗談でしょう?名無しさん:01/12/18 12:31 ID:???
Δφ0を実際に計算すればいいだけだろが
Δって何?っていうレベルならアヒャ
474457です:01/12/18 20:59 ID:EN4aGeWo
≫465さん
ありがとうございます。
■では逆に、理論上別に宇宙があったとしたら、反物質だけの世界なども存在するんでしょうか?

■水素・ヘリウム等で他の物質と反応しにくい超高温高圧による温度変化にも反応しにくい物が知りたいです。
原子崩壊?しにくい物質です。意味不明かもしれませんが教えてください。お願いします。
≫457さんもありがとうございます。
宇宙の(時間的)終わりには陽子が崩壊して光がどうとか言うのを何かで読みました。
少し調べてみます。では
475ご冗談でしょう?名無しさん:01/12/18 21:03 ID:G4i5+8HO
質問よろしくお願いします


固定端では 位相がπ変化しますよね このπ変化とか
位相でみると π増加なんでしょうかπ減少なんでしょうか?

実は次のような問題があったんです


ABと CD の平行な2直線がある この平行な2直線の距離はdである
光りがこの直線に直角に直線AB側からすすんでゆく。 ABでは光は固定端で CDでは自由端である
ABで 反射した光と CDで反射したひかりの位相差をもとめよ
ただし 光の波長をλとする
そして2直線の間には屈折率nの液体をこめてある

この問題で
僕は 2dn*2π/λ-πと答えをだしたのですが

解答には2dn*2π/λ+π  とありました

僕の答えでも正しいでしょうか?
476ご冗談でしょう?名無しさん:01/12/18 21:30 ID:???
>>475
×をつけられたら小一時間そいつを問い詰めてよし
477ご冗談でしょう?名無しさん:01/12/18 23:59 ID:hkC9CJyj
難しいなあ
478ご冗談でしょう?名無しさん:01/12/19 00:36 ID:???
3Kの宇宙背景輻射(CMB)が等方的に見える座標系(慣性系)ってのは
宇宙の進化のどの時点で決定されたのでしょうか?
宇宙の晴れ上がりの時点でのプラズマの静止系(重心系)と言い直しても良いのかも知れません。
ナイーブにはインフレーション直後の再加熱までは遡れそうですが…

また、そのような事を議論している文献があれば教えて欲しいです。
479ご冗談でしょう?名無しさん:01/12/19 02:10 ID:Y8+WXtHO
470, 478 にある「ナイーブ」ってどういう意味ですか?
480ご冗談でしょう?名無しさん:01/12/19 03:59 ID:???
カネボウ
481明菜:01/12/19 04:42 ID:2BooVaW4
助けてください。
@宇宙空間を速度Vで飛んでいるロケットは、一つの閉じた系を形作っている。
今、燃料を噴射して速さを10%増したいとき、噴射速度を2Vとすれば燃料を
どれだけ使えばよいだろう。

A無風よた命陽を、速度Vで走る自動車がある。Sを運動方向と垂直な断面積として
ρを大気密度とすると、自動車に働く空気の抵抗は、どのように表せるか。
ただし、空気分子と自動車との衝突は完全な非弾性衝突として空気の粘性は
考えない。

宜しくです〜。
482ご冗談でしょう?名無しさん:01/12/19 04:42 ID:2BooVaW4
2番無風よた命陽を=無風状態の道路を
483ご冗談でしょう?名無しさん:01/12/19 08:01 ID:m1IeKYQL
あのー…
哲学板で↓こんなこと言ってる人がいるんですが…

>でも宇宙は無から生じたというのは現代の宇宙論の定説だろう。

実際どうなんですか??
484ご冗談でしょう?名無しさん:01/12/19 09:44 ID:Euq57+C5
突然すみません質問です。生物分野のもんですが。青山学院からの超伝導の論文(nature)は物理学分野ではむちゃくちゃexcitingな論文だそうですが、なんでも学生が発見したそうですね。真実の所はどうなんですか>
485ご冗談でしょう?名無しさん:01/12/19 09:53 ID:X2y7S7FZ
警官が威嚇で上空へ向けて発射した拳銃の弾はどこへ落ちるのですか?
誰かに当たるのではと心配で夜も眠れません。
486ご冗談でしょう?名無しさん:01/12/19 10:48 ID:???
>>485
祝砲を景気よく空に打ち上げる習慣のある国では、落ちて来た
銃弾によると思われる怪我が多いと聞きました。ヨタかも
しれませんが。まぁ、どこかに落ちているのは間違いない。
487ご冗談でしょう?名無しさん:01/12/19 12:23 ID:gazmoXaH
>>486
危ないすねえ
その際は初速の何%ぐらいになってるもんなんすかねえ
488ご冗談でしょう?名無しさん:01/12/19 12:41 ID:???
2ch物理板住人の基本姿勢として・・・
自力で問題を解決しようと努力する精神に対して手助けする


だよな?あれ違ったか?
489ご冗談でしょう?名無しさん:01/12/19 13:10 ID:???
>>484
何が知りたいの?
研究内容の詳しいことなら物理学会誌探せ。
ゴシップ的なことなら板違い。
てめえ自身の妄想で勝手に補完しとけ。
490教えて:01/12/19 14:55 ID:J1whDUDu
あるクイズサイトからの出題です。
わからないので教えてください。

次の選択肢のうち電気がもっとも通りにくいものはどれでしょう?
1.てんぷら油
2.10円玉
3.ウーロン茶
4.シャーペンの芯

1か2だと思うのですが、どちらかわかりません。
よろしくお願いいたします。
491490:01/12/19 14:56 ID:J1whDUDu
すいません間違えました。

1か3だと思うのですが、どちらかわかりません。
492ご冗談でしょう?名無しさん:01/12/19 15:18 ID:???
油じゃないか?
493質問:01/12/19 16:14 ID:renhRgS3
宇宙ロケットが光の速さ近くで進むと時間が遅くなるといいますが
相対的に宇宙ロケットから地球を見れば、
地球が光の速さで遠ざかってると見えるのに
なぜ宇宙ロケットのほうが時間が遅くなるんでしょうか?
494ご冗談でしょう?名無しさん:01/12/19 16:53 ID:???
>>493 有名なバナッハ・タルスキーのパラドックスです。
495ご冗談でしょう?名無しさん:01/12/19 23:24 ID:7RiOe1Wr
初めてこの板に来たんだけど、面白い(興味をひきそうな)スレってどれか教えて。
物理は9.8m/s*sくらいしか覚えていない28歳です。
496ご冗談でしょう?名無しさん:01/12/19 23:31 ID:Qs4kbiy7
497495:01/12/19 23:50 ID:???
>>496
ありがとう。
498sage:01/12/20 00:08 ID:???
>490
典型的なひっかけ問題です。
ここでの電気とは、「おい電気つけろ」等というときの電気で電灯を指します。
つまり1や3の液体は簡単に通り抜けますし、4もへし折って貫通できます。
従って答えは2の10円玉。
ところでクイズの賞品はなんですか?
499かげお:01/12/20 00:51 ID:Rh55uCdB
>>478
3Kの宇宙背景輻射は厳密には等方ではありませんから、そのような座標系は
便宜的なものでしかないでしょう。しいて言えば、各時点で決定される、です。

>>485
希に誰かに当たる例があるそうです。
空気抵抗があるので、発射時に比べればはるかに減速されているでしょうけど。

>>493
宇宙ロケットから地球を見れば、地球の方が時間が遅いですよ。
500478:01/12/20 01:19 ID:???
>>499 ありがとうございます。
確かに、3Kの背景輻射は10^-5くらいの揺らぎを持っているため等方的ではありません。
しかしながら、ダイポール的な揺らぎ(というより前後の非一様性)は特に大きく、これは
(ほぼ等方的な)背景輻射に対する観測者の運動と理解できます。実際、COBEの
観測結果で宇宙論を議論する時も、このダイポール揺らぎは取り除き、高次モーメント
の揺らぎのみに着目しています。
でもって、知りたいのはこの観測者の運動というのは何に対する運動なのか?
と言うことです。安直には
観測者の運動=COBE衛星の運動+地球の公転+太陽の銀河内公転+銀河の運動+...+(謎の静止系)
な訳ですが、その(謎)が知りたいわけです。
501ご冗談でしょう?名無しさん:01/12/20 02:21 ID:???
>>500
現在見える範囲にある宇宙がinflationで急激に膨張する直前の、
物質・輻射すべての重心系に対してじゃないかな
inflationが起きる直前のほんの一点の状態が今見える範囲すべてに
引き伸ばされているだけで、特別な慣性系だったというわけではないでしょう。
今見える範囲の宇宙にとっては重心系だからそういう意味では特別だろうけど。
502ご冗談でしょう?名無しさん:01/12/20 03:08 ID:2bi2XyQy
今電磁気勉強しようとしてる大学1年なんだけれども、
本でお勧めあったら教えてくだせえ。

今ん所、ファインマン物理学がいいかなと思っているんですが、
1年にはつらいという話も聞きました。
503ご冗談でしょう?名無しさん:01/12/20 15:16 ID:???
>>502
図書館逝って自分で探せ!ヴォケ!!
504ご冗談でしょう?名無しさん:01/12/20 15:19 ID:???
>>499
>宇宙ロケットから地球を見れば、地球の方が時間が遅いですよ。

そんなこと無いでしょ。
505ご冗談でしょう?名無しさん:01/12/20 16:16 ID:???
>>504
そんなことあるでしょ
506ご冗談でしょう?名無しさん:01/12/20 23:53 ID:bwdwSIs/
なぜ光は光路差が微小な時のみ干渉するんですか?
実際そうなのはわかるんですが、光が波動性を持つなら干渉してもいいような気がするんですが。

あと、ニュートンはニュートンリングをどうやって作ったんですか?
507ご冗談でしょう?名無しさん:01/12/21 00:16 ID:???
>>506
そんなに光路差が大きくずれるようだとちょっとした
ことで位相差がすぐバラバラになって干渉が見えなくなるのでは?
508ご冗談でしょう?名無しさん:01/12/21 02:07 ID:80JNYiZQ
割り込みで新しい疑問。スマソ
ナイトライダーのオープニングでナイト2000が
走行中のトレーラーに乗り込むんだけど、よく考えたら
むずかしいと思うんだけど。
509困ってる人:01/12/21 02:20 ID:egN1DzWu
交通事故で停車中の車に後ろから追突された場合、停車中の車に乗車している
人はどれくらいの衝撃を受けるのでしょうか?どなたか物理的に教えてください。お願いします
510 ◆P/Pe9sxI :01/12/21 03:23 ID:vmgyrVWm
>>485
米国で、警官の威嚇射撃の弾が隣町(?)の通行人に当たって死ぬ事件がありましたね。
いつだったかは忘れました。

>>493
「時間がおそくなる」というのが“時間がゆっくりになる”という意味であれば、
>>499 にあるとおり、地球から見たロケットと同じく、
ロケットから見た地球もゆっくりになります。

>>509
追突した車・追突された車・乗っている人間のそれぞれの重さがあって、
追突した車の速さがあれば、大雑把に運動量保存則で乗っている人間の追突後の速さが求まります。
あとは運動量=力積という関係を使って、乗っていた人間に加わった力を求めることができるでしょう。
511ご冗談でしょう?名無しさん:01/12/21 04:30 ID:K/7YiX+T
>>508
そんなこと無いよ
動く歩道と一緒
512文系なんで。。:01/12/21 04:39 ID:4lsjILtn
すいません。
全然異分野なんで相対性理論に触れるのは初めてなんですが、
この「誰でもわかる相対性理論」に疑問を感じました。

http://monkey.hoops.jp/mag/mag00009.htm

特にここです。
電球を付けたら左から右ってところが全然わかりませんでした。
これって正しい説明ですか?
私がドキュソなだけでしょうか?
513ご冗談でしょう?名無しさん:01/12/21 07:13 ID:zNC9xKzv
1次コイルによって発生した磁界によって2次コイルで
相互誘電起電力が生じますね その時1次コイルで発生した
磁界の逆向きに磁界を作るような 起電力を2次コイル
では作るとおもうのですが
結局2次コイルには どっちの方向の磁界ができているのでしょうか?
 初めの1次コイルによってできた磁界の方向
ですか?それとも相互誘電によって生じた 方向ですか? どうもわからないので みなさんの意見を
ききたいのです よろしくお願いします
514ご冗談でしょう?名無しさん:01/12/21 09:36 ID:???
>>508 路面で加速しておいて、トレーラーとの相対速度を
トレーラーに乗り上げられる大きさまで増加させます。
そして乗り上げる前にクラッチで動力を切断すればいいのです。
ナイト2000にクラッチがあるかどうかは知りませんが。
やめてくださいマイケル!
515ご冗談でしょう?名無しさん:01/12/21 10:16 ID:???
正直、クラッチの無い動力系は嫌過ぎ。
516>508:01/12/21 11:45 ID:???
むずかしいことはむずかしい
でもできない相談ではない
517ご冗談でしょう?名無しさん:01/12/21 18:14 ID:???
>>510
では、また宇宙ロケットが地球に戻ってきたら
どっちの時計が遅れてますか?
518ご冗談でしょう?名無しさん:01/12/21 18:27 ID:???
>>517
戻って来るならロケットのほうが遅れる
519ご冗談でしょう?名無しさん:01/12/21 19:46 ID:713x1yBj
この板の上のバナーの人は誰ですか?ガリレオですか?
520ご冗談でしょう?名無しさん:01/12/21 19:47 ID:???
さくらたんだろ?
521520:01/12/21 19:52 ID:???
間違えた。>>519はロゴのことを言っているのね?
顔写真の横に式が書いてあるでしょ?その式に関係のある人です。
(物理板では「ロゴの人は誰か?」と聞くのは禁じられているとかいないとか・・・)
522ご冗談でしょう?名無しさん:01/12/21 21:27 ID:+URu8ua7
>>513
一次コイルによる磁界と同じ向き
523遅レスでスマソ♪:01/12/21 22:55 ID:???
>>512
結果としては正しいのですが、思考実験にしては論理が飛躍しすぎています。
(普通の人が普通に推論すれば、逆の結論になるはずです。)
ドキュソであろうがなかろうが、理解できないのは当然です。
524ご冗談でしょう?名無しさん:01/12/21 23:17 ID:???
>>523
どこが?ふつーーーーーーーのことしか言ってないじゃん。
日本語がネイティブで話せる人だったら誰でも理解できると思うけど。
525遅レスでスマソ♪:01/12/21 23:25 ID:???
>>512
自由落下するエレベーターの中では、重力は0になったように感じられます。
(地球を周回するスペースシャトルの内部も同様です。)
逆に、上向きに9.8m/s^2で加速度運動するエレベーターの中では、
重力は2倍になったように感じられます。
これらの加速度運動によって生じる(みかけの)重力も、
天体の引力によって生じる重力も、本質的に同じであると認めることが
「等価原理」であり、一般相対性理論の出発点です。
526ご冗談でしょう?名無しさん:01/12/21 23:33 ID:???
>>524
相対性理論をすでに知っているから何とも思わないだけだろう。
たとえば、これが地上に静止している箱ならどう考えるつもりだ?
527ご冗談でしょう?名無しさん:01/12/21 23:39 ID:???
>>524
要するに、内容をハショリすぎているわけよ。
一見して言葉づかいは平易だが、これだけでワカレと言うのは無理。
オタクな技術者の作ったアニュアルのようなものだな(w
528ご冗談でしょう?名無しさん:01/12/21 23:43 ID:???
>>518
なぜ地球じゃなくロケットのほうが遅れるんですか?
529ご冗談でしょう?名無しさん:01/12/22 00:27 ID:???
加速度運動するから。
530ご冗談でしょう?名無しさん:01/12/22 00:28 ID:???
>>525-527
こんなレスしてる人だぜ?
それ以前の段階さ。

157 名前:144 :01/12/21 23:11 ID:Bn4wf1vI
なるほど。
自分はあの文を読んで箱の外側の上に乗るのかと
思ってました。
でも、その勘違いが晴れても、上下運動しかないのに
左右の動きができるってのがどうも理解不能です。
箱と一緒に落ちるし、目で左側の光を追ったところで
左右の動きが加わるなんて到底考え付かないです。
(簡単な思考実験???)

「誰でもわかる相対性理論」なんていっていますが
既にわからないものがここに一名いますね。
531かげお:01/12/22 01:13 ID:GolLDycH
>>528
地球は常にほとんど同じ慣性系にいたのに対して、宇宙ロケットは行きと戻りで別の
慣性系にいたから。

>>530
そのウェブページは、図が無いのが最大の問題だと思います。
532ご冗談でしょう?名無しさん:01/12/22 01:18 ID:???
こんな板立てたので興味ある人は遊びに来てください
広告スマソ
〜電子制御・非線形根性〜
http://jbbs.shitaraba.com/study/18/d93.html
533国語算数理佳社会:01/12/22 02:27 ID:Oo+AYRfS
http://www.geocities.co.jp/WallStreet-Bull/3391/fibo.htm
http://www.geocities.co.jp/WallStreet-Bull/3391/GIF/rasengol.gif
誰かこの図形の規則制について説明して。
黄金比を調べてたら出てきた。
πという映画でもこんな感じのが出てたと思うんだけど。。
何と無くはわかるんだけど、明確に理解できないす。
x2のとき、yは3、y3の時x-方向に5、、の時y-方向に8。。
これは公式だとどうなるの?
アホな質問でゴメンね。
534512:01/12/22 04:17 ID:???
本スレのほうで解決いたしました。
話はそっちでしています。
ありがとうございました。
535ご冗談でしょう?名無しさん:01/12/22 18:02 ID:???
>>531
宇宙ロケットから見れば地球が別の慣性系にいたと見えませんか?
536名無しさん:01/12/22 18:04 ID:???
537ご冗談でしょう?名無しさん:01/12/22 18:54 ID:zXcsAJm1
すみません、教えてください。
ヒッグス粒子って実在する可能性はあるのですか?
538ご冗談でしょう?名無しさん:01/12/22 18:57 ID:???
>>537
 探している。
539ご冗談でしょう?名無しさん:01/12/22 21:25 ID:???
可能性は高いと思います。
ぜひ、>>537さんがJLCを作って探してくれることを望みます。
540ご冗談でしょう?名無しさん:01/12/22 21:40 ID:zXcsAJm1
JLCってなんですか?
食べ物ですか? JRのとのだちですか?
541ご冗談でしょう?名無しさん:01/12/22 22:34 ID:???
>>535
自分が加速すればわかるでしょ。
542539:01/12/23 07:35 ID:???
>>540
とりあえず、ここみてみれ。
ttp://lcdev.kek.jp/JLC/overview/01.cover.html
543かげお:01/12/23 22:52 ID:ZRT6i+Xw
>>535
>宇宙ロケットから見れば地球が別の慣性系にいたと見えませんか?
そう『見える』だけで、実際は地球は常にほとんど同じ慣性系にいました。
一方、宇宙ロケットは明らかに途中で慣性系を乗り換えて(加速度運動して)
います。それで、529の回答にもつながります。
544標準くん:01/12/24 00:13 ID:PIqXUicD
ヒッグスや超対称性がなかった場合のシナリオを教えて下さい
545ご冗談でしょう?名無しさん:01/12/24 00:24 ID:wfQk7xRu
オイラーの方程式て何でしょうか。
ラグランジェからどう求めるのですか?
変分原理は少し分かりますが…
546ご冗談でしょう?名無しさん:01/12/24 00:50 ID:???
>545
 釣りですか?
547初心者:01/12/24 01:19 ID:V8Ua1OAO
変分ってなんなのでしょうか?
簡単な式で例を教えていただけるとありがたいです
548ご冗談でしょう?名無しさん:01/12/24 02:43 ID:tQVC87mK
えと また疑問です
レーザーって光だから波だよね?
どうして拡散せずにまっすぐ進んで行くんでしょ???
549 ◆P/Pe9sxI :01/12/24 05:05 ID:Cv2TV8J9
>>548
拡散せずにまっすぐ進ませるようにした光がレーザーなんです。

「また疑問」という前に検索エンジンで調べてみると色々と楽しめますよ。
550ご冗談でしょう?名無しさん:01/12/24 10:08 ID:Kx08xKLU
http://www.jpo.go.jp/info/pdf/1307-047_28.pdf
誰かこの問題解いて下さい
551ご冗談でしょう?名無しさん:01/12/24 12:00 ID:???
ヴィダルサスーンのCM(シャンプーかなんか)で、大学の教室において講義中にカップルがいちゃついてる作品がありますよね。
あのシーンの板書を見るとどうも量子力学をやってるようなんです。
ブラケットがたくさん並んでて。
あれは何を計算してるんでしょうか?
量子力学に関する知識が皆無なので判りません。
552ご冗談でしょう?名無しさん:01/12/24 12:51 ID:TSZ8qWU6
>>545 オイラー方程式は、へんぶん原理に基づくある種の方程式
の一般形。その特殊な場合がラグランジュ方程式。あるいは
ナビエ・ストークス方程式で粘性項をゼロにしてもオイラー方程式
になる。

ところで、ビール瓶振って栓あけるとぷしゅーって飛び出るけど、
あれ何で?
553名古屋:01/12/24 13:17 ID:M5NFXNwI
ラケット ブラケット表示?
ほんとかなあ
一瞬しか映らんでしょ?録画してるの?
554ご冗談でしょう?名無しさん:01/12/24 17:55 ID:???
lim_δ→0 [dψ(x_1+δ)/dx]=lim_δ→0 [dψ(x_1-δ)/dx]
ならば一回積分してψはx_1で滑らかになるらしいのですが何故ですか?
dψ/dx|x=x_1なら納得出来るのですけど。
555ご冗談でしょう?名無しさん:01/12/24 20:49 ID:???
そういう意味じゃないの?
556548:01/12/24 22:16 ID:wGpAt3Ku
>>549
ありがとうございます。
いろいろ調べたんですが
わかったようなわからないような。
どうして波が指向性をもつのかなあ。
音や水面の波が一方向に進むことあります?
557ご冗談でしょう?名無しさん:01/12/24 22:48 ID:???
平面波は拡散しませんよ。
558アンドロメダ:01/12/25 00:34 ID:???
私のは拡散しますが、何か?
559ご冗談でしょう?名無しさん:01/12/25 03:46 ID:PjZ2t/Lx
棒の先端に質量がついた系のパラメータ励振で、
振動数がかなり大きくなると棒が勃った状態で安定する状態が出てきますが、
それが解析的にうまく理解できないから、誰かかなり判り易く教えてください。
560小学生:01/12/25 06:25 ID:izcHWSkX
鏡に体を映すと、左右は逆になるのに
なんで、上下は逆にならないんですか?
教えて下さい。
561ご冗談でしょう?名無しさん:01/12/25 06:28 ID:???
>>560
左右も上下も逆になっていない。
逆になっているのは前後
562三村:01/12/25 12:19 ID:???
>>558
波動砲かよ!
563ご冗談でしょう?名無しさん:01/12/25 12:25 ID:???
>>561
よくある説明だが、ほんとにそれで説明になっているのかな?
前後逆になった像を見て、なぜ左右逆になっているように感じられるのかを
説明しなくてはいけないのでは。

私がいままで聞いた中で一番納得した説明は、
上下・前後・左右のうちで左右の定義だけが相対的で
前後と上下が決まらないと決まらないから、
というもの。
564ご冗談でしょう?名無しさん:01/12/25 12:28 ID:???
>>551
私にもブラケットにみえまーす。
生成消滅演算子らしきものも見えます。
でもビデオ持ってないのでとても判読できませーん。
565ご冗談でしょう?名無しさん:01/12/25 12:52 ID:wN9ASlcz
>上下・前後・左右のうちで左右の定義だけが相対的で
>前後と上下が決まらないと決まらないから、
>というもの。

それにもかかわらす、左右は方向と関係なく定義される(手とか)から

方向と関係ある左右と関係ない左右が言葉の上では区別されないので
言葉で考えるとわけわかんなくなっちゃう。
と言うことではないか。
566ご冗談でしょう?名無しさん:01/12/25 13:30 ID:???
>>563
それは心理学の問題であって物理の問題ではないのでは?
567ご冗談でしょう?名無しさん:01/12/25 13:44 ID:M3FKA+gs
tanθ=0.1 (θ=5.71°)になんでなるのかわかりません。
教えてください。
568ご冗談でしょう?名無しさん:01/12/25 14:13 ID:???
>>567
なにが言いたいのかわかりません。
教えてください。
569ご冗談でしょう?名無しさん:01/12/25 14:42 ID:M3BExKbK
>568
すみません。この問題です。
一辺の長さAの立方体が傾斜角θの斜面に置かれています。
斜面の静摩擦係数=0、1として、立方体が斜面を滑るときの傾斜角θを式で示してください。
また、立方体が斜面を転がるときの傾斜角θと静摩擦係数はいくらになりますか?

最初の方は tanθ>0.1 (θ=5.71°)になるらしんですが、なんでなるんですか?
570ご冗談でしょう?名無しさん:01/12/25 14:52 ID:???
しね
斜面に平行で下向きの力 f = mgsinθ
斜面に平行で上向きの力 f' = μmgcosθ (ここにμ:静止摩擦係数)
滑るときはf>f'の場合だから
mgsinθ>μmgcosθ
つまり
tanθ>μ(=0.1)
しね
571569:01/12/25 15:04 ID:???
>570
厨房すぎてすいません。
いっぺん氏にます。
572ご冗談でしょう?名無しさん:01/12/25 16:49 ID:0z5MySZ5
力学の問題を解くときに
結果をイメージ>解く
解く>イメージの確認
どっち?
573ご冗談でしょう?名無しさん:01/12/25 17:14 ID:???
力学ならまず図示してイメージを明確化できるか確かめるかな
574ご冗談でしょう?名無しさん:01/12/25 18:04 ID:HK4Qhjfj
育英会廃止決定!?で貧乏人のご子息には気まずい
昨今ですが、そんなとき物理学生におすすめのアルバイトは何?
一日拘束時間限度はどのくらいがいいですか?
575ご冗談でしょう?名無しさん:01/12/25 18:42 ID:D4fpV+3J
光の速さですでに消滅した星に向かうと爆発の
瞬間は、見れますか?それを見た瞬間に光速以上
で近づいたり離れたりしたら爆発した瞬間が
何度も見れますか?
576ご冗談です。:01/12/25 21:12 ID:???
>>574
コンビニ
577ご冗談でしょう?名無しさん:01/12/25 23:11 ID:98Bfc8x6
野球のピッチングで「伸びが有る球」ってのは物理的にどういう球なんですか?
回転の問題?
578ご冗談でしょう?名無しさん:01/12/25 23:27 ID:hRJPyPx/
球に体重をのせて投げるとバッターの手元でグっと加速します。
579ご冗談でしょう?名無しさん:01/12/25 23:31 ID:???
>>577
最近どこかのスレで書いた覚えがあるのでがんばって探してください。
580577:01/12/25 23:34 ID:98Bfc8x6
>>579
ほんとですか?
ここ来たの初めてなんですが、
それって高校生でも理解できるレベルの文章でしたか?

それにしても同じような疑問持つ人僕以外にも居るんですね。
581ご冗談でしょう?名無しさん:01/12/26 00:26 ID:i9k6lfFM
>>577
よく言われるのは、
普通の球→ホームベース付近で球速が落ちる
伸びる球→球速が落ちない
ということで、普通の球を見慣れた打者が、球速が落ちてないだけなのに球速が上がったように錯覚するというもの。
582ご冗談でしょう?名無しさん:01/12/26 02:46 ID:???
映画「スタートレック」等に出てくる物体転送装置は
物理的にOKなのですか?

ドキュソ的質問でゴメソ・・・
583ご冗談でしょう?名無しさん:01/12/26 07:59 ID:???
>>582
NG
584ご冗談でしょう?名無しさん:01/12/26 13:26 ID:VrNwsOsV
既出の質問多すぎ。
過去スレ
>>6
585575:01/12/26 15:09 ID:CnQVuiSj
教えて下さい。お願いします。
586ご冗談でしょう?名無しさん:01/12/26 15:20 ID:S49X+0pr
>>585=575
光の速さでは移動できませんし、光の速さ以上でも移動できません。
587577:01/12/26 22:18 ID:xgvhT8zR
>>581
なるほど。
では「ホームベース付近で球速が落ちる」と「球速が落ちない」の違いはどういうものなのでしょうか?
やっぱり回転の問題ですか?
588ご冗談でしょう?名無しさん:01/12/27 02:20 ID:???
>>587
該当スレを探してたら、その答えも一緒に見つかったはずだが。
589おてちん:01/12/27 09:51 ID:IWc5WvJA
天文板でも質問してきたのですが、未だ答えが出ていないので、
ここでも質問させてください。よろしくおねがいします。
生まれたばかりの若い銀河の写真を見ていると、二本の帯が
だんだん渦巻き状になっていくのが確認できますよね。
素人目で見ると惑星状星雲が砂時計状に出している二本のガ
スのジェットと、若い銀河の二本の帯が関係が有るように思
えてきます。だから惑星状星雲が白色矮星を中心に新しい小
さい銀河に成っていくのかなと考えてしまいます。
もしそうだとすると、太陽がこのまま白色矮星になるとする
なら、太陽系が新しい小さな銀河に生まれ変わり、この太陽
系が所属する銀河さえも元はとてつもなく大きな一つの星系
だったのかな?そしてその大きな星系さえも元は・・・
なんて考えていると、夜も眠れません。
誰か詳しい方がいましたら教えてください。お願いします。
590おてちん:01/12/27 09:56 ID:IWc5WvJA
お手数おかけします。あともうひとつ質問です。
原子崩壊とは、いったいどおいうことなんでしょうか?
自分はバカなので、解りやすく説明できる方よろしくお
ねがいします。 
591おてちん:01/12/27 10:30 ID:IWc5WvJA
>>589
自己レスです。大体解決できました。
590が未だ解りません。よろしくおねがいします。
あと宇宙の大きさは変化していると聞いた事があるのですが
それはほんとうでしょうか?
あと変化しているとすると、その変化によって物理常識や
ガスが恒星になるための臨界量などが変わってしまう事が
あるんでしょうか?
よろしくおねがいします。
592  :01/12/27 11:18 ID:???
>591
 原子核を区別するものは陽子の数と中性子の数である。この数の
組み合わせによって、原子核が安定か不安定化決まる。不安定なもの
は、原子核崩壊を起こす。
 宇宙の大きさは変化している。そして、何度か相転移を起こしている。
593ご冗談でしょう?名無しさん:01/12/27 18:15 ID:3CPjXfXm
一回転だけ半径rで巻いたコイルに電流iを流したとき、
コイルを貫く磁束は全部でいくつになりますか?透磁率はμです。
594ご冗談でしょう?名無しさん:01/12/27 19:04 ID:P4wYajbE
>>593
積分でシコシコ計算すれ!
595593:01/12/27 19:16 ID:???
>>594
分かりました。頑張ってシコってみます。
596ご冗談でしょう?名無しさん:01/12/27 19:53 ID:oVYac/jl
光は水に入ると周期が変わるけど何で色が変わんないんだっけ?
忘れちゃった。おせーて?
597ご冗談でしょう?名無しさん:01/12/27 19:57 ID:???
>>596
周期(振動数)は変わらない。だから色も変わらない
598ご冗談でしょう?名無しさん:01/12/27 20:03 ID:???
>>597
そうだった
思い出したサンキュ
599名無し:01/12/27 20:07 ID:wG39UTAV
確かに振動数は変わらないかも。
でも、色は変わるでしょ。
海って青くねぇ?
ま、あれは振動数が変わるからじゃないんだけどね。
どんなチープな雑学本にも載ってると思うから、見てみ。
600思い出の名無しさん:01/12/27 20:14 ID:???
>>599
海が青いのは俺が小さいころ絵の具を水道で流しちゃったからなんだ
海と空を書くのが大好きだったから青色をいっぱい使ったんだ
え?環境破壊だって?
子供に罪はないんだよ・・・
601ご冗談でしょう?名無しさん:01/12/27 20:47 ID:???
夕暮れの海や湖は夕焼け色してるね
緑色の光で海を照らせば緑色になると思うよん
602場科学性:01/12/27 21:18 ID:Iq+0vsbF
振動と波のところって難しいですかね?テスト前だけどぜんぜん勉強してなかったんですよ
603ご冗談でしょう?名無しさん:01/12/27 21:45 ID:???
>>601
風呂にたまってる水なんかもちょっと青っぽく見えないかい?
604ご冗談でしょう?名無しさん:01/12/27 22:47 ID:YktfaMkJ
電子ニュートリノの質量が、
やっと理論的にはじき出せれました!
記念カキコ。
0.023 eV/c^2 もしくは 0.0075 eV/c^2
605ご冗談でしょう?名無しさん:01/12/27 22:51 ID:???
>604
 妄想か? 何で理論的に導かれた値が2つあって、桁が違うんだ?
606604:01/12/27 23:19 ID:???
>>605
妄想じゃないけど、理論が不完全だから、
どちらの状態をとっているのかまだ判断できないんです・。。
607ご冗談でしょう?名無しさん:01/12/28 00:35 ID:bULkxxZj
レーザー光線を丁度レーザーがぎりぎりナイフの歯に当たるように
当てたものを壁に投影してみたら(以下図)

====レーザー====>> ─(ナイフ) ====<< |(投影した光)

のようにナイフの向きと90度違ったものになりました。
つまりナイフを水平に置いたら、壁の光の様子が垂直向きだったんですが
どうしてでしょうか?
608ご冗談でしょう?名無しさん:01/12/28 00:48 ID:???
>>607
すまん
さっぱりわからない。
609ご冗談でしょう?名無しさん:01/12/28 02:33 ID:bULkxxZj
>>608
っていうことは結果が間違っていたんですかね。。。
他にも、一本のスリットを通した場合の像の見え方と

レーザーの通り道に髪の毛を横切らせた場合の像の
見え方で、前者は細い部分だけを通り、後者は細い部分
だけが遮られているのに、像自体はなんで似るんだろう?
とか、これから髪の毛の太さが求められるんだよ、という
話でしたが、よく分からないのです。
大学の実験とかだとこんなのはやらないですか?
610名古屋:01/12/28 02:43 ID:jXm+KB0v
607>
偏向ふぃるたーの原理と同じじゃねーの?
レーざーって電磁波なんだろ?
611ご冗談でしょう?名無しさん:01/12/28 02:43 ID:???
うみがあおいのはみずがほかのいろをきゅうしゅうしてしまうからでしょ?
>601
そういえばもりにかこまれたみずうみはみどりいろがおおいね(W

>607
なにをいってるのかわからないんだよ。
612ご冗談でしょう?名無しさん:01/12/28 04:06 ID:LpkbzD0Q
http://www.newtonphysics.on.ca/index.html
このPaul Marmetってまともな学者ですか?
613ご冗談でしょう?名無しさん:01/12/28 11:36 ID:???
age
614ご冗談でしょう?名無しさん:01/12/28 17:00 ID:???
>>611
 >うみがあおいのはみずがほかのいろをきゅうしゅうしてしまうからでしょ?

その説が正しいと仮定すると、海の植物は、青色を吸収する葉緑素を持っている
はずだよね。しかし、そうじゃないでしょ。
615ご冗談でしょう?名無しさん:01/12/28 20:18 ID:???
>614
いっていることがわからないよ。
あおいろをきゅうしゅうしない葉緑素というものがあるの?
ここはなっとくできないよ。
ひかりのなかであおいろがいちばんみずのなかをすすめるときいていたんだけど、ちがったのかな?
616ご冗談でしょう?名無しさん:01/12/28 20:32 ID:???
>>615
 これは、これは、馬鹿君ですか?
葉緑素が何種類あるか知ってる?

>ひかりのなかであおいろがいちばんみずのなかをすすめるときいていたんだけど、ちがったのかな?
 なんでよ。 論理的に説明できるか?

無能者が! カッコ付けてんじゃねぇ!
617ご冗談でしょう?名無しさん:01/12/28 20:50 ID:???
青色は水の中で散乱されて、その散乱光が目に入ってくるんだよな?
618ご冗談でしょう?名無しさん:01/12/28 20:51 ID:???
あ、それは空の場合か。
619617:01/12/28 21:01 ID:???
>>616
つーことでちょっと検索してみたけど、どうやら615のほうが正しいね。
水が赤〜黄の光を吸収し、波長の短い青い光を散乱するから青く見えるらしいよ。
http://www.google.com/search?num=100&hl=ja&q=%90%85+%8EU%97%90+%90%C2&btnG=Google+%8C%9F%8D%F5&lr=lang_ja
620 :01/12/28 21:05 ID:???
>>619
 おいおい、青色を散乱しているのに水の中を進めるとはどういうことだ!
621ご冗談でしょう?名無しさん:01/12/28 21:06 ID:???
>>620
散乱と吸収は違うでしょ…
622 :01/12/28 21:09 ID:???
 馬鹿なことを要っているじゃねぇ。吸収なんか一言も一とらんぞ。
すすまないと要っているのだ。
623ご冗談でしょう?名無しさん:01/12/28 21:31 ID:???
そもそも散乱されなきゃ見えないでしょうが
624 :01/12/28 21:38 ID:???
>>623
 だれも、それを否定してないでしょ!
シッタゲに発言してるの?
625ご冗談でしょう?名無しさん:01/12/28 22:02 ID:???
・・・結局何が言いたいの?
626ご冗談でしょう?名無しさん:01/12/28 22:30 ID:???
産卵と急襲?
海ガメを守れ!
627ご冗談でしょう?名無しさん:01/12/29 00:05 ID:nJuqWIaL
なんでドンペリの泡は一筋になって上にあがるの?
ほかのシャンパンとはなんで違うの?
628ご冗談でしょう?名無しさん:01/12/29 00:06 ID:???
愛一筋だからさ、坊主。
629ご冗談でしょう?名無しさん:01/12/29 01:53 ID:owNp+ZMP
この板の何割くらいがトンデモですか?
630ご冗談でしょう?名無しさん:01/12/29 03:00 ID:???
>>629
君とあの人。
631ご冗談でしょう?名無しさん:01/12/29 03:06 ID:owNp+ZMP
自分とをっさんですか。
632ご冗談でしょう?名無しさん:01/12/29 17:33 ID:???
616にはせいしきにしゃざいをねがいたい。
しらべもしてないのにしったっかこいてんじゃないよ。

620
さんらんされててもほかのいれはきゅうしゅうされてしまうのだったら
あおいひかりがいちばんすすむでしょ?
さんらんはいっかいじゃないし、さんらんのほうこうはいっぽうだけじゃないのよ
633ご冗談でしょう?名無しさん:01/12/29 18:12 ID:???
>>632
まあまあ。ほら,冬休みだしさ。
634ご冗談でしょう?名無しさん:01/12/29 21:54 ID:cgnT0lcA
宿題でごめんなさい。
上端が固定されたバネの下端に質点を吊るして、質点をYsin(Ωt)で
上下に動かしたときの運動を解析せよ。という問題が出ました。
バネが安定してる状態から、Ysin(Ωt)で上下に変化するから
m(d^2x/dt^2)=mg-kYsin(Ωt)
と考えたのですが、なんかダメっぽい気がしています。考え方を教えて下さい。
635ご冗談でしょう?名無しさん:01/12/30 00:12 ID:jsV8gTAg
>>634
何がysin(ωt)なのかがわからないので正確な答えがいえないけど
多分質点の位置だろうって事で答えときます。というか答えるも
なにもその式のままでほぼ答えで計算する事は何もありません。
最大値(振幅)yをとる周期2π/ωのsinカーブを描いて動く、
というだけです。もしもう少し述べたいのなら、この質点の初期条件が
初期位置が重りをつるした状態でばねが安定した位置で
初速度がy/ωだという事を書いとけばいいと思います。
636ご冗談でしょう?名無しさん:01/12/30 01:24 ID:T7lxqnAV
>>634,(635)
これって、力がFsin(Ωt)という問題設定なんじゃありませんか?
637ご冗談でしょう?名無しさん:01/12/30 01:32 ID:hbZn2Q5s
そんな事より1よ、ちょいと関いてくれよ。スレとあんま聞係ないけどさ。
昨日、近所で牛鮭定食食ってたんです、俺。
そしたらなんか人がめちゃくちゃいっぱいでお薦め出来ないんですよ。
で、よく見たらなんか垂れ幕下がってて、諸刃の剣、とか書いてあるんです。
もうね、アホかと。これ最強。
お前らな、150円引き如きで普段来てない隣の奴にはねぎが多めに入って
る。そん代わりつゆだくが少なめ。これ。
150円だよ、これが通の頼み方。
なんか親子連れとかもいるし。一家150人で吉野家にマークされる。これだね。
よーしパパ特盛頼んじゃうぞー、ボケが。
お前らな、つゆだくやるからその席空けろと。
吉野家ってのはな、もっと何が、やっぱり、だ。
Uの字テーブルの向かいに座った奴といつつゆだくが始まってもおかしくない。
刺すか刺されるか、そんなのすっこんでろ。
で、やっと座れたかと思ったら、肉がぶち切れですよ。
そこでまた、喧嘩大盛で、とか言ってるんです。
あのな、つゆだくは雰囲気がいいんじゃねーか。最新流行はきょうび流行んねーんだよ。
得意げな顔して、問いたい。問い詰めたい。小1時間問い詰めたい。
お前は本当に殺伐としてるべきなんだよ。女子供を食いたいのかと。
お前、つゆだくが4円って言いたいだけちゃうんかとか言ってるの。
吉野家通の吉野家から言わせてもらえば、今、吉野家の間での吉野家通はもう見てらんない。
ねぎだく、おめでてーな。
大盛りねぎだくギョク。150円。
ねぎだくって来てんじゃねーよ、ボケが。
で、それにギョク(馬鹿大盛り)。玉子かと。
しかしこれを頼むと次から店員150円引きという危険も伴う。
素人は座れないんです。
まあお前らド素人は、吉野家でも行ってなさいってこった
638ご冗談でしょう?名無しさん:01/12/30 01:37 ID:R9RxQS0X
>>635,636
Ysin(Ωt)は、質点の位置です。
…それだけなのでしょうか。
「Ωがω=√(k/m)…k=バネ定数、m=質量
 の場合とそれ以外の場合に分けて考えろ、」
と言われたような気がするので悩んでいたのですが…。
639635:01/12/30 13:23 ID:jsV8gTAg
>>638
ysin(ωt)が質点の位置ならそれだけで終了。

>「Ωがω=√(k/m)…k=バネ定数、m=質量
 の場合とそれ以外の場合に分けて考えろ、」

なんか変な事言う先生だな…。分けて考える事は無理です。
というかそれ以外の場合は存在しないと言ったほうがいいかも。
もしあるとすればそれは質量やばね定数が時間と共に変化するという
空想上でしかありえないタイプの問題になります。そうだとしても
ただ単に周期(sinカーブの幅)や振幅が変化しながらおかしな形の周期
を描くだけです。この変化は質量やばね定数の時間変化の式が与えられていないと
求める事は出来ないので、多分この問題ではそこまで求めさせないはず。
640634:01/12/30 14:27 ID:???
>>639
なんか、共振とかそういうことを言われたような覚えが
あるのですが、ひょっとすると覚え違いかもしれません。
特に気にしなくてもいいんでしょうか?
641ご冗談でしょう?名無しさん:01/12/30 16:44 ID:???
QEDでもQCDでもいいんだけど、
非摂動効果を解析的に扱おうと試みてる文献なにか知りませんか?。
ふぇいず・とらんじっしょんとの絡みで。
完全に解析的じゃなくてもいいです。
漸近展開とか。とにかく結合定数以外の展開。
642高卒ですが・・:01/12/30 17:57 ID:j4OxOp9w
物理に詳しい大学生の方へ
時間について教えてください。
時間とは学者が物事の現象を説明するために
都合が良い為に考え出した実態の無い単位?なんですよね?
つまり、どのような状況でも時は正確に刻みつづける(その方が都合がいい)
だから、相対性なんとかで、遅くなったり進んだり(矛盾)するんじゃないの?
つまり時間の概念に無理が生じた?
じゃないんですか?
643636:01/12/30 18:44 ID:XeB0HD4T
>>640
共振がどうとかいうのは、典型的には力が正弦的に変化する場合の話題です。
だから、問題文は正しくは
>質点をYsin(Ωt)「の力」で上下に動かしたときの運動を解析せよ。
だと思う。で、Yが力を表すのは気持ち悪いので、たぶんFの見間違えじゃないかと想像した次第。
644ご冗談でしょう?名無しさん:01/12/30 18:57 ID:XeB0HD4T
>>642
だいたいそういうことでいいと思います。

「都合がよいように時間を考え出した学者」というのは一般にはニュートンであると捉えられます。
しかし、ニュートンの考え出した“絶対時間”と呼ばれる時間の概念による解釈では、色々な観測結果との
矛盾が生じたため、アインシュタインによって新しい時間の概念が考え出されました。

アインシュタインの理論では、動いている時計の進み方が遅くなったりするなど、一見矛盾が生じます。
これを矛盾と思うのは、“絶対時間”という古い時間の概念の方が私たちの直感とよく合っているからです。
645ご冗談でしょう?名無しさん:01/12/30 19:07 ID:cNDXSb0y
水から空気に入るときの
屈折率はいくつでしたっけ?
646工房:01/12/31 03:02 ID:Xcto2XDG
光より早く情報を伝えるモノは無いと聞きます.
でも,剛体を押したり引いたりしたら,一瞬にして情報が伝わるのではないでしょうか?
647ご冗談でしょう?名無しさん:01/12/31 03:09 ID:0YkDyvZ0
>>646
その矛盾によって、剛体の存在が否定されます。
648ご冗談でしょう?名無しさん:01/12/31 06:19 ID:???
ふと思い出した疑問なんだけど
ある超高層ビルの屋上にボールを置く。
そのままボールをビルから落とすと影は地面に移る。
この影の移動速度は高速を越えている!って何年か前に言われたことあるんですが
本当なんでしょうか? この板見ていたら思い出した(w
649ご冗談でしょう?名無しさん:01/12/31 08:12 ID:???
>>648
影の移動速度は光速を超えることもありますので本当です。
地球は1日で1回転します。地球からはるか遠くに見える星も
地球基準で考えれば光速を超えた速さで動いています。
とくに問題はありません。
650ご冗談でしょう?名無しさん:01/12/31 09:35 ID:???
>>649
 おかしなこと言うな〜
651ご冗談でしょう?名無しさん:01/12/31 09:44 ID:???
ここには、何人かコテハンがいるけど、それぞれの評価はどうなの?
652648:01/12/31 15:46 ID:???
>>649
ふむふむ、豪語されただけあって本当だったかw
ちなみに高速× 光速○ ですね^^;
しかし影が光速を越えるのなら、それって相対性理論の足かせにはならないのかな?
653ご冗談でしょう?名無しさん:01/12/31 16:26 ID:j75yJxe8
>>649
どうもそれは変な気がするんですけど。影が出来るのは光をさえぎっているから
なんだから、どう考えても、影の変化の速度=光速、これ以上にも以下にも
ならない気がします。単なる自分の勉強不足かも知れないんで
出来れば証明か、もしくは参考文献なんかあったら教えて下さい。
654593:01/12/31 16:38 ID:aOteggrn
やっぱり分かりません。
655ご冗談でしょう?名無しさん:01/12/31 16:46 ID:???
>>593
 どの様に考えたか書いて
656593:01/12/31 17:11 ID:???
原点を中心に半径rの円を考えて、その円に電流iが走っていると考えて
rより小さい距離の点の磁束密度を求めて面積分して行こうと思ったのですが、
馬鹿過ぎるので最初の磁束密度さえ分かりません。
原点の磁束密度さえもなんかおかしいです。もう駄目です。
657ご冗談でしょう?名無しさん:01/12/31 17:18 ID:???
>>656
 高校生でしょうか、大学生でしょうか?
大学生としましょう。

 電磁気学の問題は全て、マックスウェルの方程式から解くことが
出来る。
  div D = ρ
  div B = 0
rot H = j
rot E = -dB/dt
ここまで、分かる?
658    :01/12/31 17:24 ID:G7Rc6KLd
基本的な質問です。
磁石のN極とS極が引っ張り合うのは、磁力があるからだと思いますが、
その磁力は何故あるのでしょうか?
659593:01/12/31 17:25 ID:???
分かりません。数学の記号は分かりますが。
ρは電荷密度ですか?
660593:01/12/31 17:52 ID:???
あー、ありました。マックスウェルの方程式ですか。
もっと電磁気を勉強してきます。
コイルがちょっと気になっただけなんですが。
661 :01/12/31 18:18 ID:zdnMS7dk
アインシュタインって人間的にどのくらいハズレてましたか?
挨拶とかしたこともない系?
662名古屋:01/12/31 18:44 ID:2gVYQJk8
659>簡単な質問やから答えるw
電束密度だっけ?電荷密度だっけw
ようは、単位面積あたり 入ってくる量と出て行く量が一定なんだっけ???
最近 みそかつ食いすぎて胃がおもい
663名古屋:01/12/31 18:55 ID:iPpHUIdw
あああみす
単位体積あたりの電荷密度ね
664657:01/12/31 19:31 ID:???
>>656
 ちょっと食事に言ってました。
あれ、もう分かっちゃたんでしょうか?
665657:01/12/31 20:10 ID:???
>>656
返事ありませんね。マックスウェルの方程式のうち
 rot H = jを使えは出ます。
終わり。
666ご冗談でしょう?名無しさん:01/12/31 20:32 ID:???
>>658
 それを完全に理解するためには、理論系で大学院へ行かなければならない。
まぁ、独学でも理解できるが。
667名古屋:01/12/31 21:59 ID:W1otcotN
磁性はむずい
668ご冗談でしょう?名無しさん:02/01/01 02:17 ID:2JQuKph1
質問なのですが
TV番組である家の中から生中継していて、その家の中のテレビも
生中継の番組にあわせていました。
私の自宅のテレビに映っているテレビ(中継場所に置いてある
テレビ)の映像はあわせ鏡の様にテレビの中にテレビが次々と
映っている状態が映ってました。
よく見ると自宅のテレビに映っているテレビは自宅のテレビよ
り少し遅れて映ってた様な気がしたのですが、自宅のテレビと
自宅のテレビに映っているテレビは時間軸的には、ほぼ同じだと
思うのですがなぜ遅れた映像が映るのでしょうか?
説明が分かり辛いかもしれませんがすみません。
また勘違いでしたらすみません。
よろしくお願いします
669ご冗談でしょう?名無しさん:02/01/01 02:37 ID:aN/yVh3Q
映像情報が伝送されて表示されるのに時間がかかるから。
670ご冗談でしょう?名無しさん:02/01/01 02:40 ID:2JQuKph1
>>669
その中継場所のお隣の家のテレビで見れば遅れないということなの
でしょうか?
671ご冗談でしょう?名無しさん:02/01/01 02:47 ID:2JQuKph1
>>669
あと書き忘れていたのですが、テレビを見ていたのも東京23区
内で中継がされていたのも23区内だったのですが、その位の
距離でもやはり伝送速度(光の速度)の誤差は目で見てわかるの
でしょうか?
質問ばかりですみません。
672ご冗談でしょう?名無しさん:02/01/01 03:01 ID:aN/yVh3Q
ダイレクトな通信ならともかく一般向けの
テレビ放送なら間にいろいろな設備機器通すでしょう。
その家のテレビは、中継カメラから直接データをもらってるわけじゃないから。
673ご冗談でしょう?名無しさん:02/01/01 03:35 ID:Kt7ei/px
>>672
自宅のテレビと自宅テレビに映ってるテレビも中継カメラから
直接データをもらってないので映る映像は同じでもいいと思う
のですがどうでしょうか?
674ご冗談でしょう?名無しさん:02/01/01 03:37 ID:aN/yVh3Q
>>673
テレビの中に映ってるテレビに流れる映像は、
倍、余分な経路を通ってきたことになるんじゃないかい
675ご冗談でしょう?名無しさん:02/01/01 03:47 ID:Kt7ei/px
>>673
中継している家のテレビの中に映ってる映像ならそう思うのですが
たとえば私の自宅で7:00にテレビを見たら、
自宅のテレビに映ってるテレビの映像も同じ7:00の映像じゃ
ないんですか?
 物分り悪くてすみません・・・
676    :02/01/01 05:55 ID:Fpkui1iZ
>>673
遥か上空の衛星を往復しているので、少しは遅れるかも。
よく衛星生中継の時に、音声が遅れるでしょ。
それにしても、あなたの文章は読みにくい・・・
677ご冗談でしょう?名無しさん:02/01/01 20:02 ID:locgScTb
観測不可能な領域は宇宙に含まれますか?
678ご冗談でしょう?名無しさん:02/01/01 20:15 ID:???
観測不能というなら今現在の目の前にある空間の凄く凄ーくミクロの
空間も観測不能なのではないでしょうか?
679ご冗談でしょう?名無しさん:02/01/01 20:17 ID:???
>>678
観測不可能と因果が無いのは違う
680ご冗談でしょう?名無しさん:02/01/01 20:20 ID:???
観測の定義とは?
681ご冗談でしょう?名無しさん:02/01/01 23:04 ID:???
重力ってなんであるの?
分からないとパパがお年玉くれないって言うんだもん。
682ご冗談でしょう?名無しさん:02/01/02 08:55 ID:???
d'Alembertian って何て読むの? t 読むの?
だらんべりあん? だらんばーちゃん?
683ご冗談でしょう?名無しさん:02/01/02 15:03 ID:hhEoTw5o
エネルギーって何でしょうか?
684ご冗談でしょう?名無しさん:02/01/02 15:05 ID:hhEoTw5o
逆立ちをすると、地球を持ち上げたことになるのでしょうか?
685ご冗談でしょう?名無しさん:02/01/02 15:29 ID:???
>>684
もしそうなら私たちは常に足の裏で地球を持ち上げてることにならんかね?
686ご冗談でしょう?名無しさん:02/01/03 14:32 ID:KcA8esDM
フレミングの法則ってなんですか?
687ご冗談でしょう?名無しさん:02/01/03 14:37 ID:KcA8esDM
いや、違う、フレミングの法則で大きな物が動かせますか?
688ご冗談でしょう?名無しさん:02/01/03 18:47 ID:qLU7e5kR
質問です
半径rの球体の内部に一様に電荷が分布していて、球体の電荷の合計がQとする
そのときの静電エネルギーを求めよ。
で順番に手順を追って説明してもらえないでしょうか?
式と考え方。
初学者ということでおねがいします。
定義から求めてもらえませんか?手間がかかるかもしれないけれど
689 :02/01/03 19:15 ID:/5kMZy8K
どんな基準でIDが付く板と付かない板で分かれているのかがわかりません
690理学部:02/01/03 19:36 ID:ZGjENyNm
>>682
ダランベルシアン。記号は「□」っす。
691688:02/01/04 02:33 ID:hdtITnCK
助けてください
むずいです。ちなみに導体球ではないです。
692ご冗談でしょう?名無しさん:02/01/04 06:18 ID:???
>>690
おお、打欄ベル思案って、犬じゃないのか!
それはダルメシアン。
693ご冗談でしょう?名無しさん:02/01/04 13:33 ID:JEEyeI66
真空中に完全導体があり、電磁波を入射したときに、
表面電流が流れますが、これと磁場により生ずる力を考えるのですが、
導体中では磁場0なので、全空間がその磁場であったときの
半分の力がかかると考えていいのでしょうか?
694682:02/01/04 15:34 ID:???
>>690
お返事さんきうそーまっち。
>>692
なんとなく反応さんくす。
695688:02/01/04 16:23 ID:XkUPrXbd
お返事御願いします
助けてください
696ご冗談でしょう?名無しさん:02/01/04 17:14 ID:CEQmAzGl
エントロピーについての厨房質問なんですが、
エントロピー増大の法則の導出の仕方って、
「分子1つ1つのミクロな条件はわからないので、系を区間ごとに分けて、
系のマクロな状態に基づいて区間ごとの分子の個数と速度を確率分布で
表現、すると、ニュートン力学の運動方程式を使っても、Δt経った後に
エントロピー(の定義は状態がもつ確率で定めるけどそれはおいといて)が
増大している確率が圧倒的に1に近い」
でいいんでしたっけ?
697教えてください:02/01/04 21:40 ID:cPbXUX75
彼女にするならどんなタイプの子がいいですか??

好きな人がいるのですが、どうやら頭のいい女性が好きみたいで。
やっぱり物理の話がわかるような人がいいのでしょうか??
698ご冗談でしょう?名無しさん:02/01/04 22:02 ID:???
どうして「物理の話がわかる=頭がよい」なのかわかりません。
どこからのその発想が生まれたのか教えてください。
頭のよさを示すための方法なんていくらでもあるでしょ?
さらに、あなたの専攻がなんなのかも697の文章からは
窺い知ることができません。

あなたの専攻を書いて、さらに物理の話がわかる=頭がよいという
発想がどこから生まれたのか示してください。
699ご冗談でしょう?名無しさん:02/01/04 22:43 ID:Jyc+gw5D
>>698
物理じゃなくても、何かの知識に特化してる人は頭(・∀・)イイ!と思うだろ。
何をそんなに熱くなっている?(w
700ご冗談でしょう?名無しさん:02/01/04 22:57 ID:???
>>689
>頭のよさを示すための方法なんていくらでもあるでしょ?
だから「物理の話が分かる=頭がよい」はいくらでもある中のひとつの方法だろ?
>>689に同意で
何をそんなに熱くなっている?(w
701ご冗談でしょう?名無しさん:02/01/04 22:59 ID:???

>>689じゃなくて>>699
702ご冗談でしょう?名無しさん:02/01/04 23:01 ID:???
>>700
すみません
さいしょのは>>689じゃなくて>>689
で後のが>>689じゃなくて>>699
703ご冗談でしょう?名無しさん:02/01/04 23:04 ID:Jyc+gw5D
>>702
なんかそれも間違ってるように見えるんだが、どうよ?
704ご冗談でしょう?名無しさん:02/01/04 23:06 ID:???
>>703
ホントだっ・・・・・すみません・・・・
でも言いたいことはわかるよネ?
705ご冗談でしょう?名無しさん:02/01/04 23:09 ID:Jyc+gw5D
分かる分かるオッケーよ、うん。
706132人目の素数さん:02/01/04 23:26 ID:???
ちょっとした疑問なんですがどうしてうんこは臭いのですか?教えてください
707ご冗談でしょう?名無しさん:02/01/04 23:29 ID:???
鼻があるから
708ご冗談でしょう?名無しさん:02/01/04 23:30 ID:Jyc+gw5D
>>707
感動した
709ご冗談でしょう?名無しさん:02/01/04 23:35 ID:???
>>708
すんな
710中卒塗装工:02/01/05 01:15 ID:???
>>668
衛星中継が間に入っていなければ、伝送時間は無視して良いでしょう。
伝送時間がゼロであっても、動画の1コマ(1フレーム)を表示するのに
1/60秒の時間がかかります。(物理の問題というより、NTSCの規格として。)
このため、テレビ画面をテレビカメラで撮影し、その映像を直接にそのテレビ画面で
表示させると、テレビの中のテレビ…は、各々1/60秒づつ遅れる事になります。
この遅れは十分に肉眼で確認できます。

#やってみると面白いよ。あと、テレビ画面とテレビカメラの角度を変えるとか、
#ネガポジ反転をかましてみるとか。
711誰かお願いします。 :02/01/05 01:34 ID:irmXsFbv
(問)
容量10立方メートル、質量2.0kgの気球がある。
下方は外気に通ずる。この中の空気を熱すると何℃でこれを浮揚させることができるか。
ただし、外気の圧力は1atm、温度は20℃、空気の密度は標準状態(0℃・1atm)
で1.29kg/立方メートルとする。

で、
ΔU(内部エネルギーの増加量)+pΔV(気体がする仕事)
                  =Q(外部から与えられる熱量)

の式や定積変化等を考えてみましたが式のたてかたが分かりません・・。
何かヒントを教えて頂ければ、幸いです。
厨房質問ですいません。
712高卒警備員:02/01/05 01:53 ID:???
>>688 方針だけ。
1. (a) div 電場 = 4π 電荷密度、 (b) 系の球対称性
(c) 積分公式 (∫div E 体積要素) = (∫ E・面積要素)
を使って電場分布を出す。
2. 静電エネルギ = (∫ |E|^2 体積要素)/2
から答えを求める。
713大卒ホームレス:02/01/05 02:20 ID:???
>>711
理想気体を仮定して PV = mRT を使う。定数Rは標準状態から求める。
温度Tに於ける気体の密度は ρ(T) = m/V = P/(RT)
Toutを外気の温度、Tinを気球内の温度、Vは気球の体積として
質量差 ΔM = V(ρ(Tout) - ρ(Tin)) が気球の質量2キログラムを越える温度 Tin
を求める。
714院卒テロリスト:02/01/05 03:10 ID:???
わかんねー
715688:02/01/05 07:01 ID:gfuzgasD
>712 thxそれは知ってます
でも出てこないんです。院生の方とかいないですか?
716688:02/01/05 07:08 ID:gfuzgasD
電場はラクに出せます。
そこからダメなんです。
717厨房マッサージ師:02/01/05 07:08 ID:xHv0dHdq
712の言ってることがわかるならできそうなものですが。。。
電場は導けたんですか?。

718厨房マッサージ師:02/01/05 07:10 ID:xHv0dHdq
それならあとは体積積分すればよいのでは?。
球面座標とってエネルギー密度を積分すれば、
すぐ答えがでると思いますが?。ダメなんですか?。
719688:02/01/05 07:12 ID:gfuzgasD
出ましたよ
ガウスの法則で
答えみても、あってます。電場は
そこから静電エネルギー出す計算がダメでした。tt
やり方がさっぱりです。答えしか書いてない問題集なので、、
720688:02/01/05 07:13 ID:gfuzgasD
えーと
極座標とってやるんですか?
721厨房マッサージ師:02/01/05 07:14 ID:xHv0dHdq
712に書いてある、

> 2. 静電エネルギ = (∫ |E|^2 体積要素)/2

ができないと言うことですか?。
どうやろうとしました?。
722688:02/01/05 07:16 ID:8GiwempT
そのまま電場Eをぶち込んで
球体の体積との積を取りました。
体積積分がまちがってるのかも、、(^-^;
723厨房マッサージ師:02/01/05 07:17 ID:xHv0dHdq
電場は球の中心からの距離rのみの関数ですよね?。
だから当然体積積分の体積要素は
球面極座標を取ったほうがラクです。
角度積分はできちゃいますから。
724厨房マッサージ師:02/01/05 07:20 ID:xHv0dHdq
球面極座標での体積要素は 4πr^2 dr です。
だから、

∫ E(r)^2 4πr^2 dr

の積分ができれば答えが出るはずです。
725688:02/01/05 07:21 ID:8GiwempT
です。
距離のみの関数です。
つまり球体の中心を原点にして
角度2つとって座標だして それぞれx,y,zを角度と距離の関数にして
電場の中のrのところに代入して、両方0から2π やるんですか?
726688:02/01/05 07:26 ID:8GiwempT

体積要素の意味がわからないですttすいません
727厨房マッサージ師:02/01/05 07:27 ID:xHv0dHdq
ん?。
申し訳ないけど、

> 電場の中のrのところに代入して、両方0から2π やるんですか?

の意味がちょっとワカラナイです。

電場はx,y,zで書く必要はありません。
書いてもいいけど積分はメンドーなことになります。
むしろ体積要素であるところの dxdydz を、
極座標に直すのです。
この変換はヤコビヤンやらで出すんですが、
とにかく結果は、

dxdydz = r^2 sinθ drdθdφ

となります。
728688:02/01/05 07:29 ID:8GiwempT
なるほど
dv=dxdydzですね?
729厨房マッサージ師:02/01/05 07:30 ID:xHv0dHdq
体積要素とはdxdydzのことです。

"静電エネルギー" = (∫ |E|^2 dxdydz )/2

↑これはオーケーですか?。
ε_0は省いてますが。
730厨房マッサージ師:02/01/05 07:31 ID:xHv0dHdq
>>728

yes! yes!
731688:02/01/05 07:32 ID:8GiwempT
729の式は、わかります。
732厨房マッサージ師:02/01/05 07:36 ID:xHv0dHdq
で、電場E(r)は角度θ、φには依存しないので、
729右辺の積分のθ、φに関する積分はできちゃいます。

∫sinθdθdφ = 4π

で、結局rについての積分だけが残ります。
それは簡単にできます。
733厨房マッサージ師:02/01/05 07:37 ID:xHv0dHdq
dV = dxdydz = r^2 sinθ drdθdφ

↑これは知ってますか?。
734688:02/01/05 07:40 ID:8GiwempT
なるほど
すると rの関数を積分するのですね?
0から球体の半径R?まで?

電磁気にお詳しいようなのでついでに質問します。
もし球体の表面だけに分布している場合。
中心までの電位とその球体の全電荷をかけて0.5倍すればいいのでしょうか?
735688:02/01/05 07:40 ID:8GiwempT
733については全く知りませんでした。。。
ベクトル解析でしょうか?3重積分?
736厨房マッサージ師:02/01/05 07:45 ID:xHv0dHdq
いま、ややこしいので導体の半径をaとします。
rに関する積分は0から無限大までです。
電場は r<a の場合と、r>a の場合で違う関数形ですよね?。
rが0からaまでと、aから無限大までの積分をそれぞれやります。
積分範囲は無限大までですが、収束するはずです。

申し訳ないですが、

> 中心までの電位とその球体の全電荷をかけて0.5倍すればいいのでしょうか?

の意味がわかりません。
「なにを」0.5倍するんですか?。なにを導く話しですか?。
737厨房マッサージ師:02/01/05 07:49 ID:xHv0dHdq
>>735
そうですか。それは電磁気なり力学なりの教科書の、
どこかに載ってるはずです。
同じ教科書を持ってればどこに載ってるか教えられるんですが、
たぶんそうもいかないでしょうし・・・。

とにかく探してみてください。
で、自分で一度証明してみることをお薦めします(僭越ながら)。
証明は覚えておく必要はないですが、
結果は覚えておいたほうがのちのち便利だと思います。
738688:02/01/05 07:50 ID:8GiwempT
ああ全空間に電場があるからですね、、
そうです、半径で変わります電場

ああ全く僕電磁気理解してないかも、、、
0,5倍のやつは、導体球のことについてです。
もしかしたら、導体球の場合は球内は電場が打ち消しあって0になるから
積分のrは無限大から半径まででいいのですか?
739688:02/01/05 07:57 ID:8GiwempT
ありがとうございました。
とても参考になりましたm(__)m
くだらない質問でごめんなさい(^-^;
うーん電磁気むずい、、
740厨房マッサージ師:02/01/05 07:59 ID:xHv0dHdq
> ああ全空間に電場があるからですね、、
そのとうりです。

> 0,5倍のやつは、導体球のことについてです。
失礼かもしれませんが、
なにを0.5倍してなにを出そうとしてるのかがワカラナイのです。

> もしかしたら、導体球の場合は球内は電場が打ち消しあって0になるから
> 積分のrは無限大から半径まででいいのですか?
そのとおりです。
導体球の場合の電場がどうなるか知ってますか?。
結果は、導体内では電場はゼロ、
導体外では一様な電荷分布の場合の球体外での電場
(つまりrの2乗分の1)と同じ電場となります。
導体内部の電場は0なんだから、
電場の2乗を、
おっしゃるように導体の半径aから無限大まで積分すればいいです。
それが静電エネルギーに比例します。
741厨房マッサージ師:02/01/05 08:01 ID:xHv0dHdq
くだらない質問ではないですよ。
一度は手を動かしてみないとわからないものだし
一度手を動かしておくとけっこうあとで思い出せます。
だんだん慣れてきます。
742ラリリーマン:02/01/05 16:46 ID:???
共鳴の出来るだけ一般的な定義を教えて下さい。
743688:02/01/05 19:09 ID:JJg99JBL
>厨房マッサージ師さん
どうも有り難うございました。
744ご冗談でしょう?名無しさん:02/01/05 23:33 ID:???
大学院で理論物理選考してる人って
普段どういうことしてるの?
実験とかしないでしょ。
745高一:02/01/06 02:12 ID:???
物理って面白いですか?
746REAL工房:02/01/06 11:12 ID:???
教えてください
地面の上に重さ10kgwの物体Aを置き、その上に重さ5kgwの物体を乗せた。
物体Aにはたらく力をすべてあげよ
物体Bにはたらく力をすべてあげよ
これらの中で作用・反作用の関係になっているのはどの組み合わせか?
つりあいの力になっているのはどの力とどの力か?

お願いします。
747びいかめ:02/01/06 14:20 ID:xyXGQZ3i
地球は太陽の回りを1年で一周すると言いますが、回転している宇宙船から見たらその年数は変わってしまいませんか。
さらに、その年数が変わると遠心力の大きさも変わるので、地球から太陽までの距離も変わりませんか。
748ご冗談でしょう?名無しさん:02/01/06 21:24 ID:mvgbKfTy
>>747
考え過ぎ
749ドテチン:02/01/06 23:41 ID:3UeSK5mR
すいません、引力って何ですか?全ての物質(電子や陽子等)に有るのでしょうか?
仕組みと言うか、概念が分かりません。
小さな頃から当たり前の事として教えられて来たのですが、不思議でたまりません。

寝る前に考えると理解出来なくて発狂しそうになります。
750ご冗談でしょう?名無しさん:02/01/07 08:41 ID:???
引力 物質と物質が近づこうとする力
斥力 物質と物質が遠ざかろうとする力

引力は、質量をもつ物質なら全てある。
751ご冗談でしょう?名無しさん:02/01/07 09:09 ID:lb3Em9Cl
一昨年くらいに入射角よりも屈折角が大きくなる媒質が発見された。
なるニュースを見たのですが、その詳細の載っているページご存知ありませんか?

確か、光ではなくて電磁波を、媒質は何かの合金だったと思うのですが...
このニュース自体が私の記憶違いでしょうか?
752本田尚哉サマ:02/01/07 09:17 ID:???
中大物理4年生の本田尚哉サマは絶対実際に天才だ!!!奴隷募集中
だそうだから会費1年100万円〒136−0076東京都江東区南砂2−30−1
マンション東陽町504に送って差し上げろ!!!詳しくは携帯09023277674
753ご冗談でしょう?名無しさん:02/01/07 09:51 ID:???
>>752
2chで電話番号っぽい文字列書いたらIPアドレス記録されて、
さらに度がすぎたら警察に通報されるって知ってた?
754 :02/01/07 10:20 ID:???
>>752
よし、コイツに被害届出したくなるくらいイタズラしてみよう。
後は任せておいてくれ752よ、心配すんな、お前には迷惑かからない様にしておくから。
755ご冗談でしょう?名無しさん:02/01/07 15:34 ID:LFVFX0MJ
752の書きこみを、みんなに知らしめる方法はないかな?
756ご冗談でしょう?名無しさん :02/01/07 16:05 ID:wJtk3K8R
ねぇ。
757ご冗談でしょう?名無しさん:02/01/07 16:41 ID:4tgPY+Y7
ここで訊いていいのかな?

新宿から富士山見えるのどうして?
つーか上空3千メートルの高さって絶対的にどんなものかわからない。

あと、上空100kmというと地球の輪郭や日本全土見えて
くる(のはそれぐらいだっけ?)が
横方向に100km移動してもそんなに動いたとも思えない。

人間の眼がそう見てるだけ?   なんかへんな質問ですが。
758ご冗談でしょう?名無しさん:02/01/07 20:21 ID:0r6i232t
質問です
3次元は、縦、横、高さ
4次元は、3次元+時間
じゃあ5次元は、4次元+何?
759ご冗談でしょう?名無しさん:02/01/07 20:25 ID:oG+wz1cO
質問です。
光源に向かって超高速で近付いたら観測者はx線を浴びますか?
760物理ど素人:02/01/07 21:04 ID:hdax5Lwx
***質問します***
粘土を細かくするために切る時の摩擦熱と、
凍らせた粘土を細かくするために切る時の摩擦熱は、
どちらが大きいでしょうか?
(粘土はプレートの上にあり、そのプレートは左右に往復します。
 切るための刃物は、同じ場所で上下に動きます。
 この左右、上下の動きで粘土はどんどん細かくなっていきます。)
761中卒塗装工:02/01/07 21:57 ID:???
>>759
可視光のエネルギを2eV、軟X線の端を2keVとすると、
可視光の光源と観測者との相対速度からの2γが1000を越えると観測者は
X線を浴びる事になる。
よって、相対速度が大体 v/c = 0.999998 を越えて向かい合えばよい。
762ご冗談でしょう?名無しさん:02/01/07 23:38 ID:LpKSRgEw
>>757(2)
人間の視野を160°、標高hm、みえる範囲をLmとすると、
tan(160°/2)=L/2h
L=2htan80°=11.342h
つまり人間は上にあがればその11倍もの長さを一望できるんだ。
763中学生:02/01/08 00:21 ID:pPikcqxQ
教えて君ですいませんが、是非教えてください。
数学が入るとどうも苦手です。
これはどう考えればいいのでしょうか?
数式と答え教えてください。

「太陽の放射エネルギー」
太陽から1天文単位の距離にある地球で1uの面積に一秒間
に当たる光のエネルギーの量は1,37×10㎥(J)である。
では、太陽が毎秒宇宙空間に放出している全エネルギーは
いくつか?半径rの球の表面積は4πr二乗である。
764ご冗談でしょう?名無しさん:02/01/08 00:24 ID:Tg+RhbcD
教えてくださったら幸いです。
一月ほど前、大学生活版で下記のようなスレを見ていたのですが、そこで、
なんでテコの原理を利用すると、少ない力で物が持ちあがったりするのか
と言う議論がありました。
しかしどれもまともなものではなくいまだに僕も分りません。
教えてください、お願いします。
http://school.2ch.net/test/read.cgi/campus/1008726120/l50
765ご冗談でしょう?名無しさん:02/01/08 00:30 ID:Tg+RhbcD
テコの原理なんて説明できんよ。経験的なもんだから。
万有引力の現象を数式を用いて記述することはできても、引力そのもの
について説明する事は難しい。
766ご冗談でしょう?名無しさん:02/01/08 00:36 ID:???
仕事=力×距離
距離を大きくしたら力少なくて済みます
767ご冗談でしょう?名無しさん:02/01/08 01:20 ID:???
>>765
おいおい…766見てちゃんと考えてみろよ
高校物理の内容だぞ。
768ご冗談でしょう?名無しさん:02/01/08 01:44 ID:P0+FEpAQ
すみません、宮城大の方いますか?
理論物理学の佐治晴夫先生をご存じでしょうか。
今、読売新聞日曜版で連載をなさっているのですが、なんだか言っていることが、
おかしいのです。
面白い、という意味ではなくて、ハァ?というかんじで……。

お人柄の良さは感じられる文章なのですが、学者として
物理学を(というか理系全般を)どう把握しているのか、
ものすごく疑問なのです……。
769ご冗談でしょう?名無しさん:02/01/08 03:04 ID:???
>>758
なんでもいいよ。じゃあ、なんかおもしろいものを考えてよ。
>>763
太陽が発生してる熱を半径1天文単位の球の表面積のみんなが分け合ってるイメージでいいよ。
分け合う、これ、割り算。難しいことではない
770ドテチン:02/01/08 06:31 ID:???
すいません749のドテチンです。

引力=質量という考え方で良いのでしょうか?
そしてE=mc2 の理屈からすると引力とは、
物質が持っているエネルギーを定義する一つである、と。

それと、たとえば真空な空間であれば何億光年離れていても、
限り無く0には近いけど引力は作用するんですよね?

そして引力が変化した時の変化分はどれ位のスピードで伝わるのでしょうか?
そういえば磁気の変化も物質の無い空間でも伝わるんですよね?
何故だろう、不思議です、眠れなくなります。

高卒のドキュソなので難しい定義なども理解出来ないのですが、何故か気になってしまいます。
771 :02/01/08 08:51 ID:+8P4kNA+
>>770
引力(質量)が変化したときにそれが伝わるスピードは
光速=スピードで確かいいと思った。引力に限って言うなら
それは重力波。それと質量があると言う事は、空間が歪んでいる
と考えるほうがいいYO-。
772ご冗談でしょう?名無しさん:02/01/08 09:23 ID:xW4TQHIP
>>770
私はポテンシャルというでっかいクッションに星がボンボンと置いてあるイメージです。
星があるとクッションが沈み込んで傾斜ができて他の星が★ ☆))...ツツ-と。
773ご冗談でしょう?名無しさん:02/01/08 13:11 ID:???
a
774ご冗談でしょう?名無しさん:02/01/08 13:28 ID:???
>>770-771
引力って言葉は引っ張り合う力全般に使われるので、
重力は重力または万有引力と呼びましょう。
775ご冗談でしょう?名無しさん:02/01/08 15:21 ID:???
宇宙の物質の99.9%はプラズマの形態で存在する。
このような宇宙では、重力よりも電磁力のほうがはるかに大きな役割を演じる。
従って、宇宙のしくみは電磁力を中心に解明されなくてはならず、重力を中心にした相対論的宇宙観は間違っているのだ。
776ご冗談でしょう?名無しさん:02/01/08 15:25 ID:???
>>775
 正電荷と負電荷の割合は?
そしてそれらの偏りは?
777ご冗談でしょう?名無しさん:02/01/08 15:27 ID:???
>>776
電荷の遮蔽効果も考慮できないキチガイのことはほっといてやれ
778ご冗談でしょう?名無しさん:02/01/08 19:21 ID:70nMpTMS
量子力学で出てくるコヒーレント状態ってなんなの?
コヒーレントだと、どーだというのでしょうか?
いくつか教科書見てみたけど、どーもわからんのよね。
779中卒塗装工:02/01/08 19:37 ID:???
>>778
コヒーレント状態は消滅演算子の固有状態です。
で、物理的には古典的な運動に最も近いものといった感じでしょうか。

古典的な調和振動子の場合には、ある振幅で振動子は正弦運動をするわけですが、
量子力学の調和振動子のコヒーレント状態では、波動関数のウエーブパケットが
最小不確定を保ったまま正弦運動しているように見えます。

まあ、少し大きな固有値のコヒーレント状態の波動関数をx表示なりp表示で
追ってみると分かると思う。
780ご冗談でしょう?名無しさん:02/01/08 20:36 ID:???
>>779
わけわかんねー
781教えて下さい。:02/01/08 21:28 ID:???
コリオリの力の導き方とかが詳しく載っている
ページを教えて下さい。
自分で色々と検索したのですが、
なかなか見つかりません。
782中卒塗装工:02/01/08 22:55 ID:???
>>781
自由粒子のラクランジアン L = (m/2)(速度)^2 を角速度ωで回転する座標系で表せば、
コリオリ力はωに比例する項として、遠心力はω^2に比例する項として自動的に出てくるよ。

#ラクランジュ形式を知らないと面倒かもね…
783ご冗談でしょう?名無しさん:02/01/08 23:17 ID:70nMpTMS
>>779
Thanks.追ってみます。
784理学部:02/01/09 00:49 ID:iLzuL4id
 3次関数に乗ることまでは分かっているデータがあります。
 で、WindowsとExcelがあれば、データ点打ち込んで
「近似曲線の追加」→「多項式近似・3次」
 として得られるのですが、ちと今、研究室にWindowsマシソが無いんです。

 で、PC-UNIXで手軽に近似曲線が得られるソフトってありますか? Xアプリでも
コンソールアプリでも、Gnome限定とかでもPC-UNIXで動けば構いません。
785ご冗談でしょう?名無しさん:02/01/09 01:07 ID:???
>>781
F=maを回転座標系で書き直せば自動的に出て来る
>>782
別にラグランジュ形式でなくてもいいでそ
786レポ:02/01/09 01:09 ID:Pf1vgDuO
地球上に存在する元素は100種類程度で、もっと巨大な原子番号の元素や、
巨大な同位体が存在しないのはなぜですか。
核力・クローン力・原子核の崩壊の観点から、だれか教えてください。
すみません。今日提出なのにわからないんです。
787中卒塗装工:02/01/09 01:11 ID:???
>>784
状況は良く分からないけど、MathematicaのInterpolation関数なぞはどう?
Interpolation[{{x1,y1}, {x2,y2}, ....}, InterpolationOrder -> 3 ]
で3次関数でフィットした関数を返してくれます。
788理学部:02/01/09 01:35 ID:iLzuL4id
>>787
あふぇ、すみません、フリーソフトでありませんかね?
789中卒塗装工:02/01/09 01:45 ID:???
>>788
gnuplotのfitなんかどう? 使った事ないけど…
790中卒塗装工:02/01/09 01:47 ID:???
gnuplotのfitのマニュアル http://www.linux.or.jp/JF/JFdocs/gnuplot-12.html
791Lain:02/01/09 03:29 ID:???
しつもーん!
100万ボルトの電圧って簡単に作れる?
15万ボルトくらいならスタンガンとかであるけど・・・
プラズマ状態にしたいんです。
792ドテチン:02/01/09 03:54 ID:0lwocTqt
770のドテチンでございます。
私の初歩的な質問に答えてくださった方々、有難うございました、少しスッキリしました。
空間が歪むっての解んなかったのでググってみました、私の中での空間の定義が歪んでしまいました。
空間って何なんでしょう、重力とは、電荷とは、。
少し理解出来ると又、疑問が増えてしまいます。
私には理解出来そうにないので、取り敢えず、そうゆう物だと自分に言い聞かせて、寝る事にします。オヤスミナサイ
793ご冗談でしょう?名無しさん:02/01/09 03:55 ID:???
>>791
バンデグラフだと800万Vくらい行きますね
794ご冗談でしょう?名無しさん:02/01/09 07:34 ID:???
>>793
「簡単に」
795中卒塗装工:02/01/09 08:41 ID:???
>>794
用意するもの
1.出力電圧100kVのスタンガン 1台
2.耐圧200kV以上の整流用ダーオード 1本
3.耐圧100kV以上のコンデンサ 10個
全部、アキバなり日本橋で手に入ると思う。

コンデンサを個別に100kVに充電する。具体的には、スタンガン、ダーオード、
コンデンサ1個を直列に継いでコンデンサを充電する。
スタンガン出力に余力があれば、コンデンサ10個を並列にして、一挙に充電する事も出来る。

その上で、全てのコンデンサを直列に接続すると、両端には1MVの電圧が現れる。

ダイオードが多数入手出来るなら、スタンガンを電源とし、
コンデンサとダイオードを組み合わせた梯子昇圧回路を構成るという手もある。

もっと簡単な方法があれば誰か教えて!

でも、スタンガンの放電でプラズマ状態は出来てるんだけど…
796ご冗談でしょう?名無しさん:02/01/09 10:21 ID:xoPBdIgS
あほな疑問です。
地球大の球形の構造物があるとします。
その構造は、外殻だけでできていることにします。
いわゆる卵の殻のような構造です。
とりあえず、総重量は地球と同じとします。

さて、私は、この外側に立つことはできるのでしょうか。
内側に入ったら、真ん中に落ちていくのでしょうか。
797坂田は ◆AHOzZjUk :02/01/09 10:25 ID:dO98F3lj
>>1は♥も出せないダメな奴。
798ご冗談でしょう?名無しさん:02/01/09 10:58 ID:M+qgvuRz
>この外側に立つことはできるのでしょうか。

できます。

>内側に入ったら、真ん中に落ちていくのでしょうか。

落ちません
799ご冗談でしょう?名無しさん:02/01/09 11:02 ID:???
真中に落ちるんじゃネーノ?
800ご冗談でしょう?名無しさん:02/01/09 11:28 ID:???
じゃネーノ。
801ご冗談でしょう?名無しさん:02/01/09 11:32 ID:???
そっか無重力か
802ご冗談でしょう?名無しさん:02/01/09 14:35 ID:???
>この外側に立つことはできるのでしょうか。

死亡

>内側に入ったら、真ん中に落ちていくのでしょうか。

死亡
803ご冗談でしょう?名無しさん:02/01/09 15:22 ID:32MS9SXx
>>796に便乗
殻を閉じた自由曲線に拡張しても中では無重力が成り立ちますか?
804質問:02/01/09 15:26 ID:esibE9o6
物理板の一番上にある画像に載ってる人って物理学的にどの程度の人なんですか?
それとも有名物理学者のそっくりさんなんですか?
それともただのどこかの人ですか?
805796:02/01/09 16:24 ID:xoPBdIgS
>>803
ところで、正解はどうなんでしょう。

球体の中は、無重力ってことですか????
806 :02/01/09 16:34 ID:???
>>805
 自分で計算してみ。
807ご冗談でしょう?名無しさん:02/01/09 17:14 ID:3Vk+OQNS
質問です。
のっぺりとした球形の惑星(構成物質は単一として考えて下さい)に、その惑星の直径よりも1メートル程大きな金属
の輪を回したら、釣り合って浮きますよね?
808 :02/01/09 17:20 ID:???
回る速度によるんじゃない?
809ご冗談でしょう?名無しさん:02/01/09 17:24 ID:xrx5Upd7
実際には地殻の厚さにムラがあるからちょっと厚めの地殻がある部分に
落っこちて終り。ただどんな場合でも中空のどこかに落ちていって宙ぶ
らりんになることはない(アーン・ショウの定理)。
810807:02/01/09 21:15 ID:dvCaCwMN
>808
すいません。言い方が悪かったです。回転させるのではなくただ
浮かせたいです。
>809
てか、深海がある場所ではアウトですね。
それは、他の何物もない真空中でもですか? もしくは、薄い金属
の球で覆っても浮いている状態は続かないのでしょうか?
自分でも調べてみます。>アーン・ショウ
811ご冗談でしょう?名無しさん:02/01/09 21:28 ID:???
>>810
 回転していないなら、浮いている位置は、不安定。
812ご冗談でしょう?名無しさん:02/01/09 22:09 ID:ACbMDwmF
検索したけどわかりませんでした。
プランクの作用量子についてだれか説明してください。
813ご冗談でしょう?名無しさん:02/01/09 22:09 ID:/L6p+5og
J.J.Thomson が比電荷の測定をしたときに、金属の箱に電子を飛ばして、その電荷を測定したそうですが、電荷の測定って一般にどうやるんでしょ。
814ご冗談でしょう?名無しさん:02/01/09 22:22 ID:???
>>813
みりかん
815ご冗談でしょう?名無しさん:02/01/09 22:24 ID:???
>>812
前期量子論で検索して下さい
816ご冗談でしょう?名無しさん:02/01/09 22:31 ID:???
何処かに、過去に実施された「大気圏外の核実験」の映像ありませんか?
どうやら、超新星におけるガスの膨張過程とそっくりらしいのですが。
817813:02/01/09 22:35 ID:/L6p+5og
>>816
あ、比電荷の測定じゃなくて、陰極線の偏向実験をしてきたときです。すんません。
比電荷はミリカンだおね。>俺
818:02/01/09 23:59 ID:ay6F1goZ
Aを横軸に、Bを縦軸にとり、Cをパラメーターとして、実験結果を図示する
という場合のCの表し方を教えてください。
819ご冗談でしょう?名無しさん:02/01/10 01:11 ID:???
>>818 ???
820818:02/01/10 01:22 ID:Rgsu/TDn
>>819
横軸に磁石への電流を、縦軸には起電力をとり、回転速度をパラメーターとして、
実験結果を図示する

という問題があるんですが
821ご冗談でしょう?名無しさん:02/01/10 02:28 ID:???
>>820
そんままプロットしていいんでないかい?
822ご冗談でしょう?名無しさん:02/01/10 02:35 ID:WqZ5q0HF
>>816
「大気圏外の核実験」ってやられているのですか?初めて聞いたけれど、、、
823ご冗談でしょう?名無しさん:02/01/10 03:09 ID:???
かなりの高度(成層圏かな?)での核実験で、爆発に伴い発生した強力な電磁波により
通信機器に影響が出たらしく、以後の対電子装置のための電磁パスル兵器開発の
きっかけになったとかいう話は聞いたきがするなぁ。
もちろんPTBT以前の実験だと思うけど。
824ご冗談でしょう?名無しさん:02/01/10 15:32 ID:08+f99Fn
火星の太陽定数はいくつですか?
火星の放射平衡温度を示す式と説明をお願いします。
火星の放射平衡温度を求めるには?(計算の途中にも単位をお願いします)

いろいろとすいません・・・。
825ご冗談でしょう?名無しさん:02/01/10 16:11 ID:???
>>820
一つの図に収めようとしなきゃいいんじゃん
826ご冗談でしょう?名無しさん:02/01/10 16:28 ID:G3cea9xQ
っs
827ご冗談でしょう?名無しさん:02/01/10 16:31 ID:975ubqqc
>>824
宇宙だから地球・太陽の平均距離と火星の場合とで計算していいんでない?
んでそれから火星への入熱と火星表面からの放射とがバランスする温度を計算
828ご冗談でしょう?名無しさん:02/01/10 16:38 ID:g7Ej8J91
院で量子関係の研究をしたいと思っています。
どの大学がよろしいでしょうか?知ってる方教えてください。まだ1年生なんです
けど、今の大学のままか他大学に行くべきかちょっと気になります。
829ご冗談でしょう?名無しさん:02/01/10 18:04 ID:???
>>828
1年生ならまずは勉強だけしとけ
830ご冗談でしょう?名無しさん:02/01/10 18:32 ID:???
>>828
 東海大学へ
831752:02/01/10 18:59 ID:???
遅ればせながら、ありがとうございます。>762
832K:02/01/10 20:22 ID:fw781GMT
ものを燃やしたとき、スペクトルを見ると黒い線がはいるのって、
どういう原理ですか。
833ご冗談でしょう?名無しさん:02/01/10 20:27 ID:???
>>832
萌えてる物質がその波長の電磁波を九州してるんぢゃネーノ?
834ご冗談でしょう?名無しさん:02/01/10 21:36 ID:YwPdRzE+
あの824の質問した私ですがこの問題文系の物理のレポートの問題で
もうチンプンカンプンなんです。
もうちょっと詳しくお願いします(;Д;)
835ご冗談でしょう?名無しさん:02/01/10 22:15 ID:???
>>834
じゃあ対話式で行こうか.
太陽定数の定義は?わかんなかったら検索で調べて.
836ご冗談でしょう?名無しさん:02/01/10 22:46 ID:???
>>834
おいもうみんな寝ちゃうぞ
こういうのは相手してれる暇人がいるうちに集中してやっちゃえ
837K:02/01/10 23:06 ID:9m86O9o9
>>833
なるほど、じゃあ、その周波数の整数倍のとこにはやはりうすく線がはいると?
838ご冗談でしょう?名無しさん:02/01/10 23:44 ID:???
>>837
なんで整数倍が出てきたんだ??
839ご冗談でしょう?名無しさん:02/01/10 23:51 ID:???
>>837
整数倍じゃなくてもっと上の状態間の遷移に対応した波長だろ
840ご冗談でしょう?名無しさん:02/01/11 00:16 ID:???
多光子吸収されてたりして。
841K:02/01/11 00:51 ID:HeIWNcRH
>>839
そうなるのか。
つまり、吸収と言っても通常の物質の吸光のようなものを考えてはいけないのですね。
842ご冗談でしょう?名無しさん:02/01/11 02:52 ID:???
数理工学関連の話題もOKの板を立てました
新板に遊びに来てください
http://jbbs.shitaraba.com/study/18/
843グローバリー:02/01/12 02:45 ID:i2VYWqls

巨額詐欺の疑いのある、グローバリーについて

掲示板を拝見されてるの皆様、新年あけましておめでとうございます。
掲示板の趣旨とは直接関係ない話で申し訳ないのですが、この世の中では許せない事があります。

http://www.max.hi-ho.ne.jp/sakimono/index.htm

この会社はありもしない儲け話をでっち上げ、巧みに客の財産を聞き出し、全財産を巻き上げます。

2002年になりましたが、一向に改善する気配すらなく、悪質化は進む一方です。
この会社の営業は世間の皆様の迷惑になっています。

それだけではなく、人を騙して破産に追い込み、騙された人が仕方なくグローバリー社員を
殺人する事件も実際に発生し、新聞ザタになっています。

http://www.mainichi.co.jp/news/selection/archive/200001/24/0124e038-400.html

みなさんもこちらの掲示板に投稿し、悪徳会社に騙される不幸な人が増えないようご協力お願いしま
す。


http://messages.yahoo.co.jp/bbs?action=q&board=8745

また、この書き込みを見て賛同頂ける方はこの内容をコピーしていろんな掲示板に書き込みをお願いします。
844ご冗談でしょう?名無しさん:02/01/12 06:43 ID:BeeaIkD8
理科の実験でやったんだけど、ガリレオのやった実験って、本当は同時につかないて本当なの?
845 :02/01/12 09:48 ID:9IyomEr4
http://choco.2ch.net/test/read.cgi/news/1010794881/
どなたかここの1さんを説得させてくれませんか?
846ご冗談でしょう?名無しさん:02/01/12 09:58 ID:???
>>844
ピサの斜塔での実験ことかね?
空気抵抗の影響が無視できないかぎりは当然だろうね。
847ご冗談でしょう?名無しさん:02/01/12 12:52 ID:a1AYl1Mg
電磁波は干渉します。
ではもし、地球上に存在する全ての電磁波の方向、位相、振動数などがランダムだったとき、
全ての電磁波を重ね合わせて積分してやったらそれは0になりませんか?
そうしたら地球上は暗闇になりませんか?
848ご冗談でしょう?名無しさん:02/01/12 13:13 ID:???
>>844
空気抵抗があるから。
849ご冗談でしょう?名無しさん:02/01/12 14:32 ID:???
何でホーキングはまだ生きてるの?
大学院の時に筋ジストロフィーを発病して、もう、60歳だ。
普通、この病気にかかったら10年持たないはずだぞ。
850ご冗談でしょう?名無しさん:02/01/12 14:33 ID:???
電磁波(輻射)のエネルギーを、熱以外(化学・電気)のエネルギーの
形態に変換する技術の効率って、今どのくらいなんでしょうか?
太陽光程度の波長レンジの話で御回答願います。
後、参考程度に葉緑体におけるこの効率はどれぐらいなんでしょうか?
さらに、この変換効率は理論的には100%を達成できるのでしょうか?
851ご冗談でしょう?名無しさん:02/01/12 15:50 ID:???
>>849
10年毎に部品を交換しているので大丈夫。
次のメンテは2003年です。
852ご冗談でしょう?名無しさん:02/01/12 15:54 ID:y+yEUIkR
>849
そこがホーキングのすごいところでしょ!?
853ご冗談でしょう?名無しさん:02/01/12 16:11 ID:???
>>849
噂では彼の体の80%は既に機械だそうです。
854ご冗談でしょう?名無しさん:02/01/12 18:33 ID:lGxD2Rla
自然界のもっとも基本的な構成粒子とその性質を教えてください。
855ご冗談でしょう?名無しさん:02/01/12 18:34 ID:???
MOPACについてはこの板で訊いても構わんですか?
856854:02/01/12 18:37 ID:lGxD2Rla
あと複合粒子は基本粒子からどのように組み立てられているのですか?
857ご冗談でしょう?名無しさん:02/01/12 19:20 ID:???
>>854 Particle Data Group によくまとめられています。
http://pdg.lbl.gov/2000/contents_tables.html
858ご冗談でしょう?名無しさん:02/01/12 19:38 ID:MC8xBcU5
この板でいいのかな。
アメリカの月面着陸はうそ?
TVでやってました。
859ご冗談でしょう?名無しさん:02/01/12 21:28 ID:???
>>854
コレ→(゚д゚)
860素朴な質問:02/01/13 00:17 ID:K+m+7wi7
初歩的なことを聞きたいのですが、
(バカにしてくれてもいいです)
ピストルの弾を空に向けて撃って、地上に落ちてくる速度は
どのくらいですか?
中学か高校で教えてもらったのは確か撃ったときと同じ速度
だと聞いたようなきがするのですが。。。
861ご冗談でしょう?名無しさん:02/01/13 01:05 ID:???
>>860
真空中なら正しい
862フヘフヘフヘ ◆XYN2P2Tw :02/01/13 13:49 ID:77+vEyvF
こんばんは。今までレポートしてて疑問に感じた事があったので質問させてください。

1オングストロームは、一般的に 1 / 1億cm と説明されますが、なんでセンチを基準に説明するんでしょうか。
つまりなぜ1オングストロームを 1 / 100万m と説明しないんでしょうか?
単位の基準は、cmじゃなくてmなので、1 nm = 10^-9 m のようにメートルで説明するのが普通と思ってしまうんです。。
863ご冗談でしょう?名無しさん:02/01/13 13:57 ID:???
>>862
昔は CGS のほうがよく使われていた名残でしょう。
そもそもオングストロームなどという単位を使う時点で
MKS に拘ってもしょうがない気がしますが?
864フヘフヘフヘ ◆XYN2P2Tw :02/01/13 14:24 ID:???
>>863
ありがとうございます分かりました。
で、本当ですか?
865ナノ相対論:02/01/13 15:05 ID:???
相対論の初歩的な疑問です。

光は2点間を最短距離で伝わると何かで読んだことがありますが、
空気と水の境界で光は曲がります。
プリズムでも屈折します。
物質空間中を直進するのは光ではなくて
ニュートリノやアルファ線のようにおもいます。

また、屈折とは回り道をすることですから、
結局光よりもニュートリノが速く情報を伝える
ことだってあるはずです。

このょうに物質空間内で相対論の基準速度に
光を用いることが不適当であるとすれば、
ニュートリノのような理想素粒子を新たに設定し、
その限界速度を光速度Cの代わりに使っているのでしょうか。
なんか思いっきり勘違いをしているようで恐縮です。
866DQN質問でごめんなさい:02/01/13 15:14 ID:L3tt9y1K
∇×∇×Bを発散(div)を使って書き直すとどうなりますか?Bはベクトル場です。
867ご冗談でしょう?名無しさん:02/01/13 17:33 ID:D7Tn0MfK
>>865
最小作用原理を勉強しましょう
(屈折=回り道ではありません)
868見えない世界からの光:02/01/13 17:48 ID:???
>>865

「結局光よりもニュートリノが速く情報を伝える
ことだってあるはずです。」

>> 「『太陽ニュートリノ』がその霊です。また、屈折とは回り道をすることです。」
869ご冗談でしょう?名無しさん:02/01/13 17:50 ID:???
>>866
rot rot = ? は分かるのか?
870ご冗談でしょう?名無しさん:02/01/13 18:04 ID:4rvGHHhT
まずトイレットペーパーを転がるようにテーブルの上に置きます。
真横からみて円になります。
その円周の1点にペンで印をつけ、さらにその印と円の中心点を
結ぶ線上にもう1点印を付けます。この2個めの印は
トイレットペーパーの芯につけることにします。
そして、点を円の真下に来るようにして、トイレットペーパーを
一周させます。
すると、2つの点も一周しますよね
同じ距離で一周すると言う事は
それぞれ二つの円(トイレットペーパーとその芯)の円周は
同じといえるのでしょうか?
よろしくお願いします。
871素朴な質問:02/01/13 18:15 ID:K+m+7wi7
>>831
ありがとうございました
872ナノ相対論:02/01/13 20:36 ID:???
ご返事ありがとうございました。
さっそく過去ログの「最小作用原理について」を読ませてもらいました。
「光は2点の最短距離ではなく、最速経路を伝わる」
ですね。
そして、
・光とニュートリノでは最速経路が異なり、物質中では
 一般にニュートリノのとる経路の方が短く、そして速い。
・たとえ物質中の事象であっても、相対論の式には真空中の光速度を用いる。
こんなとこでしょうか。

伝播するモノによって最短最速経路が異なるとなると、
なんだか√(X^2+Y^2)の距離空間の感覚が揺らいできそうです。
光にとっての距離空間ってかなり歪んでいるんですね。
873ご冗談でしょう?名無しさん:02/01/13 21:13 ID:???
物質中の光は吸収放出を繰り返して進んでいるようなものだからそんなの当たり前
874ご冗談でしょう?名無しさん:02/01/14 16:47 ID:???
確率と時間に関するこんな話を聞いたんですが。あっているのでしょうか?
有名な「100万年生きる3匹の猿と動きつづけるワープロを用意し、猿にキーボードを叩かせればシェイクスピアの
文学を創りあげることも可能」という話の延長です。
我々が死んで地球がなくなってから、宇宙が消えなかったらこの宇宙内で「時間が永遠に存在」します。
また宇宙が消えたら無という存在内で「時間が永遠に存在」します。
よって、またこの宇宙と同じ場所ができて太陽系や地球、今生きてる我々が生まれる
ことも(時間が無限なので)あるらしいんですが、どうなんでしょうか?
875ご冗談でしょう?名無しさん:02/01/14 18:18 ID:???
>>874
 あるでしょ。
876ご冗談でしょう?名無しさん:02/01/14 19:57 ID:z8hv0UAG
質問では無いのですが、一人の少年が、将来あやしい宗教に
はまるか、科学に関心を持つかの瀬戸際なので助けてあげて下さい。
私をはじめニュース板の住民では説明しきれて無いようです。
「@@@大発見!!運命は決まっていた!!@@@」
http://choco.2ch.net/test/read.cgi/news/1010905041/l50
877Surely you are joking, Mr.Nanacy:02/01/14 21:33 ID:/dtfz0P4
>>874
自分で計算してみ。
878ご冗談でしょう?名無しさん:02/01/14 23:56 ID:???
>>873
ハァ?
879ご冗談でしょう?名無しさん :02/01/15 01:39 ID:RnT4XKR2
電磁気学の教育において「応力」が軽視されているのはなぜですか?
ローレンツ力はフレミング左手の法則に注意をそらせておいて、
「”何故”横向きに力を受けるか」を説明していないように思われます
電荷が電場から受ける力 など電荷は自ら電場を持っていないように考えてしまう
人もいると思いませんか?
880ご冗談でしょう?名無しさん:02/01/15 02:09 ID:bqkB5kLY
フレミングの法則やローレンツ力は、電荷が動くと磁場が発生することが原因である。
881ご冗談でしょう?名無しさん:02/01/15 02:13 ID:???
では、地場の場合、「”何故”横向きに力を受けるか」を説明して下さい。
私には分かりません。

電場に関しては単にテスト粒子の考え方を説明すれば良いだけでは?
882ご冗談でしょう?名無しさん:02/01/15 02:19 ID:qLQL62lN
ケミカルポテンシャルって何?
講義では小さい箱と大きな部屋があって、その間を粒子が行き来する、
ってゆー感じの絵を書いてたけど、超伝導とかで、固体の中に大きい部屋と
小さい箱があるの?この場合の粒子は電子?
883ご冗談でしょう?名無しさん:02/01/15 02:22 ID:???
磁束が密にならないほどエネルギーが安定するので、フレミングに従う横向きの方向に導線が移動すると安定するのでは?
紙に静磁場と誘導磁場の磁束書いてみると分かりやすいんじゃないかなあ。
884ご冗談でしょう?名無しさん:02/01/15 02:27 ID:???
ケミカルポテンシャル=フェルミエネルギー
885ご冗談でしょう?名無しさん:02/01/15 02:37 ID:???
ボゾンにフェルミエネルギーは存在しません。
886ご冗談でしょう?名無しさん:02/01/15 02:59 ID:34/86W7L
高校のころ物理なかったんだけども、
速度vのvみたいなのって何の略でなんて発音すんの?
v−tグラフってヴイティグラフって読んでいいの?
887ご冗談でしょう?名無しさん:02/01/15 03:04 ID:???
>886
ベロシチイの略。ベロシチイをタイムでプロットした図と読む。
888ご冗談でしょう?名無しさん:02/01/15 03:10 ID:34/86W7L
>887
深夜に素早いレス有難う。
じゃベロシティは速度って意味で、
v0はベロシティゼロ、v’はベロシティダッシュでよいの?
889ご冗談でしょう?名無しさん:02/01/15 03:12 ID:???
v0はベロゼロ、ベロチョンと呼ぶ。
890ご冗談でしょう?名無しさん:02/01/15 03:33 ID:34/86W7L
初速度10(m/s)、加速度2(m/s^2)
これってどんな単位なんですか。
ミリセカンド?それとも一秒間10メートル進むってこと?
加速度の後ろのs二乗はどういうこと?
891ご冗談でしょう?名無しさん:02/01/15 04:05 ID:???
>>881
>「”何故”横向きに力を受けるか」
電荷 q の静止系に移ってみると、もともとの磁場 B はローレンツ変換に
よって横向きの電場 E に変わります。この電場 E によって横向きの
力を受けることになります。

もっとも、なぜ電磁場のローレンツ変換があのような形(一般形は教科書参照)に
なるのかは、究極的には真空中の光速が一定という原理に帰着されますが、
それが成り立つ理由はまだ誰も知りません。そういう意味では、なぜ横向きの
力を受けるかは誰も知らないともいえます。
892 :02/01/15 04:32 ID:GwnwP7T/
   /::::::::::::::::::::::::ヽ        _,..-‐‐-..,,,
  l::;;-‐‐-:;;::::::::::::ヽ//-‐,,__ /:::::::::::::::::::::ヽ
  l:l    ヽ:::::::::::::::::::::::::::::::::::::::::::::::::::::::::::::::ヽ
  ヽ   /   :::::::::::::::::::::::::::::::::::::;-'^~~^'‐;;:l
   ~ヽ/      :::::::::::::::::::::::::::::::ヽミ   .ll
    / /て^ヽ   ::::::::::::::::;;;;;;;:::::ヽ  ,.ノ
    /  |o ゝ,_ノ|     ::/^'ヽヽ::::::l'^~
  ‐/-,, ヽ( )_,,ノ      |ゝ.,_ノ o.|:::::l
   l  ~^''     `‐'   ヽ..,,_( )ノ  :l
  '''l^^~~~   (         -‐‐‐--l-
   ヽ、 ,,,,   `'''''^''''~    ~^'‐..,,_/
    / (:::::}         ,,,,   イ~''
    l:  ~~         {:::::)  ::l
   l:              ~~   l
   l、                 l>
   /^‐-,,____,,,,,,,,..................,,,,,,,__,,,.--ヽ
   ~‐‐'~             ^'‐‐~
893ご冗談でしょう?名無しさん:02/01/15 05:14 ID:j4cLtmKI
>>870
芯だけ取り出して一周させると同じ距離ではないでしょ(当たり前だけど)

まあ、言いたい事はわかるような気がする。
勝手に推し量って回答するならば、それは「すべる」からです。
894ご冗談でしょう?名無しさん:02/01/15 05:16 ID:j4cLtmKI
>>889
チョトワラタ

>>888
発音する時はヴイゼロ、ヴイダッシュで良いです。
895ご冗談でしょう?名無しさん:02/01/15 05:17 ID:???
>>890
>初速度10(m/s)、加速度2(m/s^2)
>これってどんな単位なんですか。
>ミリセカンド?
違います。ミリセカンドはmsです。
>それとも一秒間10メートル進むってこと?
そうです
>加速度の後ろのs二乗はどういうこと?
(m/s)/s――つまり1秒間に速度が何m/s変わるかってこと。
896ご冗談でしょう?名無しさん:02/01/15 05:28 ID:j4cLtmKI
>>890
m/s → メートル パー セカンド
(1秒につき10メートル進む;秒速10メートル)

m/s^2 → メートル パー セカンド2乗
(1秒につき速度が2m/s大きくなる。例の場合1秒後12m/s、2秒後14m/s)

「パー」は分数の横棒と同義(s分のmと思って可)

ちなみにミリセカンド(ms)は1000分の1秒
897ご冗談でしょう?名無しさん:02/01/15 05:33 ID:j4cLtmKI
>>895
スマソ。かぶった。

ところで、secondってセカンドって表記が多分本来の発音に一番近いんだろうけど
何か「セコント」って発音する人が多いような気がする。(特に、年配の研究者)

俺の気のせいか
898ご冗談でしょう?名無しさん:02/01/15 12:20 ID:FyUcQ7tj
>>895
>>896
すんごい感謝。
899ご冗談でしょう?名無しさん:02/01/15 19:28 ID:m6TrnChC
ガスバーナーで空気取り入れ口からガスが逆流しないのはなぜですか?
900ご冗談でしょう?名無しさん:02/01/15 20:18 ID:Yuy93Z+Q
〉〉899
ガスの流れが運動量を持っているからでないの?
逆にガスと空気層との摩擦が、空気を出口側に引き込むんだと思うけど。
(分かる人、詳しくお願い。スマソ。)

ガスと言えば疑問なのが、ライター用のガス充填ボンベ。
ボンベ側もライター側も液化ガスが入っている。
→って事は、ガスの蒸気圧はいっしょ。
 (むしろ、ボンベ側のほうが温度が下がって低いはず)
なのになんでライターに流れ込むの?

他のガス(窒素とか?)で与圧してるのかな?
901ご冗談でしょう?名無しさん:02/01/15 20:33 ID:uFVSOQiJ
太鼓やタンバリンのような筒状の口に膜をはったものの振動モードは
どんな風になるのでしょうか?
水面に水滴を落としたときの波紋みたいな感じでしょうか?
四角い板状のものだとXYの2次元のモードになりますよね?
902ご冗談でしょう?名無しさん:02/01/15 22:15 ID:1fQ8VUnh
>>899 >>900
ベルヌーイの定理により、流速の早い流体側では圧力が低くなるので
空気がガス側に引き込まれる。
903高校生:02/01/15 22:15 ID:BhJnGv9K
高校生 :02/01/15 22:05 ID:BhJnGv9K
4次元空間では時間軸が加わるってほんと?
904ご冗談でしょう?名無しさん:02/01/15 22:20 ID:???
5次元空間ではどこでもドアが加わります。
905ご冗談でしょう?名無しさん:02/01/15 22:20 ID:1fQ8VUnh
>>900
只単にボンベ側の圧力が高いから。
ガスを入れ続けるとそのうち平衡に達する。

また、ボンベ内の液化ガス(ブタンなど)が少なくなると
圧が弱まり、ライター内にガスが入っていかなくなるが
空気中に放出すると、まだ結構な勢いで出る。
(だから穴あけ器でガスを抜いてから捨てる)
906ご冗談でしょう?名無しさん:02/01/15 22:27 ID:1fQ8VUnh
>>901
円の直径を半波長とする定在波となる。

ギターの弦の振動をぐるっと半周させた円を想像すると良い。
907ご冗談でしょう?名無しさん:02/01/15 22:45 ID:oj1tuGkp
僕の意識は一瞬で十億億光年先まで飛ぶのですが、これって
相対論違反ですよね。何か僕にぴったりの理論ください。
908 :02/01/15 22:46 ID:???
>>901
たとえば
ttp://www5b.biglobe.ne.jp/~abekan/Math_thr/wave2d_f/
膜の波動方程式を解く問題は大学院入試に出たりします。
909901:02/01/15 23:13 ID:uFVSOQiJ
>>906,>>908
ありがとうございました。
908のページは難しくて私には理解できません。
でも、波動方程式を満たすいくつかの振動の様子を動画で見れるなんて、
感動しました。
これをきっかけに勉強します。
910化学板から流れ:02/01/15 23:41 ID:???
mentaiサーバがこの時間帯ダウンします。なんでなんだぁぁ。
911ご冗談でしょう?名無しさん:02/01/15 23:41 ID:???
>>905
ライターに液化ガスが流れ込むのは当然「只単にボンベ側の圧力が高いから」
と思うが、それは >>900 の言うように窒素か何かで与圧してるって事になる?
912ご冗談でしょう?名無しさん:02/01/15 23:48 ID:???
ライターのガスボンベのパッケージをみてみたけど、ブタンNET40gとしか書いてないね。
まあ、窒素が入っていてもそんな事パッケージには書かないか。。。
でも、やはり窒素か何かは入ってる気が擦るな。
913ご冗談でしょう?名無しさん:02/01/15 23:59 ID:???
>>905
ボンベの残量が少なくなると圧が弱まるのは何故?
温度が一定なら蒸気圧も一定のはずだし。
これって、窒素ガスなどで与圧してある証拠かな?
あるいは、液化ガスの温度が下がる速度が速いだけ?
914ご冗談でしょう?名無しさん:02/01/16 00:01 ID:???
>>907 理論を作ってあげるので、十億億光年先の状況を詳細にレポートして下さい。
915  :02/01/16 00:10 ID:NM8j3ZBU
水中って、真空中の3分の2のスピードで光は進むんですよね。
水中で光速以上に(水中の)スピードを出して進む物体から
スタート地点を観察出来たとしたらどんな光景になるんですか?
916ご冗談でしょう?名無しさん:02/01/16 00:14 ID:???
>>915
後方ではなく前方にスタート地点が見える。しかも、時間反転した光景が見える。
917  :02/01/16 00:24 ID:NM8j3ZBU
ファイナルアンサー?
918ご冗談でしょう?名無しさん:02/01/16 00:27 ID:???
ファイナルアンサーです。
919  :02/01/16 00:40 ID:???
それって、もしスタート地点から水中を光速以上で
進んでて、ある地点でスピードを落としたら、
しばらくしてスタートする自分がやってくるのが
見えるんですかね? そのうち自分に再会するんでしょうか?
920ご冗談でしょう?名無しさん:02/01/16 06:03 ID:52FJ3Kzr
>>911 >912
入っていない。入っていれば通常書く。

こう考えたらどうか。
窒素ガスの代わりにブタンガスそのもので与圧する。
921ご冗談でしょう?名無しさん:02/01/16 06:07 ID:52FJ3Kzr
ちなみに窒素ガスなどで与圧をかけるのは不揮発性の物質。
殺虫剤などではLPガスを使う。(安いから)

余談だが、殺虫剤スプレーが火炎放射器となるのは
このLPガスが燃えるため。
922ご冗談でしょう?名無しさん:02/01/16 06:10 ID:52FJ3Kzr
>>913
ボンベ内の空間が大きくなるから。
923ご冗談でしょう?名無しさん:02/01/16 06:20 ID:52FJ3Kzr
>>921 >不揮発性 (自己レス)
いや、すまん、間違えた。
常温1気圧で気体にならない物質にしておいてくれ。
924ご冗談でしょう?名無しさん:02/01/16 07:19 ID:???
>>920-923
問題点(>>900とか>>913)をもう少し理解して下さい。
ボンベのブタンガスによる与圧では、ライターの充填に不足するのでは?
というのが問題点。
925ご冗談でしょう?名無しさん:02/01/16 08:02 ID:ypamO+xX
>>924
ライターの火がつかない→内圧がほとんどない
→ボンベの方が圧力が高い
でしょ?
926ご冗談でしょう?名無しさん:02/01/16 08:09 ID:ypamO+xX
PV=一定なので容器内の空間が大きくなれば
内圧は当然下がる。

スプレーの出なくなった缶を踏み潰して小さくすれば
また少し出るようになるよ。
927ご冗談でしょう?名無しさん:02/01/16 08:17 ID:ypamO+xX
あ〜念のため。
ボンベ(ないしライター)内の空間が大きくなるというのは
液体のブタンが減るから。
928ご冗談でしょう?名無しさん:02/01/16 08:26 ID:???
>>925
ライターの液ガスが半分でも充填出来るよ。
このときのタンク内の圧力はボンベ内と同じと思うけど。
929ご冗談でしょう?名無しさん:02/01/16 08:39 ID:ypamO+xX
>>928
その「同じ」という根拠がわからん。

まあ、いずれにせよ、ライターのタンクの断面積とタンクの断面積の
大きさの違いに原因がある。

山用のガスボンベってひどくズングリムックリに作ってあるでしょ?
930ご冗談でしょう?名無しさん:02/01/16 08:40 ID:???
>>926 の主張はむしろ窒素ガスか何かの存在を伺わせるが…

ボンベ内の液化ガスがほとんど満タンの時(空洞小)でも半分の時(空洞大)でも、
液化ガスからのガスの供給があるから、ガスの分圧は蒸気圧と同じはず。

よって、ボンベ内が全てガス成分なら
ボンベ内の残量に関わらず「出だし」のガス圧は変わらないはず。

もし、窒素ガスがあれば窒素の分圧をP、体積をVとしてPV=一定より
窒素の分圧は、ボンベ内の液化ガスの残量が少なくなるほど(Vは大きくなり)
小さくなる。トータルとして全圧力は小さくなっていく。
931ご冗談でしょう?名無しさん:02/01/16 08:43 ID:ypamO+xX
断面積の小さなピストンと大きなピストンに水をいれ、
ゴム管でつなぎました。

水を加熱するとピストンはどちらに動くでしょうか?

という問題。
932ご冗談でしょう?名無しさん:02/01/16 08:43 ID:???
>>929
同じ物質の液体と気体の共存する系では、気体の圧力(分圧)は温度にのみ依存する
と言うのはOK?
933ご冗談でしょう?名無しさん:02/01/16 08:48 ID:???
>>931
全体を加熱するんだよね?
どっちに動くかってこの条件で決定可能?
934ご冗談でしょう?名無しさん:02/01/16 08:51 ID:???
>>929
タンクの体積とかじゃなくて断面積?
何の関係があるの? 液化ガスの蒸発効率を高めるためとか?
935ご冗談でしょう?名無しさん:02/01/16 08:57 ID:???
>>929
あ゛! もしかしてライターのガスがタンク内では「液化ガス」で、
炎になる直前に気体になるというのを存知じなかったのですね。
936ご冗談でしょう?名無しさん:02/01/16 09:34 ID:4/hCvpuq
なんで同じ大きさの力なのに等加速するんだ。
だったらものすごい遠くの重力に引っ張られたら無限に速度が上がるのか
937ご冗談でしょう?名無しさん:02/01/16 09:36 ID:???
?? 何の問題?
938900:02/01/16 09:56 ID:1jpPZrRz
900@ガスライター話の発起人です。

言い出しっぺとして、話を整理しましょう。
 ライター側:少量の液化ガスと、残りは気化ガスで満たされている。
 タンク側:大量の液化ガスと、残りは気化ガス(仮定)
とします。
内部圧力は、それぞれのガスの蒸気圧で決定される。
(『蒸気圧』で決まるから、気体の状態方程式は関係無いよ。念のため。)
さて。
この状態からライター側が一杯になるためには、
 @タンク側の気体容積が大きくなり、
 Aライター側の気体容積が小さくなる。
→気化熱によって、タンク側では温度が下がり、ライター側では上がる。
これは、ライター側の蒸気圧が大きくなる事を示し、すぐに止まってしまうのでは?

にもかかわらず、ライターをほぼ一杯にするまでガスが流れる。
これはタンク側の圧力が大きい事を示す。(仮説が崩れる。)
 →蒸気圧以外の圧力の原因がある。
こう考えたわけです。

なお、議論はあくまで『圧力』です。断面積は関係ありません。
こう考えてみてください。タンクとライターの接続部は、同じものなんだから、面積は一緒。
なのに、押し合う力に不均衡があるからライターに流れ込みます。
939mire:02/01/16 10:40 ID:MN+A2jRO
アルミのプレートはレンジに入れても乱反射を起こさないのに
なぜアルミ箔は乱反射を起こすのですか(@_@)
乱反射を起こす金属と起こさない金属の違いは何なのでしょうか
疎い私でも気になってしまいます おしえてくださいー
940素人です:02/01/16 10:46 ID:???
>>930
ボンベの中に窒素が入ってるんだったら
ライターの方にも窒素が行くのではないのですか?
結局同じ話かと・・・

的外れなこと言ってたらスミマセン(汗
941ご冗談でしょう?名無しさん:02/01/16 11:28 ID:???
液化ブタンに窒素が大量に解けこむ様な事がなければ良いのでは?
942ご冗談でしょう?名無しさん:02/01/16 11:52 ID:???
>>939
乱反射をするかどうかは、金属の種類というよりも形状に依存します。
複雑な形をしていれば乱反射し、波長に比べて十分に滑らかな平面であれば
単なる反射をします。
よって、平らなアルミ箔なら乱反射しません。
943ご冗談でしょう?名無しさん:02/01/16 12:21 ID:???
「ガスライター充填問題」ですが、化けの知識も必要かと思い化学板にも投げました。
http://mentai.2ch.net/test/read.cgi/bake/1009375603/258
944ご冗談でしょう?名無しさん:02/01/16 12:22 ID:???
↑すみません、ハイフン忘れました
http://mentai.2ch.net/test/read.cgi/bake/1009375603/258-
945ご冗談でしょう?名無しさん:02/01/16 12:51 ID:???
質問スレも、そろそろ新スレ誕生の時期かな?
今度は1に過去ログリンク付けようね
こんなかんじhttp://cheese.2ch.net/test/read.cgi/sci/1003082106/1-4
946900:02/01/16 13:12 ID:F4Q2FGhM
>>940さん
900@それ言ったらおいらも素人です。

>ボンベの中に窒素が入ってるんだったら
>ライターの方にも窒素が行くのではないのですか?
そこで思うのは。
ライターにガスを入れる時って、ライターを下にしますよね?

さっき確認しましたが、『共用ガスボンベ』とやらの箱
(※タンク自体では無い)にも、そういう絵が描いてあります。
(別に、『ライターを下に』と明言されているわけでは無いけど。)

ボンベの中の構造を見たわけでは無いけれど、これが、液化ガスが優先的に
導入される理由かな?と考えてます。

逆に、ライターを上にしてつないだら、ガスは入らんのだろか。。。

窒素(か、どうかはいまだ不明ですが。)が液化ブタンに溶け込むかどうかですが。
液化ブタンは油みたいなもん(?)だから。溶けない気がするけど???
確かに、化学やさんの意見は聞きたい。 (943=944様、感謝。)
947タノモー:02/01/16 16:33 ID:j9CujYCl
船体における応力・断面係数・モーメントの関係

σ(係数)
Z(断面係数)
M(モーメント)

σ=M/Zの関係について述べよ。

だれか解ける?
948 :02/01/16 16:36 ID:lCgLQR9b
σ=M/Z
949ご冗談でしょう?名無しさん:02/01/16 17:24 ID:6pyMYhsH
こっちにも書いとくか

もしボンベを上に,ライターを下に,って指定があるなら
やはり窒素かなんかで与圧してるんだよ
その状態は,ガス成分の平衡蒸気圧=分圧だから
液化ガスに溶けにくいガスも気相部にあらかじめいれておけば
気相部の全圧は蒸気圧プラス余分に入れたガスの圧力
その気相をライターに入れないためには必ずボンベの
出口を下にして使う必要がある
で,確認法だけど,いったんボンベの口を上にして気相部分の
ガス抜きをちょっとする
その後,もう一度ライターに入れられるかどうかを見る
どうよ?
950工房:02/01/16 17:40 ID:2gl1YaRQ
光は波長が短い方が屈折しやすいのはなぜですか?
951ご冗談でしょう?名無しさん:02/01/16 17:49 ID:???
>>947
σは材料内部に発生している応力じゃないか?
つーか機械・工学板じゃないか?
952ご冗談でしょう?名無しさん:02/01/16 18:13 ID:j9CujYCl
>>951
ありがとうございます
逝ってきます
953ご冗談でしょう?名無しさん:02/01/16 18:57 ID:+NUELsoo
ひょっとして900氏はボンベ内にブタンガスしか入っていないという前提なのか?

ボンベ内には当然元々入っていた空気があるので、
それが与圧をかけているだけのような気もするが・・・?
954ご冗談でしょう?名無しさん:02/01/16 19:27 ID:???
>>950
ガラスの場合には波長によって屈折率が変わるという「分散特性」がある。
屈折率:n = √(εμ) (ε = 比誘電率(波長によってかわる)、μ = 比透磁率(ガラスではほぼ常に1))
つまり屈折率には比誘電率εが関係する。

ここからは高校生には難しいけど…
比誘電率:ε = ε0(1+χe) (ε0 = 真空中の光の誘電率、χe = 分極率(波長によりかわる))
というわけで分極率χeが波長によってかわるのが問題。

分極率χeは電磁波の瞬間電界によって変わるが、
電界の変化よりすこし遅れるというヒステリシス特性(残留分極)があり、
光(=電磁波)のような並みの入力を与えると、
周波数の多い波に対してはその速い変化についていくことができず、
大きい値をとる。
よって波長が短いと分極率が大きくなり、比誘電率が大きくなり、
屈折率が大きくなる。
955工房:02/01/16 20:13 ID:Xs9Z4aB9
>954
ありがとうございます。

> 屈折率:n = √(εμ)   ←これが成り立つとはビックリしました。

後半は難しかったですが、
 波長が短いとχeが増大 → εが増大 → nが増大
ということはわかりました。
本当にありがとうございました。
956mire@レンジは便利!:02/01/16 20:55 ID:wHqKR3Ld
>>942さん
平らにするとアルミ箔も乱反射しないんですね!
初めて知りました ちょっと感動です今度試してみます

ところで乱反射が起きているときのあの火花は何が原因なのですか?
電子の衝突とかなのでしょうか(適当)
95717歳@大検合格:02/01/16 22:34 ID:1ERyCr1S
初歩的な質問でごめんなさい。。
等加速度直線運動の公式のうち

・v=v0+atはv-tグラフでいうvの値
・x=v0t+1/2at^2はv-tグラフでいう三角形の面積ということでいいんですか?

あとv^2-v0^2=2axが覚えにくいんで困ってます。
95817歳@大検合格:02/01/16 22:38 ID:1ERyCr1S
>>957
>>x=v0t+1/2at^2はv-tグラフでいう三角形の面積・・・

訂正 別に三角形に限りませんね(v0が0以外のときは台形だし)
物理はいちから始めたばかりなので何もわかりません。
959900:02/01/16 22:54 ID:ymrH/btJ
>>949 だよね。試せば、分かるか。ごめんよ。
>>953 ボンベの製造工程の事は頭に無かった。

ともあれ、大体答えは出たよね。みんなありがと。
960ご冗談でしょう?名無しさん:02/01/16 23:17 ID:CDRMu2hG
非定常摂動論で遷移確率が時間の次元をもってなすが
なぜ無次元じゃないんですか?
961ご冗談でしょう?名無しさん:02/01/17 01:09 ID:???
>>957
>あとv^2-v0^2=2axが覚えにくいんで困ってます。
そんなもん覚えるな!
962ご冗談でしょう?名無しさん:02/01/20 21:19 ID:???
時間によって固有状態が変わるから、遷移確率も変わります。
963ご冗談でしょう?名無しさん:02/01/21 23:42 ID:orNyaGYC
水素分子の結合性軌道と反結合性軌道をLLCAO-MO法をつかって固有関数を近似してそれで
Hatree−Fock近似をつかって求めたいんですが
結合性軌道は
E_b =E_1s +1\2(U_AA+U_BB)-t
とわかるんですが
反結合性軌道のエネルギー固有値を求めると
E_b =E_1s +(U_AA+U_BB)+t
となるらしいんですが どうして、1/2(U_AA+U_BB)まで変わるのかがわからないっす。

誰か教えておくんなまし
964ご冗談でしょう?名無しさん:02/01/21 23:48 ID:j3+2MJjK
★静電気が発生するメカニズムを教えてください★
プラスとマイナスだったら、擦れば擦るほど「中和」されていく。
ような気がしますが…
965ご冗談でしょう?名無しさん:02/01/22 00:23 ID:or1cjY+X
太陽では、水素にエネルギーが加わってヘリウムになるそうですが
ヘリウムから水素に変換することって可能なの?
将来的にできるようになる見込みってあるんですか?
このままいくと宇宙はすべて重い元素になってしまう気がするんですが?
初心者な質問でごめんなさい。
966ご冗談でしょう?名無しさん:02/01/22 00:33 ID:ADiWlTqk
>>964
そんな書き方じゃ誰も分からんよ。
まず、記号の定義から書きなさい。
967966:02/01/22 00:35 ID:???
間違えた。
964→963
968ご冗談でしょう?名無しさん:02/01/22 20:41 ID:???
>>963
LCAO法を使う限りハートレーフォック近似は使えない。
お前はもう氏んでいる。
969ご冗談でしょう?名無しさん:02/01/22 20:44 ID:???
>>960
エネルギーの固有状態が時間に依存するから。
970ご冗談でしょう?名無しさん:02/01/22 20:57 ID:???
age
971ななしさん:02/01/22 21:31 ID:???
受験生です。
物理の先生方
おしえてください

しつもん
センター試験物理IB第4問 問1と問2がわかりません

問1は問題文に”振幅が大きくなった場所”と書いてありますが
これは壁とマイクロホンの間に立つ定常波の腹のことですか?

問2は問題になっているのは”定常波”のはず
それなのになぜ@が正解なのですか?波の干渉と定常波は関係あるといえば
ありますがその程度の関連性で正解なのですか?
972ご冗談でしょう?名無しさん:02/01/22 21:38 ID:???

波の干渉=定常波じゃなくて、
波の干渉 ⊃ 定常波ではないのかね。
973ななしさん:02/01/22 21:48 ID:???
972先生
レス有難うございます。
問1の方はどうでしょうか?
あくまで高校生レベルですので
よろしくお願い致します
974ご冗談でしょう?名無しさん:02/01/22 21:51 ID:???
コヒーレント状態のこと?
975ななしさん:02/01/22 21:55 ID:???
974先生
レス有難うございます
すみません。先生方にお聞きするようなことではありませんね
申し訳ありませんでした
976ご冗談でしょう?名無しさん:02/01/22 22:14 ID:???
質問するなら問題文書くか問題文へのリンク貼るとかしねーと
誰も自分から探してなんてくれないと思うが。
977ご冗談でしょう?名無しさん:02/01/22 22:28 ID:???
976先生すみません
問題は下記URLで参照してくださいますようお願い致します。
http://con.oo.kawai-juku.ac.jp/nyushi/center/02/image1/412/11.gif
http://con.oo.kawai-juku.ac.jp/nyushi/center/02/image1/412/12.gif
978DQN大学生です:02/01/22 22:48 ID:VNrW4aK0
ベクトルの前につくdivとgradの違いがよくわかりません。
同じものではないと思うんですが・・・。
979ご冗談でしょう?名無しさん:02/01/22 23:02 ID:???
>>978
君の人生に関係ない。安心してティムポしごいとけ。
980ご冗談でしょう?名無しさん:02/01/22 23:22 ID:???
>>978
質問がDQNです。ベクトルの前にはgradはつきません。

grad f は関数fがどっちの方向にどれだけ傾いているかを示すベクトル場。
div V はベクトル場Vがどれくらい保存していないかを示すスカラー場。
981ご冗談でしょう?名無しさん:02/01/22 23:50 ID:un3XQjyb
昼間晴れたら 夜寒いのはなんでですかね?
982ご冗談でしょう?名無しさん:02/01/22 23:54 ID:un3XQjyb
>>968
LCAO法を使って混成させた軌道を使って、電子が同士のクーロンエネルギーを
平均場近似で書けばつかえるじゃん。
ちがう? やっぱ屍んでる?
983ご冗談でしょう?名無しさん:02/01/23 00:46 ID:???
>982
それはハートリーフォック近似とは言いません。ハートリー近似は、軌道の重なりがあまり無いとき2粒子の波動関数の積をとって、1粒子の存在確率を重みとして平均場近似つかって粒子の数だけあるシュレーディンガー方程式とポテンシャルの方程式を連立させて遂次近似するやつだろ。
で、ハートレーフォックは粒子の対称、反対称を考慮したやつ。
984ご冗談でしょう?名無しさん:02/01/23 00:55 ID:3JzmQzPo
プランク定数のdimensionは?
985ご冗談でしょう?名無しさん:02/01/23 01:01 ID:???
ていうか、おまえらこっち行け
http://cheese.2ch.net/test/read.cgi/sci/1011199123/
986ご冗談でしょう?名無しさん:02/01/23 01:05 ID:???
>>981
放射冷却の為。
雲が上空に有る時は地表からの熱が雲で反射されるが
雲が無い時は、そのまま宇宙空間に逃げて行く。

ちなみに、
冬の寒い日に車を建物のそばに一晩置いておいた時、
建物側には霜が付いていないのに
空が見える側には霜が付くのも同じ原理。

つ〜か↓こっち行け
http://cheese.2ch.net/test/read.cgi/sci/1011199123/
987ご冗談でしょう?名無しさん:02/01/23 01:50 ID:???
>>980
ベクトルの前にはgradはつけることはできます。
988ご冗談でしょう?名無しさん:02/01/23 01:54 ID:???
>982
それはパウリの排他率を考慮してないのかね。電子管相互作用考えたらのことも考慮がいるぞ。
ただ波動関数の線形結合で原子のポテンシャルだけを含むハミルトニアン挟めばええんとちゃう?1次摂動で。
989ご冗談でしょう?名無しさん:02/01/23 01:55 ID:???
>982
それはパウリの排他率を考慮してないのかね。電子管相互作用考えたらスピンのことも考慮がいるぞ。
ただ波動関数の線形結合で原子のポテンシャルだけを含むハミルトニアン挟めばええんとちゃう?1次摂動で。

990ご冗談でしょう?名無しさん:02/01/23 09:50 ID:???
>>987
スカラーのではなく、ベクトルのgradってなんだ?
定義を教えてくれ。
991990:02/01/23 09:57 ID:???
定義はいくらでも出来るから、
ベクトルのgradを使った具体例を教えてくれると有難い。

>>978 こんなと所で質立てるな!
992ご冗談でしょう?名無しさん:02/01/23 10:18 ID:???
993ご冗談でしょう?名無しさん:02/01/23 12:21 ID:RPi0fovN
>>991
裳華房流体力学の最後の付録に載っている。
994ご冗談でしょう?名無しさん:02/01/23 14:03 ID:???
  ∧_∧
 ( ・∀・) ニヤニヤ
 ( 1000 )
 | | |
 (__)_)
995ご冗談でしょう?名無しさん:02/01/23 14:12 ID:???
ヒソーリ
996ご冗談でしょう?名無しさん:02/01/23 14:35 ID:P/gkH66d
  ∧_∧
 ( ・∀・) ニヤニヤ
 ( 1000 )
 | | |
 (__)_)
997ご冗談でしょう?名無しさん:02/01/23 14:38 ID:???
  ∧_∧
 ( ・∀・) ニヤニヤ
 ( 1000 )
 | | |
 (__)_)
998ご冗談でしょう?名無しさん:02/01/23 14:40 ID:???
 サッ
)彡
999ご冗談でしょう?名無しさん:02/01/23 14:40 ID:???
(・∀・)アト1
!!
10001000 ◆h.gHK9LE :02/01/23 14:40 ID:???
                / ̄ ̄ ̄ ̄ ̄ ̄ ̄ ̄ ̄ ̄
         Λ_Λ <  コソーリ
        (´∀` )  \__________
  ∧ ∧   (___,,)_     
 (.,゚Д゚)/ ̄ ̄ ̄ ̄ /|     
 │ /∧ ∧    //||
 (/___(   ,,) _//  / ̄ ̄ ̄ ̄ ̄ ̄ ̄ ̄ ̄ ̄
 ||  ,,/  つ ||/   <  1000!!
 ||  (__丿  ||      \__________
10011001
このスレッドは1000を超えました。
もう書けないので、新しいスレッドを立ててくださいです。。。